Download as pdf or txt
Download as pdf or txt
You are on page 1of 249

Page 0 of 249

Mohammad Abdul Halim, Assistant Professor in Mathematics


Page 1 of 249

Engineering Mathematics

Mohammad Abdul Halim, Assistant Professor in Mathematics


Page 2 of 249

Mohammad Abdul Halim, Assistant Professor in Mathematics


Page 3 of 249

Chapter 01: Formation of ODE

Chapter 02: First Order ODE

Chapter 03: Higher Order ODE

Mohammad Abdul Halim, Assistant Professor in Mathematics


Page 4 of 249

FORMATION OF ORDINARY DIFFERENTIAL EQUATIONS

Differential Equations have been the key to unlocking the nature deepest secrets. Over 300 years ago, Sir
Isaac Newton invented differential equations to understand the problem of motion and he developed
Calculus in order to solve differential equations. Differential equations have been the essential tool for
analyzing the process of change whether in Physics, Engineering, Biology and any other field where it’s
important to predict how something behaves over time. Differential equations assume a basic knowledge
of Calculus to find its solution. The main objective of this subject is to formation of mathematical model
using differential equations and solving it in various techniques and observes the behavior of this solution.
Differential equation is mainly concerned with the qualitative aspects of solution. That is “How does
solution behaves”.
Examples:
dy dy u u
1. = f ( x) 2. = f ( x, y ) 3.x + y =u
dx dx x y
The first two equations contain ordinary derivative of one or more dependent variables with respect to a
single independent variable x and are known as Ordinary differential equation (ODE). The third equation
involve the partial derivatives of one dependent variable u and two independent variables x and y
respectively is called Partial differential equation (PDE).
Applications of First-order Differential Equations to Real World Systems:
Model Name Model Name
1. Cooling/Warming Law (Newton) 7. Draining a tank
8. Economics and Finance
2. Population Growth and Decay 9. Mathematics Police Women
3. Radio-Active Decay and Carbon Dating 10. Drug Distribution in Human Body
4. Mixture of Two Salt Solutions 11. A Pursuit Problem
5. Series Circuits 12. Harvesting of Renewable Natural Resources
6. Survivability with AIDS

Differential Co-efficient:
dy
Differential coefficient means the changing rate of y in terms of x.
dx
There are two types of differential coefficient such as:

1.Ordinary differential coefficient:


The differential coefficient of a function with respect to one independent variable is called ordinary
differential coefficient.
For Example:
dy d 2 y d 3 y dny
, , , ,
dx dx 2 dx3 dx n

Mohammad Abdul Halim, Assistant Professor in Mathematics


Page 5 of 249

2. Partial differential coefficient:


The differential coefficient of a function with respect to more than one independent variable is called Partial
differential coefficient.
For Example:
u u  2 u
, , etc
x y xy
Differential Equation:
An equation involving derivatives of one or more dependent variables with respect to one or more
independent variables is called a differential equation.

Example: For examples of differential equation, we list the following:


d 2 y dy
1. + +y=0
dx 2 dx
d 4u d 2u
2. + 5 + 3u = sin t
dt 4 dt 2
 2u  2u  2u
3. + + =0
x 2 y 2 z 2
u v
4. + =v
s t
Notation for derivatives / differential Coefficients:
1. Leibnitz notation:
dy d 2 y d 3 y dny
, , , ,
dx dx 2 dx3 dx n
2. Prime/Dash notation:
y, y, y, y iv , , yn
3. Newton Dot notation:
y, y, y ,......etc , Here dot means the time derivative.
4. Subscript notation:
ux , uxx , ut , utt ,................etc.
Order of Differential Equation:
The order of a differential equation (either ODE or PDE) is the order of the highest order derivative involved
in a differential equation. In other words, the order is a number how many times it is differentiated.
For example:
3
d2y  dy 
+ 5   + 4 y = 3e x
 dx 
2
dx
Above differential equation is of order 2.
Degree of Differential Equation:
The degree of a differential equation is the power or degree of the highest differential coefficient when the
equation has been made rational that means equations free from radicals and fractions.
3
d2y  dy 
+ 5   + 4 y = 3e x
 dx 
2
dx
The degree of the above differential equation is 1, since the power of the highest order derivative is 1.

Mohammad Abdul Halim, Assistant Professor in Mathematics


Page 6 of 249

Solutions of differential equations:


If any differential equation be satisfied by any defined and differentiable function, then the function is called
solution of the differential equation.
𝑑𝑦
Example: 𝑑𝑥 = 2𝑥 is satisfied by 𝑦 = 𝑥 2 𝑜𝑟 𝑦 = 𝑥 2 + 𝑐. So 𝑦 = 𝑥 2 𝑜𝑟 𝑦 = 𝑥 2 + 𝑐 is a solution of given
differential equation.
General Solution of DE:
The relation between dependent and independent variable containing n arbitrary constants which satisfies
an DE of order n is called its complete primitive or general solution. Geometrically general solution
represents a family of parallel curves.
For example:
𝑑𝑦
𝑦 = 𝑥 2 + 𝑐 is a solution of given differential equation 𝑑𝑥 = 2𝑥 .
Particular Solution of DE:
A particular solution of DE is one obtained from the primitive by assigning definite values to the arbitrary
constant.
For example:
𝑑𝑦
𝑦 = 𝑥 2 + 2 is a solution of given differential equation 𝑑𝑥 = 2𝑥 .
Singular Solution of DE:
A Singular solution of DE is special one which not obtained from the primitive by assigning definite values
to the arbitrary constant and it is free from arbitrary constant.
𝑑𝑦 2
For example: y=0 is the singular solution is the equation (𝑑𝑥 ) = 𝑦 .

Explicit solution: If the function 𝑦 = 𝑓(𝑥) is the solution of a differential equation of variables 𝑥 𝑎𝑛𝑑 𝑦,
then the solution is called explicit solution.
𝑑2 𝑦
Example: 𝑦 = 𝑥 3 + 𝐴𝑥 + 𝐵 is an explicit solution of 𝑑𝑥 2 = 6𝑥. where 𝐴 𝑎𝑛𝑑 𝐵 are arbitrary constants.
Implicit solution: If the function 𝐹(𝑥, 𝑦) = 0 is the solution of a differential equation of variables 𝑥 𝑎𝑛𝑑 𝑦,
then the solution is called implicit solution.
Example: 𝑥 2 + 𝑦 2 = 𝑐 is an implicit solution of 2𝑥𝑑𝑥 + 2𝑦𝑑𝑦 = 0. Where 𝑐 an arbitrary constant.
Classification of Differential Equation:
1. Classification based on type:
Ordinary Differential Equation (ODE):
An equation together with ordinary differential coefficient/derivative is called ODE.
Or
A differential equation involving derivatives of one or more dependent variables with respect to only one
independent variable is called an ordinary differential equation.
d2 y dy
Example: 1. 2
+2 + y =0
dx dx
3
d y d2y
2. 3 + 5 2 + 2 y = sin x
dx dx
Partial Differential Equation (PDE):
An equation together with partial differential coefficient/derivative is called PDE.
Or
A differential equation involving derivatives of one or more dependent variables with respect to more than
one independent variable is called a partial differential equation.
u v  2u  2u
Example: 1. + =v 2. + =0
s t x 2 y 2

Mohammad Abdul Halim, Assistant Professor in Mathematics


Page 7 of 249

2. Classification based on order:


1.First order
2.Second Order
3. Higher Order
3. Classification based on Linearity:
Linear ordinary differential equation: An ordinary differential equation of order n is called a linear
ordinary differential equation of order n if it does not contain,
1. the transcendental functions of dependent variable
2. the product of dependent variable
3. No product of dependent variables and its derivatives
4. The dependent variable and all its derivatives are of the first degree i.e. the power of each term
involving is 𝑦 is 1.
It can be expressed as
dny d n −1 y dy
a0 ( x ) n
+ a1 ( x ) n −1
+ ... ... ... + an −1 ( x ) + an ( x ) y = b ( x )
dx dx dx
where, a0 is not identically zero.
For example:
𝑑3 𝑦 𝑑𝑦
𝑑𝑥 3
+ 𝑥 𝑑𝑥 − 5𝑦 = 0 → Linear third order ordinary differential equation
Nonlinear ordinary differential equation: A nonlinear ordinary differential equation is an ordinary
differential equation that is not linear.
d2 y dy
Example: 1. 2
+ 5 + 6 y2 = 0
dx dx
3 2
d y y d y
2. 3
+e 2
+ xy = xe x
dx dx
2
d y  dy 
2
3. +   + 6y = 0
dx 2  dx 

Mathematical Problems
Problem:
d 3 y 5 dy 2 dy
What is the order and degree of the differential equation 3
= ( ) +5 + y ?
dx dx dx
Solution:
Given differential equation is,
d 3 y 5 dy 2 dy
3
= ( ) +5 + y
dx dx dx
Squaring both-sides we get,
5
 d3y  dy 2 dy
 3  = ( ) +5 + y
 dx  dx dx
Here highest derivative is 3, so the order derivative is 3.
The power of the highest order derivative is 5, so the degree of this equation is 5. (As desired)

Mohammad Abdul Halim, Assistant Professor in Mathematics


Page 8 of 249

Home works:
◆ State the order and degree of the following differential equations:
5
d2y  dy 
1. + 4   + 10 y = 0
 dx 
2
dx
2
  dy 2  3 d 2 y
2. 1 +    = 2
  dx   dx
2
 dy 
3. 1+   = 1+ x
 dx 
◆ Define linear and non-linear differential equation. Also define order and degree of the differential
equation. Find the order and degree of the differential equation of the followings
4
 d3y 
2
d2y  dy 
4
 3 − 5 x + y = 5   + y2 − x
 dx 
2
 dx  dx
◆ Identify which are linear and non-linear of the given following equations:
𝑑𝑦
1. + 𝑥𝑦 = 𝑥 2
𝑑𝑥
𝑑2 𝑦 𝑑𝑦
2. 𝑑𝑥 2
+ 2 𝑑𝑥 + 3𝑦 2 = 0
𝑑3 𝑦 𝑑2 𝑦
3. 𝑥 3 + 2𝑥 2 2 − 𝑦 = 𝑒 𝑥
𝑑𝑥 𝑑𝑥
𝑑𝑦
4. + 𝑦5 = 0
𝑑𝑥
𝑑2 𝑦 𝑑𝑦
5. 𝑑𝑥 2
+ 3 𝑑𝑥 − 2𝑦 = 0
𝑑2 𝑦 𝑑𝑦 2
6. 2 + 3( ) + 𝑦 = 𝑥
𝑑𝑥 𝑑𝑥
𝑑3 𝑦 𝑑𝑦
7. 𝑑𝑥 3
+ 𝑦 𝑑𝑥 + 𝑦 = 𝑒 𝑥
𝑑3 𝑦 𝑑2 𝑦 𝑑𝑦
8. 𝑥 3 3 + 𝑥 2 2 − 3𝑥 + 𝑦=0
𝑑𝑥 𝑑𝑥 𝑑𝑥
3
2𝑑 𝑥 2
9. 𝑥 𝑑𝑡 3 + 𝑦 = 𝑥𝑦
10. 𝑥(𝑦 − 𝑡)𝑑𝑡 = 𝑑𝑥

Mohammad Abdul Halim, Assistant Professor in Mathematics


Page 9 of 249

Formation of differential Equations


Formation of differential equation means to form the differential equation in x, y of n-th order from its
complete primitive or general solution by eliminating n arbitrary constants.
Problem 01: Eliminate the constants from y = ax + bx 2 .
Solution:
We have
y = ax + bx 2 (i)
Differentiating both sides of (i) with respect to x we get the following
dy dy
= a + 2bx  a = − 2bx
dx dx
Again, differentiating above equation w.r to x we get the following
d2y 1 d2y dy d2y
= 2b b= . 2  a= −x 2
dx 2 2 dx dx dx
Putting the value of ' a ' and ' b ' in the above equation (i ) we get the desired differential equation.
 dy d2y   1 d2y 
y =  − x 2  x +  . 2  x2
 dx dx   2 dx 
dy d 2 y x2 d 2 y
y = x − x2 2 + . 2
dx dx 2 dx
2 2
dy x d y
y=x −
dx 2 dx 2
dy d2y
2 y = 2x − x2 2
dx dx
2
d y dy
x 2 2 − 2 x + 2 y = 0 (As desired)
dx dx
Problem 02: From the differential equation of which c ( y + c ) = x 3 is the complete
2

Integral. Or Derive the differential equation of which c ( y + c ) = x 3 is a solution.


2

Solution:
Given that,
c ( y + c ) = x3
2
(i )
Differentiating both sides of (i) with respect to x we get the following
dy
2c ( y + c )  = 3x 2 (ii )
dx
Dividing the equation (i) by the equation (ii) we get
y+ c x
=
dy 3
2
dx
2 x dy
y+ c = 
3 dx
2 x dy
c=  −y
3 dx

Mohammad Abdul Halim, Assistant Professor in Mathematics


Page 10 of 249

Putting the value of ‘c’ in the equation (ii), we get


 2x dy    2 x dy   dy
2  − yy +   − y    = 3x 2
 3 dx    3 dx   dx
 2x dy  2 x dy  dy
2  − yy +  − y = 3x 2
 3 dx  3 dx  dx
 2x dy   2 x dy  dy
2  − y    = 3x 2
 3 dx   3 dx  dx
2
 4 x dy  2 x  dy 
  − 2 y      = 3x 2
 3 dx  3  dx 
3 2
8 x 2  dy  4 xy  dy 
  −    = 3x 2
9  dx  3  dx 
3 2
8x2  dy   dy 
 8 x   − 12 y   = 27 x (As desired)
9  dx   dx 
Problem 03: Find the differential equation from the relation y = A cos x + B sin x where A and
B are arbitrary constants.
Solution:
We have
y = A cos x + B sin x (i )
Differentiating both sides of (i) with respect to x we get the following
dy
= A ( − sin x ) + B cos x
dx
dy
= − A sin x + B cos x
dx
dy
= B cos x − A sin x
dx
Again, differentiating above equation w.r to x we get the following
d2y
= B ( − sin x ) − A cos x
dx 2
d2y
= − B sin x − A cos x
dx 2
d2y
= − ( A cos x + B sin x )
dx 2
d2y
= −y [ By equation (i ) ]
dx 2
d2y
+ y = 0 (As desired)
dx 2
Problem 04: Find the differential equation of the family of curves y = Ae2 x + Be−2 x , for
different values of A and B.
Solution:
Given that, y = Ae2 x + Be−2 x (i )

Mohammad Abdul Halim, Assistant Professor in Mathematics


Page 11 of 249

Differentiating both sides of (i) with respect to x we get the following


dy
= 2 Ae 2 x + B (−2e −2 x )
dx
dy
= 2 Ae 2 x − 2 Be −2 x
dx
Again, differentiating above equation w.r to x we get the following
d2y
2
= 2 A ( 2e2 x ) − 2 B ( −2e−2 x )
dx
d2y
2
= 4 Ae2 x + 4 Be −2 x
dx
d2y
= 4 ( Ae 2 x + Be −2 x )
dx 2
d2y
= 4y [ By equation (i ) ]
dx 2
d2y
− 4 y = 0 (As desired)
dx 2
Problem 05: Find the differential equation of the family of curves y = e− x ( A cos x + B sin x ) ,
where A and B are arbitrary constants.
Solution:
We have
y = e− x ( A cos x + B sin x )
y e x = A cos x + B sin x (i)
Differentiating both sides of (i) with respect to x we get the following
dy
y ex + ex  = A ( − sin x ) + B cos x
dx
dy
y ex + ex  = − A sin x + B cos x
dx
dy
y ex + ex  = B cos x − A sin x
dx
Again, differentiating above equation w.r to x we get the following
dy x dy x d 2 y
y ex + ex  + e  + e  2 = B ( − sin x ) − A cos x
dx dx dx
2
dy d y
y e x + 2e x  + e x  2 = − B sin x − A cos x
dx dx
dy x d 2 y
y e + 2e  + e  2 = − ( A cos x + B sin x )
x x

dx dx
dy d2y
y e x + 2e x  + e x  2 = − y e x [ By equation (i ) ]
dx dx
dy d 2 y
y + 2 + 2 = −y
dx dx

Mohammad Abdul Halim, Assistant Professor in Mathematics


Page 12 of 249

d2y dy
2
+2 + y+ y = 0
dx dx
2
d y dy
2
+ 2 + 2y = 0
dx dx
This is the required differential equation.
Problem 06: Show that A x 2 + B y 2 = 1 is the solution of the differential equation

 d 2 y  dy  
2
 dy
xy 2 +   = y .

 dx  dx  
 dx
Solution:
We have
A x2 + B y 2 = 1 (i )
Differentiating both sides of (i) with respect to x we get the following
dy
2 A x + 2B y =0
dx
dy
Ax+ B y =0 (ii )
dx
Again, differentiating above equation w.r to x we get the following
 d 2 y dy dy 
A+ B y 2 + .  = 0
 dx dx dx 
Multiplying above equation by x we get
 d 2 y dy dy 
Ax + B x y 2 + .  = 0
 dx dx dx 
 d 2 y  dy 2 
 
A x + B x y 2 +    = 0 (iii)
 dx  dx  
 
Now subtracting equation (ii) from equation (iii) we find,

 d 2 y  dy  
2
  dy 
A x + B x y 2 +   − A x + B y  = 0
 dx  dx  
   dx 

 d 2 y  dy  
2
 dy
B x y 2 +   − B y =0

 dx  dx  
 dx
 d 2 y  dy 2 
  dy
B x y 2 +    = B y
 dx  dx  
  dx
 d 2 y  dy  

2
 dy
xy 2 +   = y (Showed)

 dx  dx  
 dx
Problem 07: Find the differential equation of which y = A + B ln x + c ( ln x ) + 3x 2 is an explicit
2

solution.
Solution:
We have y = A + B ln x + c ( ln x ) + 3x 2
2
(i )

Mohammad Abdul Halim, Assistant Professor in Mathematics


Page 13 of 249

Differentiating both sides of (i) with respect to x until the arbitrary constant removed.
dy 1 1
= 0 + B. + 2c ln x. + 6 x
dx x x
dy
x = B + 2c ln x + 6 x 2
dx
Again differentiating,
d2y dy 1
x 2
+ 1. = 0 + 2c. + 12 x
dx dx x
2
d y dy
x2 2 + x = 2c + 12 x 2
dx dx
Again differentiating,
d3y d2y d 2 y dy
x2 + 2 x + x + = 0 + 24 x
dx3 dx 2 dx 2 dx
d3y d 2 y dy
x 2 3 + 3x 2 + = 24 x
dx dx dx
Which is the required differential equation. (As desired)
−x
Problem 08: Find the differential equation of the solution xy = Ae + Be + x .
x 2

Solution:
We have
xy = Ae x + Be− x + x 2 (i)
Differentiating both sides of (i) with respect to x until the arbitrary constant removed.
dy
x + 1. y = Ae x − Be − x + 2 x
dx
Again differentiating,
d2y dy dy
x 2
+ 1. + = Ae x + Be− x + 2
dx dx dx
d 2 y dy dy
x 2 + + = Ae x + Be− x + 2
dx dx dx
2
d y dy
x 2 + 2 = Ae x + Be − x + 2
dx dx
2
x 2 + 2 = ( xy − x 2 ) + 2
d y dy
[Using (i)]
dx dx
d2y dy
x 2 + 2 − xy + x 2 = 2
dx dx
(As desired)
( ) ( )
Problem 09: Show that the solution of 4 xy + 3 y − x dx + x( x + 2 y )dy = 0 is x 3 4 xy + 4 y 2 − x = c
2

,where c is a constant.
Solution:
We have
x3 ( 4 xy + 4 y 2 − x ) = c (i )
4 x 4 y + 4 x3 y 2 − x 4 = c

Mohammad Abdul Halim, Assistant Professor in Mathematics


Page 14 of 249

Differentiating both sides of (i) with respect to x, we get


dy dy
16 x 3 y + 4 x 4 . + 12 x 2 y 2 + 8 x 3 y. − 4 x 3 = 0
dx dx
dy dy
4 x 3 y + x 4 . + 3 x 2 y 2 + 2 x 3 y. − x 3 = 0
dx dx
4 x ydx + x .dy + 3x y dx + 2 x3 y.dy − x3dx = 0
3 4 2 2

4 x3 ydx + 3x 2 y 2 dx − x3dx + x 4 .dy + 2 x3 y.dy = 0


( 4 x y + 3x y − x ) dx + ( x + 2 x y ) dy = 0
3 2 2 3 4 3

x ( 4 xy + 3 y − x ) dx + x ( x + 2 y ) dy = 0
2 2 3

( 4 xy + 3 y − x ) dx + x ( x + 2 y ) dy = 0
2

Hence x 3 ( 4 xy + 4 y − x ) = c is the solution of ( 4 xy + 3 y − x ) dx + x ( x + 2 y ) dy = 0 .


2 2

(As desired)
Problem 10: From the differential equation of which y = ae + be− x + c cos x + d sin x is a solution.
x

Solution:
We have
y = ae x + be− x + c cos x + d sin x (i )
Differentiating both sides of (i) with respect to x until the arbitrary constant removed.
dy
= ae x − be − x − c sin x + d cos x
dx
Again differentiating,
d2y
2
= ae x + be− x − c cos x − d sin x
dx
Again differentiating,
d3y
= ae x − be− x + c sin x − d cos x
dx3
d4y
4
= ae x + be− x + c cos x + d sin x
dx
d4y
=y [Using (i)]
dx 4
d4y
−y=0
dx 4
Which is the required differential equation.
(As desired)
Problem 11: Find the differential equation in its simplest form from the relation
1 − x2 + 1 − y 2 = a ( x − y ) .
Solution:
We have
1 − x2 + 1 − y 2 = a ( x − y ) (i)
Differentiating both sides of (i) with respect to x, we get

Mohammad Abdul Halim, Assistant Professor in Mathematics


Page 15 of 249

1 1 dy  dy 
 ( −2 x ) +  ( −2 y ) . = a 1 − 
2 1− x 2
2 1− y 2 dx  dx 
−x −y dy  dy 
+ . = a 1 − 
1 − x2 1− y2 dx  dx 
 x y dy   dy 
− + .  = a 1 −  (ii)
 1 − x2 1 − y 2 dx   dx 

Dividing the equation (i) by (ii) to eliminate a, we get
1 − x2 + 1 − y 2 a ( x − y)
=
 x dy   dy 
− +
y
.  a 1 − 
 1 − x2 1 − y 2 dx   dx 

1 − x2 + 1 − y 2
=
( x − y)
 x dy   dy 
− +
y
.  1 − 
 1 − x2 1 − y 2 dx   dx 

1 − x2 + 1 − y 2
=
( x − y)
 x dy   dy 
 +
y
.   − 1
 1 − x2 1 − y 2 dx   dx 

 x dy 
 dy 
 − 1
 dx 
( )
1 − x2 + 1 − y 2 = ( x − y ) 
 1 − x2

+
y
. 
1 − y 2 dx 

 x2 dy 
dy
dx
( )
1 − x2 + 1 − y 2 − 1 − x2 − 1 − y 2 = 
 1 − x2

+
xy dy
. −
1 − y 2 dx
xy
1 − x2

y2
. 
1 − y 2 dx 
 x 2 − xy xy − y 2 dy 
dy
dx
( )
1 − x2 + 1 − y 2 − 1 − x2 − 1 − y 2 = 
 1 − x2

+
1 − y
. 
2 dx 

dy
dx
(
1 − x2 + 1 − y 2 − )
xy − y 2 dy
. = 1 − x2 + 1 − y 2 + 
1 − y 2 dx
 x 2 − xy 
 1− x 
2

dy  xy − y 2  x 2 − xy
 1− x + 1− y −
2 2
 = 1− x + 1− y +
2 2

dx  − 2 
1 − x2
 1 y 
dy  1 − x 1 − y + (1 − y ) − xy + y  (1 − x ) + 1 − x 1 − y + x − xy
 2 2 2 2  2 2 2 2

=
dx  1 − y 2  1 − x2
 
dy  1 − x 2 1 − y 2 + 1 − xy  1 + 1 − x 2 1 − y 2 − xy
 =
dx  − 2  1 − x2
 1 y 

Mohammad Abdul Halim, Assistant Professor in Mathematics


Page 16 of 249

dy  1  1
 =
dx  1 − y 2  1 − x2
 
dy 1− y2
= , This is the required differential equations.
dx 1 − x2
(As desired)

Short Questions
Find differential equations of:
1. All parabolas having their axes parallel to y-axis.
2. All the circles having a constant radius “a”.
3. The system of circles having a constant radius “a” and having their Centre’s on the x-axis.
4. All circles passing through the origin and having their Centre’s on the x-axis.
5. All straight lines passing through the origin.
6. All parabolas each of which has a latus rectum 4a and whose axes are parallel to x-axis.
7. All tangent lines to a parabola.
8. All conics whose axes coincide with axes of coordinate.
Board Questions
Type 01:
From differential equations for the given following functions:
1. r = a (1 + cos  )
2. r 2 = a 2 cos 2
3. ( x − h) 2 = 4a ( y − k )
A
4. v = + B
r
b
5. y = ax +
x
6. y = e ( A sin x + B cos x )
x

7. y = e x ( A sin 2 x + B cos 2 x )
8. y = Ae3 x + Be−2 x + sin 5x
x2 y 2
9. + =1
a 2 b2
10. xy = ae x + be− x
11. y = a cos( x +  )
Type 02:
1. Show that the differential equation of a concentric circles having the centres at the origin is
xdx + ydy = 0 . Interpret the result geometrically.
2. Prove that the differential equation of all circles touching the x-axis at the origin is
( )
x 2 − y 2 dy − 2 xydx = 0 .
3. Show that the differential equation of the family of circles x 2 + y 2 + 2 gx + 2 fy + c = 0 is
(1 + y ) y
2
1 3 − 3 y1 y22 = 0 .
( )
4. Verify that y + x + 1 = 0 is a solution of the differential equation ( y − x ) dy − y 2 − x 2 dx = 0 .

Mohammad Abdul Halim, Assistant Professor in Mathematics


Page 17 of 249

A d 2V 2 dV
5. Show that V = + B is a solution of the differential equation + . = 0.
r dr 2 r dr
6. Find the differential equation of the straight lines y = ax .
7. Find the differential equation eliminating a, b, c from the relation ax + by + c = 0 .
8. Find the differential equation of parabola y = ax 2 + bx + c where a, b, c are constants.
( )
9. Eliminate c from the relation x 3 y + c 2 + c = 0 .
−1
10. Eliminate a and b from y = a tan x + b .
11. Obtain the differential equation of the family of curves y = e x ( A cos x + B sin x ) .
d2y
12. Show that the general solution of the differential equation + y = 0 is y = A cos x + B sin x .
dx 2
13. Find the differential equation associated with the primitive x = a cos t + b sin t .
Obtain the differential equation corresponding to y = ae + be .
3x x
14.
dy 2
15. Show that x3 + 3xy 2 = 1 is an implicit solution of the differential equation 2 xy + x + y2 = 0
dx
on the interval 0  x  1 .
Show that every function f ( x ) defined by f ( x) = ( x + c ) e where c is the arbitrary constant,
3 −3 x
16.
dy
is a solution of the differential equation + 3 y = 3x 2 e −3 x .
dx
dy
17. Show that the first order differential equation + y + 1 = 0 has no real solutions.
dx
MCQ Questions

1. y = cx − c 2 is the general solution of the differential equation.


a. ( y  ) − xy + y = 0
2
b. y  = 0 c. y = c d.
( y ) + xy + y = 0
2

2.The differential equation governing the solutions y ( x) = 5 x + c where c is an arbitrary


constant.
a. y 2 = 5 x + c b. yy = 5 2.5 d. yy = 2.5
c. y =
5x + c
2
 dx  1
3.The differential equation   + 5 y 3
= x is
 
dy
a. linear of degree 2 b.non-linear of order 1 c.non-linear of order 1 d.none of these
and degree 2 and degree 6
4. The equation y2 = cx is general solution of :
2y 2x x y
a. y = b. y = c. y = d. y =
x y 2y 2x
5.The differential equation whose solution is Ax + By = 1 where A and B are constants is of
2 2

a.first order and b. first order and first c. second order and first d. second order and
second degree degree degree second degree

Mohammad Abdul Halim, Assistant Professor in Mathematics


Page 18 of 249

4
d3y  dy  d4y
6.The degree of the differential equation + x   = 4 log is
dx 3  dx  dx 4
a.1 b.3 c.4 d.None of these
7. The order of the differential equation satisfying 1 − x 4 + 1 − y 4 = a ( x 2 − y 2 ) is
a.1 b.2 c.3 d.None of these
3
 d4y  5 d3y d2y dy
8. The degree of the differential equation  4 
− 5 3
+ 6 2
− 8 + 5 = 0 is
 dx  dx dx dx
a.2 b.3 c.4 d.5
2
 dy   dy 
9.A solution of differential equation   − x   + y = 0 is
 dx   dx 
a. y = 2 b. y = 2 x c. y = 2 x − 4 d. y = 2 x − 4
2

(−1
10.Differential equation of y = sec tan x )
b. (1 + x 2 ) c. (1 + x 2 ) d. (1 + x 2 )
a. dy dy dy x
= y−x+c = xy + c = +c
(1 + x ) dy
2

dx
= y+ x+c dx dx dx y
2

11.The degree of the differential equation 1 + 3  = 4 3 are


dy 3 d3y
 dx  dx
a.(1,2) b.(3,1) c.(3,3) d. None of these

Mohammad Abdul Halim, Assistant Professor in Mathematics


Page 19 of 249

Differential equation of First order and first degree

Differential equation of First order and first degree


Definition:
dy
A differential equation of the type M + N = 0 , where M and N are functions of x and y or constants
dx
is called a differential equation of the first order and first degree.
There are two standard forms of differential equations of first order and first degree as
dy
❖ = f ( x, y )
dx
❖ M ( x, y ) dx + N ( x, y ) dy = 0
We classify the first order and first-degree differential equation followings eight categories according to its
solution Method:
❖ Equation of variable separable form
❖ Equation reducible to variable separable form
❖ Homogeneous Differential Equation
❖ Equation reducible to Homogeneous form
❖ Linear differential equation
❖ Equation reducible to linear differential equation (Bernoulli’s Equation)
❖ Exact differential equation
❖ Equation reducible to exact differential equation
◊◊ Equation of variable Separable form:
The process of reducing the first order and first-degree differential equation
M ( x, y ) dx + N ( x, y ) dy = 0 to the form f1 ( x) dx = f 2 ( y) dy or f1 ( x) dx + f 2 ( y) dy = 0 , where
f1 ( x) is a function of x only and f2 ( y) is a function of y only, is called the separation of variables. In
others way an equation of the form F ( x ) G( y)dx + f ( x ) g ( y)dy = 0 is called an equation with variables
separable or simply a separable equation.
Such a variable separable equation f1 ( x) dx = f 2 ( y ) dy or f1 ( x) dx + f 2 ( y ) dy = 0 are solved by
integrating both-sides directly and adding an arbitrary constant of integration to any one of the two sides.
Thus the solution of f1 ( x) dx = f 2 ( y) dy is  f ( x)dx =  f
1 2 ( y )dy + C where C is an arbitrary constant.
Note:
There is no need to use two constants in the integration of a separable equation because difference of
constants can be replaced by a single constant that is solution of first order and first-degree equation contain
only one arbitrary constant. It is to keep in mind that to simplify the solution of above-mentioned equation,
C
the constant of integration C can be selected in any suitable form as , log C , tan C , tan −1 C , eC etc
3

Mohammad Abdul Halim, Assistant Professor in Mathematics


Page 20 of 249

.Sometimes transformation to the polar coordinates facilitates separation of variables. So it is convenient to


remember the following differentials. If x = r cos and y = r sin  then
• xdx + ydy = rdr

• xdx − ydy = r 2 d
• dx 2 + dy 2 = dr 2 + r 2 d 2

Mathematical Problems based on variable separable form

(
2 2
) ( )
Problem 01: Solve the differential equation 1 + y dx + 1 + x dy = 0 by variable separable Method.
Solution:
Given differential equation
(1 + y ) dx + (1 + x ) dy = 0
2 2

Dividing the above differential equation by (1 + x )(1 + y ) , we get


2 2

dx dy
+ =0
1+ x 1+ y2
2

Above differential equation is of the variable separable form.


So, integrating both-sides we find the desired solution
dx dy
 1+ x  1+ y2 = C
2
+

tan −1 x + tan −1 y = C
x+ y
tan −1 =C
1 − xy
x+ y
tan −1 = tan −1 a [ letting arbitrary const , C = tan −1 a ]
1 − xy
x+ y
=a
1 − xy
x + y = a (1 − xy )
It is considered as the complete solution of the given differential equation.
(As desired)
(
2 2
) ( )
Problem 02: Solve the differential equation 4 + y dx + 4 + x dy = 0 by variable separable Method.
Solution:
Given differential equation
( 4 + y ) dx + ( 4 + x ) dy = 0
2 2

Dividing the above differential equation by ( 4 + x )( 4 + y ) , we get


2 2

dx dy
+ =0
4 + x 4 + y2
2

Above differential equation is of the variable separable form.

Mohammad Abdul Halim, Assistant Professor in Mathematics


Page 21 of 249

So, integrating both-sides we find the desired solution


dx dy
 4+ x + 4+ y2 2
=C

dx dy
2 2
+x 2
+ 2 2 =C
2 +y
x y
tan −1 + tan −1 = C
2 2
x y
+
tan 2
−1 2 =C
x y
1− .
2 2
x+ y
tan −1 2 = C
xy
1−
4
x+ y
tan −1 2 = C
4 − xy
4
x+ y 4
tan −1 (  )=C
2 4 − xy
2( x + y )
tan −1 =C
4 − xy
2( x + y )
= tan C = a [ letting arbitrary const , tan C = a ]
4 − xy
2( x + y )
=a
4 − xy
2( x + y ) = a (4 − xy )
It is considered as the complete solution of the given differential equation.
(As desired)
dy
Problem 03: Solve the differential equation = 1 − x 2 1 − y 2 by variable separable Method.
dx
Solution:
Given differential equation
dy
= 1 − x2 1 − y2
dx
Rearranging the term given equation will be a variable separable form
dy
= 1 − x 2 dx
1− y 2

So, integrating both-sides we find the desired solution

Mohammad Abdul Halim, Assistant Professor in Mathematics


Page 22 of 249

dy
 1− y 2
=  1 − x 2 dx + C

−1 x 1 − x 2 1 −1 x
sin y = + sin +C
2 2 1
x 1 − x 2 1 −1
sin −1 y = + sin x + C
2 2
 x 1 − x 2 1 −1 
y = sin  + sin x + C 
 2 2 
 
(As desired)
( )
Problem 04: Solve the differential equation e + 1 cos x dx + e (1 + sin x) dy = 0 by variable separable
yy

Method.
Solution:
Given differential equation
(e y
+ 1) cos x dx + e y (1 + sin x) dy = 0

( )
Dividing the above differential equation by 1 + e (1 + sin x ) , we get
y

cos x dx e y dy
+ =0
1 + sin x 1 + e y
Above differential equation is of the variable separable form.
So, integrating both-sides we find the desired solution
cos x dx e y dy
 1 + sin x  1 + e y = C
+

 f ( x) 
ln 1 + sin x + ln 1 + e y = C 

 f ( x)
dx = ln f ( x) 

ln(1 + sin x)(1 + e y ) = C


(1 + sin x)(1 + e y ) = eC = a  say eC = a 
(1 + sin x)(1 + e y ) = a
Above equation is the desired solution of the given differential equation.
(As desired)
x− y y−x
Problem 05: Solve e dx + e dy = 0 .
Solution:
Given differential equation
e x − y dx + e y − x dy = 0
e x .e− y dx + e y .e− x dy = 0
ex ey
.dx + x dy = 0
ey e
x y
Multiplying both-sides of above equation by e .e , we get
e x .e x dx + e y .e y dy = 0

Mohammad Abdul Halim, Assistant Professor in Mathematics


Page 23 of 249

e2 x dx + e2 y dy = 0
Taking on both-sides of above equations we get
e dx +  e 2 y dy = c
2x

1 2x 1 2 y
.e + .e = c
2 2
. ( e + e 2 y ) = c
1 2x
2
e2 x + e2 y = 2c
e2 x + e2 y = c [Say 2c = c]
Which is the solution of the given equation.
(As desired)

dy
Problem 06: Solve the differential equation = e x − y + x 2e− y .
dx
Solution: Given differential equation is,
dy
= e x − y + x 2e− y
dx
Separating the variables, we get
 e y dy = ( e x + x 2 ) dx
Integrating both-sides, we find
 e dy =  ( e + x 2 ) dx + C
y x

x3
 e y = ex + +C
3
 x x3 
 y = ln  e + + C 
 3 
Which is the complete or general solution of the given differential equation.
(As desired)
( )(
Problem 07: Solve x + 1 y − 1 dx + x y dy = 0
2 2
)
Solution:
( )( )
Given equation, x + 1 y − 1 dx + x y dy = 0
2 2

Dividing both sides by (y − 1) x , we get


2

(x + 1) dx + y dy = 0
2

x y 2 −1


(x 2
+1 ) y
x
dx + y 2
−1
dy = c 1

 1 1 2y
   x +  dx +  2 dy = c 1
 x 2 y −1

+ log x + log (y 2 − 1) = c 1
x2 1

2 2

Mohammad Abdul Halim, Assistant Professor in Mathematics


Page 24 of 249

( )
 log y 2 − 1 = 2c 1 − x 2 − log x 2
2 c − x 2 − log x 2
 y 2 −1 = e 1
1
 y 2 =1 + 2 c e − x
2

x
(As desired)
dy
= e x+ y + x 2 e x +y
3
Problem 08: Solve .
dx
Solution:
dy
= e x+ y + x 2 e x + y
3
Given that
dx
 e − y dy = e x dx + x 2 e x dx
3

 −e −y = e x + xe
2 x3
dx
Let x = t  3x dx = dt
3 2

1 t 1 1 3
 x 2 e x dx =  e dt = e t = e x
3
Then,
3 3 3
1 x3
Therefore, the general solution is − e −y = e x + e +c (As desired)
3
Problem 09: Solve ( x − 1)( y + 1) = (2 y + 1) x .
dy
dx
Solution:
y +1 x
dy = dx
2 y +1 x −1
 2y + 2 
1 x −1 +1
   d y = dx
 2 y +1 
2 x −1
1  2 y +1 1   x −1 1 
  +  d y =  +  dx
2  2 y +1 2 y +1   x −1 x −1 
 1 1   1 
 1+  d y = 1 +  dx
 22 y +1   x −1 
1 
  y + ln (2 y + 1) = x + ln ( x − 1) + c
1
2 2 
Which is the required solution. (As desired)

dy
Problem 10: Solve = 2 xy .
dx
dy
Solution: Given that, = 2 xy
dx
Separating variables we obtain,

Mohammad Abdul Halim, Assistant Professor in Mathematics


Page 25 of 249

dy
= 2 xdx
y
Now, integrating,
dy
 y
= 2 xdx

x2
ln y = 2 + ln c
2
ln y = x 2 + ln c
ln y = ln e x + ln c
2

( )
ln y = ln ce x
2

 y = ce x
2

(As desired)

dy xy
Problem 11: Solve =− .
dx x +1
Solution: Given that,
dy xy
=−
dx x +1
Separating variables we obtain,
dy xdx
=−
y x +1
dy  1 
= − 1 −  dx
y  x +1
Now, integrating,
dy  1 
 y
= −  1 −
 x +1
 dx

ln y = − x + ln ( x + 1) + ln c
ln y = ln e− x + ln ( x + 1) + ln c
ln y = ln ( ce − x ( x + 1) )
 y = ce− x ( x + 1) (As desired)

dy x + 1
Problem 12: Solve = 2 .
dx y
Solution: Given that,
dy x + 1
= 2
dx y
Separating variables we obtain,
y 2 dy = ( x + 1) dx
Now, integrating,

Mohammad Abdul Halim, Assistant Professor in Mathematics


Page 26 of 249

 y dy =  ( x + 1) dx
2

y3 x2
= + x+c
3 2
(As desired)

dy
Problem 13: Solve = x2 y
dx
Solution: Given that,
dy
= x2 y
dx
Separating variables we obtain,
dy
= x 2 dx
y
Now, integrating,
dy
 y 
= x 2 dx

x3
ln y = + ln c
3
x3
ln y = ln e 3
+ ln c
 x 
3

ln y = ln  ce 3 
 
 
x3
 y = ce 3

(As desired)
dy
Problem 14: Solve = e x+ y
dx
Solution: Given that,
dy
= e x+ y
dx
dy
= e x .e y
dx
Separating variables we obtain,
dy
y
= e x dx
e
−y
e dy = e x dx
 
−y
Now, integrating, e dy = e dx
x

Mohammad Abdul Halim, Assistant Professor in Mathematics


Page 27 of 249

−e − y = e x + c
(As desired)
dy
Problem 15: Solve = e x− y
dx
Solution: Given that,
dy
= e x− y
dx
dy
= e x .e − y
dx
Separating variables we obtain,
dy
−y
= e x dx
e
e y dy = e x dx
Now, integrating,

e dy = e dx
y x

e y = ex + c
(As desired)

dy
Problem 16: Solve = e2 x +3 y
dx
Solution: Given that,
dy
= e2 x +3 y
dx
dy
= e 2 x .e3 y
dx
Separating variables we obtain,

dy
3y
= e 2 x dx
e
e−3 y dy = e2 x dx
Now, integrating,

e dy = e 2 x dx
−3 y

e −3 y e2 x
− = + c (As desired)
3 2
= (1 − 2 x 2 ) tan y
dy
Problem 17: Solve x
dx

Mohammad Abdul Halim, Assistant Professor in Mathematics


Page 28 of 249

= (1 − 2 x 2 ) tan y
dy
Solution: Given that, x
dx
Separating variables we obtain,
dy 1 − 2x2
= dx
tan y x
1 
cot y dy =  − 2 x  dx
x 
Now, integrating,
1 
 cot y dy =   x − 2 x  dx
x2
ln siny = ln x − 2. + ln c
2
ln siny = ln x − x 2 + ln c
ln siny = ln x + ln e− x + ln c
2

(
ln sin y = ln cx.e− x
2

)
sin y = cx.e− x
2
(As desired)

dy xy + 3x − y − 3
Problem 18: Solve =
dx xy − 2 x + 4 y − 8
Solution: Given that,
dy xy + 3x − y − 3
=
dx xy − 2 x + 4 y − 8
dy x ( y + 3) − 1( y + 3)
 =
dx x ( y − 2 ) + 4 ( y − 2 )
dy ( y + 3)( x − 1)
 =
dx ( y − 2 )( x + 4 )
dy y + 3 x − 1
 = 
dx y − 2 x + 4
Separating variables form, we obtain
y−2 x −1
dy = dx
y+3 x+4
y +3−5 x+4−4
 dy = dx
y+3 x+4
 5   4 
 1 −  dy = 1 −  dx
 y +3  x+4
Integrating both-sides of above equation, we find desired solution

Mohammad Abdul Halim, Assistant Professor in Mathematics


Page 29 of 249

 5   4 
 1 − y + 3  dy =  1 − x + 4  dx
 5   4 
 1 − y + 3  dy =  1 − x + 4  dx

y − 5ln y + 3 = x − 4ln x + 4 + c
(As desired)

Short Questions
1. What do you mean by variable separable form?
2. Find a function whose square plus the square of its derivative is 1.
Broad Questions
Type 01:

Solve the following differential equations:


dy
1. = 1 + x2 1 − y2
dx
2. x 1 − y 2 dx + y 1 − x 2 dy = 0
3. (1 + cos ecx)dy = dx
dy
4. = 1 − y 2 cos 5 x e3 x
dx
5. sec2 x tan ydx + sec2 y tan xdy = 0
6. (e y
+ 1) cos x dx + e y sin xdy = 0
7. 3e x tan ydx + (1 − e x )sec2 ydy = 0
8. (1 − x 2 )(1 − y)dx = xy(1 + y)dy
dy  dy 
9. y − x = a  y2 + 
dx  dx 
10. dx = (1 − cos x)dy
( )
11. x sin ydx + 1 + x 2 cos ydy = 0
dy
12. = e x − y + x 2e− y
dx
13. x2 ( y − 1) dx + y 2 ( x −1)dy = 0

( )
14. 1 + y 2 dx + 1 − x 2 dy = 0

15. y x − 1 dx + x y − 1 dy = 0
2 2

16. cos y dy = sec 2 xdx


17. 5cos 2 y dx + csc 2 x dy = 0
dy 7 xe2 y
18. =
dx 5 + 3x 2

Mohammad Abdul Halim, Assistant Professor in Mathematics


Page 30 of 249

= (1 + y 2 ) sin 2 x e5 x
dy
19.
dx
20. sec2 x tan ydx + sec2 y tan xdy = 0
21. y sec2 xdx + ( y + 7 ) tan xdy = 0
dy x(2 log x + 1)
22. =
dx sin y + y cos y
 dy 
23. ln   = ax + by
 dx 
dy xy + 2 y − x − 2
24. =
dx xy − 3 y + x − 3
dy y 2 + y + 1
25. + =0
dx x 2 + x + 1
dy x(1 + y 2 )
26. =
dx y (1 + x 2 )
dy ( x 2 − 1)( y 2 − 1)
27. + =0
dx xy
28. ( x 2 − yx 2 ) dy + ( y 2 + xy 2 ) dx = 0
29. x cos2 ydx − y cos2 xdx = 0
30. ( x + y )( dx − dy ) = dx + dy
31. ( x + y ) dy + ( x − y ) dx = 0
dy x ( 2 ln x + 1)
32. =
dx sin y + y cos y

Type 02:
dy  
1. Solve x + cot y = 0 provided that ( x, y ) =  2,  .
dx  4
dy  
2. Solve x + cos y = 0 provided that ( x, y ) =  2,  .
dx  4
y 1 
−1
3. Solve sin xdy + dx = 0 given that ( x, y ) =  , 2  .
1 − x2 2 
df
4. Find the function f which satisfy the equation = 2 f , f (0) = e3 .
dx
 df 
5. Find the curves passing through (0,1) and satisfying sin  =c.
 dx 

(
6. Without the use of technology, how would you solve x + x )dy
dx
= ( )
y+y .

Mohammad Abdul Halim, Assistant Professor in Mathematics


Page 31 of 249

7. Show that the equation of the form yf1 ( xy)dx + xf 2 ( xy)dy = 0 can be separated by the substitution
xy = v .
8. If f ( x) + f ( x) = 0 and f (0) = 2 then find f ( x ) .
9. Find f ( x ) when f ( x) = x f ( x) and f (0) = 1 .

( −x
)
10. Find the particular solution of cos ydx + 1 + 2e sin ydy = 0 when x = 0, y =
4
.

◊◊ Equation reducible to variable separable form:


dy
Prove that equation of the form = f (ax + by + c) is called the equation reducible to variable separable
dx
form and it can be solved by choosing the transformation ax + by + c = v .
Proof:
Given differential equation is
dy
= f (ax + by + c) (i )
dx
dy dv dy 1  dv 
Now choosing a transformation ax + by + c = 0 such that a + b =  =  −a
dx dx dx b  dx 
Now from equation (i), we get
1  dv 
 − a  = f (v )
b  dx 
dv
− a = bf (v)
dx
dv
= a + bf (v)
dx
dv
= dx
a + bf (v)
This is the equation of variable separable form and it can be solved by integrating both-sides and adding an
arbitrary constant.

Mathematical Problems based on reducible to variable separable form

dy
= ( 4 x + y + 1) by choosing appropriate transformation.
2
Problem 01: Solve the differential equation
dx
Solution:
Given differential equation is
dy
= ( 4 x + y + 1)
2
(i )
dx
Now choose a transformation 4 x + y + 1 = v such that
dy dv dy dv
4+ =  = −4
dx dx dx dx
Now from equation (i), we get

Mohammad Abdul Halim, Assistant Professor in Mathematics


Page 32 of 249

dv
− 4 = v2
dx
dv
= 4 + v2
dx

dv
= dx
4 + v2
It is the equation of the variable separable form.
Now integrating both sides of above mentioned equation, we find the solution of
the above equation as follows
dv
 4+v 2
=  dx + C

dv
2 2
+ v2 
= dx + C

1 v
tan −1 = x + C
2 2
v
tan −1 = 2 x + 2C
2
v
= tan ( 2 x + 2C )
2
v = 2 tan ( 2 x + 2C )
4 x + y + 1 = 2 tan ( 2 x + 2C )
This above equation is the solution of the given equation.
(As desired)
dy
Problem 02: Solve the differential equation = tan ( x + y + 6 ) by choosing appropriate transformation.
dx

Solution:
Given differential equation is
dy
= tan ( x + y + 6 ) (i )
dx
Now choose a transformation x + y + 6 = v such that
dy dv dy dv
1+ =  = −1
dx dx dx dx
Now from equation (i), we get

dv
− 1 = tan v
dx

Mohammad Abdul Halim, Assistant Professor in Mathematics


Page 33 of 249

dv
= 1 + tan v
dx

dv
= dx
1 + tan v
It is the equation of the variable separable form.
Now,
dv
= dx
sin v
1+
cos v
dv
= dx
cos v + sin v
cos v
cos v dv
= dx (ii )
cos v + sin v
d
Let cos v = l ( cos v + sin v ) + m ( cos v + sin v )
dv
cos v = l ( − sin v + cos v ) + m ( cos v + sin v )
cos v = l ( cos v − sin v ) + m ( cos v + sin v )
cos v = ( l + m) cos v + ( m − l ) sin v
Comparing the like term on both-sides, we get
l + m =1 and m − l = 0
1 1
Solving this we find m = &l =
2 2
From above equation (ii) we get
1 1 
 ( − sin v + cos v ) + ( cos v + sin v )  dv
 2 2  = dx
cos v + sin v
Now integrating it reduces to

1 1
( − sin v + cos v ) + ( cos v + sin v )
2 2
cos v + sin v
dv =  dx

1 − sin v + cos v 1 cos v + sin v



2 cos v + sin v
dv + 
2 cos v + sin v
dv = x + c

1 − sin v + cos v 1

2 cos v + sin v
dv +  dv = x + c
2
1 1
ln cos v + sin v + v = x + c
2 2
ln cos v + sin v + v = 2 ( x + c )
Replacing the value of v in above equation we get

Mohammad Abdul Halim, Assistant Professor in Mathematics


Page 34 of 249

ln cos ( x + y + 6 ) + sin ( x + y + 6 ) + ( x + y + 6 ) = 2 ( x + c )
(As desired)
dy
Problem-03: Solve the differential equation = sin ( x + y ) + cos ( x + y ) .
dx
Solution: Given differential equation is,
dy
= sin ( x + y ) + cos ( x + y ) … … … (1)
dx
Let, x + y = v

dy dv dy dv
 1+ =  = −1
dx dx dx dx
Now from equation (1), we get
dv
− 1 = sin v + cos v
dx
dv
 = sin v + cos v + 1
dx
dv
 = dx
sin v + cos v + 1
dv
 = dx
v v 2 v
2sin cos + 2cos
2 2 2
dv
 = dx
 v
sin 
v 2
2 cos 2 1 + 
2  cos v 
 2
v
sec 2 dv
 2 = dx
 v
2 1 + tan 
 2

Integrating both sides, we get


v
sec 2
1

2  
2 dv = dx + C
v
 1 + tan 
 2

Mohammad Abdul Halim, Assistant Professor in Mathematics


Page 35 of 249

v
 ln 1 + tan = x+C
2

 ln 1 + tan
( x + y) = x+C
2
which is the complete integral or general solution of the given differential equation.
(As desired)
dy
Problem-04: Solve the differential equation ( x + y ) = a2 .
2

dx
Solution:
Given differential equation is,
dy
( x + y) = a 2 … … … (1)
2

dx
Let, x + y = v

dy dv dy dv
 1+ =  = −1
dx dx dx dx
Now from equation (1), we get

 dv 
v 2  − 1 = a 2
 dx 

dv a2
 −1 = 2
dx v

dv a 2
 = +1
dx v 2

dv a 2 + v 2
 =
dx v2

v2
 dv = dx
a2 + v2
Integrating both sides, we get

v2
 a 2 + v2 dv =  dx + C
a2 + v2 − a2
 dv =  dx + C
a2 + v2

a2 + v2 a2
 dv −  a 2 + v2 dv =  dx + C
a2 + v2

Mohammad Abdul Halim, Assistant Professor in Mathematics


Page 36 of 249

a2
  dv −  dv =  dx + C
a2 + v2
1 v
 v − a 2 . tan −1 = x + C
a a

 x + y − a tan −1
( x + y) = x + C
a

 y = a tan −1
( x + y) + C
a
Which is the complete integral or general solution of the given differential equation.
(As desired)
−1 dy
Problem-05: Solve sin = x+ y.
dx
Solution:
Given differential equation is,
dy
sin −1 = x+ y
dx
dy
= sin ( x + y ) (i )
dx
dy dv dy dv
Let x + y = v  1 + =  = −1
dx dx dx dx
From the equation (i) we get
dv
− 1 = sin v
dx
dv
= 1 + sin v
dx
dv
= dx
1 + sin v
(1 − sin v ) dv = dx
(1 + sin v )(1 − sin v )
(1 − sin v ) dv = dx
1 − sin 2 v
(1 − sin v ) dv = dx
cos 2 v
 1 sin v 
 −  dv = dx
 cos v cos v 
2 2

( sec 2
v − tan v sec v ) dv = dx
Integrating both-sides we get
 ( sec v − tan v sec v ) dv =  dx
2

Mohammad Abdul Halim, Assistant Professor in Mathematics


Page 37 of 249

 sec v dv −  tan v sec v dv = x + c


2

tan v − sec v = x + c
(As desired)
dy
Problem-06: Solve = 1− x + y .
dx
Solution:
Given differential equation is,
dy
= 1− x + y (i )
dx
dy dv dy dv
Let x + y = v  1 + = 2v  = 2v − 1
2

dx dx dx dx
From the equation (i) we get
dv
2v −1 = 1 − v
dx
dv
2v = 2−v
dx
v dx
dv =
2−v 2
2−2+v dx
dv =
2−v 2
2 − (2 − v) dx
dv =
2−v 2
 2  dx
 − 1 dv =
 2−v  2
2 dx
dv − dv =
2−v 2
Taking integration on both-sides, we get
2 dx
 2 − v dv −  dv =  2
−1 dx
−2  dv −  dv = 
2−v 2
x
−2 ln 2 − v − v = + c1
2
x
2 ln 2 − v + v = − − c1
2
x
2 ln 2 − v + v + = −c1
2
4ln 2 − v + 2v + x = −2c1
4ln 2 − v + 2v + x = c [say c = −2c1 ]
Putting the value of v we get, 4ln 2 − x + y + 2 x + y + x = c (As desired)

Mohammad Abdul Halim, Assistant Professor in Mathematics


Page 38 of 249

dy
= 1− ( x + y) .
2
Problem-07: Solve
dx
Solution:
Given differential equation is,
dy
= 1− ( x + y)
2
(i )
dx
dy dv dy dv
Let x + y = v  1 + =  = −1
dx dx dx dx
From the equation (i) we get
dv
−1 = 1 − v2
dx
dv
= 1 + 1 − v2
dx
dv
= dx
1 + 1 − v2
Let v = sin z  dv = cos z dz .
Now,
cos z dz
= dx
1 + 1 − sin 2 z
cos z dz
= dx
1 + cos 2 z
cos z
dz = dx
1 + cos z
1 + cos z − 1
dz = dx
1 + cos z
 1 
1 −  dz = dx
 1 + cos z 
1
dz − dz = dx
1 + cos z
Integrating both-sides, we get
1
 dz −  1 + cos z dz =  dx
1
 dz −  1 + cos z dz = x + c
1
 dz −  1 + cos z dz = x + c
1
 dz −  z
2
dz = x + c
2cos
2
1 2 z
 dz − 2  sec 2 dz = x + c

Mohammad Abdul Halim, Assistant Professor in Mathematics


Page 39 of 249

1 z
z − .2 tan 2 = x + c
2 2
z
z − tan 2 = x + c
2
 sin ( x + y ) 
 
−1

sin −1 ( x + y ) − tan 2  = x+c



 2 

Which is the required solution.
(As desired)
𝑑𝑦
Problem-08: Solve =𝑥+𝑦
𝑑𝑥

Solution: Given that,


𝑑𝑦
𝑑𝑥
= 𝑥 + 𝑦………………(i)

Let 𝑣 = 𝑥 + 𝑦
𝑑𝑣 𝑑𝑦
𝑜𝑟, 𝑑𝑥 = 1 + 𝑑𝑥
𝑑𝑦 𝑑𝑣
𝑜𝑟, 𝑑𝑥 = 𝑑𝑥 − 1

From (i) we get,


𝑑𝑣
𝑑𝑥
−1=𝑣
𝑑𝑣
𝑜𝑟, 𝑑𝑥 = 1 + 𝑣

Separating the variables, we obtain


𝑑𝑣
1+𝑣
= 𝑑𝑥

Now, integrating
𝑑𝑣
∫ 1+𝑣 = ∫ 𝑑𝑥

𝑜𝑟, ln(1 + 𝑣) = 𝑥 + ln 𝑐
𝑜𝑟, ln(1 + 𝑣) = ln 𝑒 𝑥 + ln 𝑐
𝑜𝑟, ln(1 + 𝑣) = ln(𝑐𝑒 𝑥 )
𝑜𝑟, 1 + 𝑣 = 𝑐𝑒 𝑥
𝑜𝑟, 1 + 𝑥 + 𝑦 = 𝑐𝑒 𝑥
∴ 𝑦 = 𝑐𝑒 𝑥 − 1 − 𝑥 (As desired)
𝑑𝑦
Problem-09: Solve 𝑑𝑥
= sin(𝑥 + 𝑦 + 6)

Solution: Given that,


𝑑𝑦
= sin(𝑥 + 𝑦 + 6) …………(i)
𝑑𝑥

Mohammad Abdul Halim, Assistant Professor in Mathematics


Page 40 of 249

Let 𝑣 = 𝑥 + 𝑦 + 6
𝑑𝑣 𝑑𝑦
𝑜𝑟, 𝑑𝑥 = 1 + 𝑑𝑥
𝑑𝑦 𝑑𝑣
𝑜𝑟, 𝑑𝑥 = 𝑑𝑥 − 1

From (i) we get,


𝑑𝑣
𝑑𝑥
− 1 = sin 𝑣
𝑑𝑣
𝑜𝑟, 𝑑𝑥 = 1 + sin 𝑣

Separating the variables, we obtain


𝑑𝑣
𝑑𝑥 = 1+sin 𝑣
1−sin 𝑣
𝑜𝑟, 𝑑𝑥 = (1+sin 𝑣)(1−sin 𝑣) 𝑑𝑣
1−sin 𝑣
𝑜𝑟, 𝑑𝑥 = 1−𝑠𝑖𝑛2 𝑣 𝑑𝑣
1−sin 𝑣
𝑜𝑟, 𝑑𝑥 = 𝑑𝑣
𝑐𝑜𝑠2 𝑣
1 sin 𝑣
𝑜𝑟, 𝑑𝑥 = (𝑐𝑜𝑠2 𝑣 − 𝑐𝑜𝑠2 𝑣) 𝑑𝑣

𝑜𝑟, 𝑑𝑥 = (𝑠𝑒𝑐 2 𝑣 − tan 𝑣. sec 𝑣)𝑑𝑣


Now, integrating
∫ 𝑑𝑥 = ∫(𝑠𝑒𝑐 2 𝑣 − tan 𝑣. sec 𝑣)𝑑𝑣
𝑜𝑟, 𝑥 + 𝑐 = tan 𝑣 − sec 𝑣
sin 𝑣 1
𝑜𝑟, 𝑥 + 𝑐 = cos 𝑣 − cos 𝑣
sin 𝑣−1
𝑜𝑟, 𝑥 + 𝑐 = cos 𝑣
sin(𝑥+𝑦+6)−1
∴𝑥+𝑐 = cos(𝑥+𝑦+6)
(As desired)
𝑑𝑦
Problem-10: Solve 𝑑𝑥
= sin(𝑥 − 𝑦 + 5)

Solution: Given that,


𝑑𝑦
𝑑𝑥
= sin(𝑥 − 𝑦 + 5) ……………(i)

Let 𝑣 = 𝑥 − 𝑦 + 5
𝑑𝑣 𝑑𝑦
𝑜𝑟, 𝑑𝑥 = 1 − 𝑑𝑥
𝑑𝑦 𝑑𝑣
𝑜𝑟, 𝑑𝑥 = 1 − 𝑑𝑥

From (i) we get,

Mohammad Abdul Halim, Assistant Professor in Mathematics


Page 41 of 249

𝑑𝑣
1 − 𝑑𝑥 = sin 𝑣
𝑑𝑣
𝑜𝑟, 𝑑𝑥 = 1 − sin 𝑣

Separating variable we obtain,


𝑑𝑣
𝑑𝑥 = 1−sin 𝑣
1+sin 𝑣
𝑜𝑟, 𝑑𝑥 = (1−sin 𝑑𝑣
𝑣)(1+sin 𝑣)

1+sin 𝑣
𝑜𝑟, 𝑑𝑥 = 1−𝑠𝑖𝑛2 𝑣
𝑑𝑣
1+sin 𝑣
𝑜𝑟, 𝑑𝑥 = 𝑐𝑜𝑠2 𝑣
𝑑𝑣
1 sin 𝑣
𝑜𝑟, 𝑑𝑥 = (𝑐𝑜𝑠2 𝑣 + 𝑐𝑜𝑠2 𝑣) 𝑑𝑣

𝑜𝑟, 𝑑𝑥 = (𝑠𝑒𝑐 2 𝑣 + tan 𝑣𝑠𝑒𝑐 𝑣)𝑑𝑣


Now integrating,
∫ 𝑑𝑥 = ∫(𝑠𝑒𝑐 2 𝑣 + tan 𝑣𝑠𝑒𝑐 𝑣)𝑑𝑣
𝑜𝑟, 𝑥 + 𝑐 = tan 𝑣 + sec 𝑣
sin 𝑣 1
𝑜𝑟, 𝑥 + 𝑐 = cos 𝑣 + cos 𝑣
sin 𝑣+1
𝑜𝑟, 𝑥 + 𝑐 =
cos 𝑣
sin(𝑥−𝑦+5)+1
∴𝑥+𝑐 = (As desired)
cos(𝑥−𝑦+5)

𝑑𝑦
Problem-11: Solve 𝑑𝑥 = cos(𝑥 + 𝑦 + 4)

Solution: Given that,


𝑑𝑦
𝑑𝑥
= cos(𝑥 + 𝑦 + 4) ……………(i)

Let 𝑣 = 𝑥 + 𝑦 + 4
𝑑𝑣 𝑑𝑦
𝑜𝑟, 𝑑𝑥 = 1 + 𝑑𝑥
𝑑𝑦 𝑑𝑣
𝑜𝑟, = −1
𝑑𝑥 𝑑𝑥

From (i) we get,


𝑑𝑣
𝑑𝑥
− 1 = cos 𝑣
𝑑𝑣
𝑜𝑟, 𝑑𝑥 = 1 + cos 𝑣

Separating variables we obtain,


𝑑𝑣
𝑑𝑥 = 1+cos 𝑣

Mohammad Abdul Halim, Assistant Professor in Mathematics


Page 42 of 249

1−cos 𝑣
𝑜𝑟, 𝑑𝑥 = (1+cos 𝑣)(1−cos 𝑣) 𝑑𝑣
1−cos 𝑣
𝑜𝑟, 𝑑𝑥 = 1−𝑐𝑜𝑠2 𝑣
𝑑𝑣
1−cos 𝑣
𝑜𝑟, 𝑑𝑥 = 𝑑𝑣
𝑠𝑖𝑛2 𝑣
1 cos 𝑣
𝑜𝑟, 𝑑𝑥 = (𝑠𝑖𝑛2 𝑣 − 𝑠𝑖𝑛2 𝑣) 𝑑𝑣

𝑜𝑟, 𝑑𝑥 = (𝑐𝑜𝑠𝑒𝑐 2 𝑣 − cot 𝑣. 𝑐𝑜𝑠𝑒𝑐 𝑣)𝑑𝑣


Now integrating,
∫ 𝑑𝑥 = ∫(𝑐𝑜𝑠𝑒𝑐 2 𝑣 − cot 𝑣. 𝑐𝑜𝑠𝑒𝑐 𝑣)𝑑𝑣
𝑜𝑟, 𝑥 + 𝑐 = − cot 𝑣 + 𝑐𝑜𝑠𝑒𝑐 𝑣
cos 𝑣 1
𝑜𝑟, 𝑥 + 𝑐 = − sin 𝑣 + sin 𝑣
− cos 𝑣+1
𝑜𝑟, 𝑥 + 𝑐 = sin 𝑣
1−cos(𝑥+𝑦+4)
𝑜𝑟, 𝑥 + 𝑐 = [ sin(𝑥+𝑦+4)
] (As desired)

Short Questions
1. What do you mean by reducible to variable separable form?
Broad Questions
Type 01:

Solve the following differential equations:


dy
1. + 1 = e x− y
dx
dy
2. = 1 + e y − x +5
dx
dy
3. = sec ( x + y )
dx
2 dy
4. ( x − y ) = a2
dx
dy
= ( x + y)
2
5.
dx
dy
6. = y−x
dx
dy
7. = 1 + tan ( y − x )
dx
dy
8. = sin 2( x + y ) + cos 2( x + y )
dx
dy
9. = sec 2( x + y )
dx

Mohammad Abdul Halim, Assistant Professor in Mathematics


Page 43 of 249

dy
10. ( x + y + 1) =1
dx
 x + y − a  dy  x + y + a 
11.   = 
 x + y − b  dx  x + y + b 
dy
= ( x + y + 1)
2
12.
dx
dy 1− x − y
13. =
dx x+ y
dy 3x + 2 y
14. =
dx 3x + 2 y + 2
dy
15. = tan 2 ( x + y )
dx
dy
16. = cos ( x + y )
dx
x + yy1 1 − x2 − y 2
17. =
xy1 − y x2 + y 2
18. ( 2cos x + 3sin x ) dy = ( 3cos x + 2sin x ) dx

Type 02:
dy
1. Solve the differential equation − x tan ( y − x ) = 1 by appropriate transformation.
dx
◊◊Homogeneous Differential Equation:
Homogeneous Function:
A function in which degree of each terms are equal is called homogenous function. In other hand a function
f(x, y) is said to be homogeneous of degree n if it is possible to expressed as
𝑦 𝑥
𝑓(𝑥, 𝑦) = 𝑥 𝑛 ∅ (𝑥 ) 𝑜𝑟 𝑓(𝑥, 𝑦) = 𝑦 𝑛 ∅ (𝑦) .
A function is homogeneous of degree n if, when each of its arguments is multiplied by any number t > 0,
the value of the function is multiplied by 𝑡 𝑛 that is 𝑓(𝑡𝑥, 𝑡𝑦) = 𝑡 𝑛 𝑓(𝑥, 𝑦).
Homogeneous differential Equation:
A first order differential equation M ( x, y ) dx + N ( x, y ) dy = 0 is said to be homogenous when it is
dy  y
expressible as the derivative form = f  .
dx x
Or
dy f1 ( x, y )
An equation of the form = is said to be homogeneous differential equation in which f1 ( x, y )
dx f 2 ( x, y )
and f 2 ( x, y ) are homogeneous function of same degree in x and y.
Note:
Every homogeneous equation of the types can be easily solved reducing it to variable separable form by
dy dv
choosing the transformation y = v x such that =v+x where v is the function of x.
dx dx

Mohammad Abdul Halim, Assistant Professor in Mathematics


Page 44 of 249

Mathematical Problems based on Homogeneous differential equation

dy y  y
Problem 01: Solve = + tan   .
dx x x
Solution:
Given differential equation is,
dy y  y
= + tan   (i )
dx x x
dy dv
Let us choose the transformation y = v x such that =v+x .
dx dx
Applying transformation the equation (i) reduces to the following form
dv
v+x = v + tan v
dx
dv
x = tan v
dx
Separating the variables, we get
1
cot v dv = dx
x
Integrating both-sides, we find the desired solution
1
 cot v dv =  x dx
ln sin v + ln c = ln x
ln c sin v = ln x
x = c sin v
Substituting the value of v, we get
 y
x = c sin  
x
(As desired)
dy x + y2 2

Problem 02: Solve = .


dx 2 xy
Solution:
Given differential equation is,
dy x 2 + y 2
= (i)
dx 2 xy
dy dv
Put y = v x such that =v+x .
dx dx
Given equation takes the form or becomes
dv x 2 + v 2 x 2
v+x =
dx 2 xvx
dv 1 + v 2
v+x =
dx 2v

Mohammad Abdul Halim, Assistant Professor in Mathematics


Page 45 of 249

dv 1 + v 2
x = −v
dx 2v
dv 1 + v 2 − 2v 2
x =
dx 2v
dv 1 − v 2
x =
dx 2v
Separating the variables,
 2v  1
 2 
.dv = .dx
 1− v  x
Integrating,
2v 1
 1− v dv =  .dx
2
x
−2v 1
− dv =  .dx
1− v 2
x
− ln 1 − v = ln x + ln c
2

ln x + ln c + ln 1 − v 2 = 0
ln cx (1 − v 2 ) = 0
cx (1 − v 2 ) = e 0
 y 
2
cx 1 − 2  = 1
 x 
 x2 − y 2 
cx  2  = 1
 x 
 
cx ( x − y ) = x 2
2 2

(As desired)
Problem 03: Solve ( x + y ) dy + ( x − y ) dx = 0 .
Solution:
Given differential equation is,
( x + y ) dy + ( x − y ) dx = 0
 ( x + y ) dy = − ( x − y ) dx
 ( x + y ) dy = ( y − x ) dx
dy y − x
 = (i )
dx x + y
dy dv
Put y = v x such that =v+x .
dx dx
Above equation (i) becomes,
dv vx − x
v+x =
dx x + vx

Mohammad Abdul Halim, Assistant Professor in Mathematics


Page 46 of 249

dv v − 1
 v+x =
dx 1 + v
dv v −1
x = −v
dx 1+ v
dv v −1 − v − v2
x =
dx 1+ v

x
dv
=
(
− 1 + v2 )
dx 1+ v
Separating the variables,
 1+ v  1
 2 
dv = − dx
 1+ v  x
Integrating,
 1+ v  1
  1 + v 2 

dv = −  dx
x
 1 v 
  1 + v 2 + 1 + v 2  dv = − ln x − ln c
1 v
 1 + v 2 dv +  1 + v 2 dv = − ( ln x + ln c )
1 2v
tan −1 v +  dv = − ln cx
2 1 + v2
1
tan −1 v + ln 1 + v 2 = − ln cx
2
1
tan −1 v + ln 1 + v 2 + ln cx = 0
2
tan −1 v + ln 1 + v 2 + ln cx = 0

tan −1 v + ln cx 1 + v 2 = 0
2
y
tan −1 v + ln cx 1 + =0
x2
2
 y
−1 x2 + y
tan   + ln cx =0
x x2
 y
tan −1   + ln c x 2 + y 2 = 0
x
(As desired)
dy
Problem 04: Solve x − y = x2 + y 2 .
dx
Solution:
dy
Given differential equation is, x − y = x2 + y 2 (i )
dx

Mohammad Abdul Halim, Assistant Professor in Mathematics


Page 47 of 249

Equation (i) can be written as,

x
dy
dx
−y= (x 2
+ y2 )

x
dy
dx
= y+ (x 2
+ y2 )

dy y + (x 2
+ y2 )
 = (ii)
dx x
dy dv
Choosing y = vx such that =v+x .
dx dx
The equation (ii) becomes,

dv vx + (x 2
+ v2 x2 )
v+x =
dx x
dv vx + x (1 + v )
2

v+x =
dx x
= v + (1 + v 2 )
dv
v+x
dx
= v + (1 + v 2 ) −v
dv
x
dx
= (1 + v 2 )
dv
x
dx
dv dx
 =
(1 + v2 ) x
Integrating both sides, we get
dv dx
 = +c
(1 + v ) 2 x

 ln v + 1 + v 2 = ln x + c

 ln v + 1 + v 2 = ln x + ln c1 ; ln c1 = c 

 ln v + 1 + v 2 = ln c1 x

 v + 1 + v 2 = c1 x
2
y y
 + 1 + 2 = c1 x
x x

Mohammad Abdul Halim, Assistant Professor in Mathematics


Page 48 of 249

y x2 + y 2
 + = c1 x
x x
 y + x2 + y 2 = c1 x 2
 x 2 + y 2 = c1 x 2 − y
 x 2 + y 2 = ( c1 x 2 − y )
2

(As desired)
 y y  y y  dy
Problem 05: Solve  x cos + y sin  y =  y sin − x cos x .
 x x  x x  dx
Solution: Given differential equation is,
 y y  y y  dy
 x cos + y sin  y =  y sin − x cos x (i )
 x x  x x  dx
Equation (i) can be written as,
 y y  dy  y y
  y sin − x cos  x =  x cos + y sin  y
 x x  dx  x x
 y y
y  x cos + y sin 
= 
dy x x
 (ii )
dx  y y
x  y sin − x cos 
 x x
dy dv
Choosing y = vx such that =v+x .
dx dx
The equation (ii) becomes,
 vx vx 
vx  x cos + vx sin 
= 
dv x x
v+x
dx  vx vx 
x  vx sin − x cos 
 x x
dv v ( cos v + v sin v )
v+x =
dx ( v sin v − cos v )
dv v ( cos v + v sin v )
x = −v
dx ( v sin v − cos v )
dv v cos v + v 2 sin v − v 2 sin v + v cos v
x =
dx ( v sin v − cos v )
dv 2v cos v
x =
dx v sin v − cos v
v sin v − cos v dx
 dv = 2
v cos v x
 1 dx
  tan v −  dv = 2
 v x

Mohammad Abdul Halim, Assistant Professor in Mathematics


Page 49 of 249

Integrating,
 1 dx
   tan v − dv = 2  + c
 v x
 ln sec v − ln v = 2ln x + c
sec v
 ln = 2 ln x + ln c1 ; ln c1 = c ( say ) 
v
sec v
 ln = ln c1 x 2
v
sec v
 = c1 x 2
v
x  y
 sec   = c1 x 2
y x
 y
 sec   = c1 xy (As desired)
x
2 3
(
Problem 06: Solve x ydx − x + y dy = 0 .
3
)
Solution: Given differential equation is,
x 2 ydx − ( x 3 + y 3 ) dy = 0 (i )
Equation (i) can be written as,
x 2 ydx − ( x3 + y 3 ) dy = 0
 ( x3 + y 3 ) dy = x 2 ydx
dy x2 y
 = 3 (ii )
dx ( x + y 3 )
dy dv
Choosing y = vx such that =v+x .
dx dx
The equation (ii) becomes,
dv vx3
v+x = 3 3 3
dx x + v x
dv v
v+ x =
dx 1 + v 3
dv v
x = −v
dx 1 + v 3
dv v − v − v 4
x =
dx 1 + v3
dv −v 4
x =
dx 1 + v3
1 + v3 dx
 4 dv = −
v x

Mohammad Abdul Halim, Assistant Professor in Mathematics


Page 50 of 249

1 + v3 dx
Integrating,  v4 dv = − x + c
 1 1 dx
   4 + dv = −  + c
v v x
1
 − 3 + ln v = − ln x + c
3v
1
 ln v + ln x = 3 + c
3v
1
 ln vx = 3 + c
3v
x3
 ln y = 3 + c
3y
(As desired)
2 dy dy
Problem 07: Solve y + x = xy
2
.
dx dx
Solution: Given differential equation is,
dy dy
y 2 + x2 = xy (i )
dx dx
Equation (1) can be written as,
dy dy
y2 + x2 = xy
dx dx
 ( xy − x 2 )
dy
= y2
dx
dy y2
 = (ii )
dx ( xy − x 2 )
dy dv
Choosing y = vx such that =v+x .
dx dx
The equation (ii) becomes,
dv v2 x2
v+x = 2
dx vx − x 2
dv v2
v+ x =
dx v − 1
dv v2
x = −v
dx v − 1
dv v 2 − v 2 + v
x =
dx v −1
dv v
x =
dx v − 1
v −1 dx
 dv =
v x

Mohammad Abdul Halim, Assistant Professor in Mathematics


Page 51 of 249

Integrating,
v −1 dx
 v
dv =  + c
x
 1 dx
   1 − dv =  + c
 v x
 v − ln v = ln x + c
 ln v + ln x = v − c
 ln vx = v + ln c1 ; ln c1 = −c ( say )
 ln vx = ln ev + ln c1
 ln vx = ln c1 ev
 vx = c1 ev
y
 y = c1 e x

(As desired)

dy y ( x + y )
Problem 08: Solve + = 0.
dx x2
Solution: Given differential equation is,
dy y ( x + y )
+ =0 (i)
dx x2
Equation (i) can be written as,
dy y ( x + y )
+ =0
dx x2
dy y ( x + y)
 =− (ii )
dx x2
dy dv
Choosing y = vx such that =v+x .
dx dx
The equation (ii) becomes,
dv vx ( x + vx )
v+x =
dx x2
dv
v+x = v (1 + v )
dx
dv
x = v + v2 − v
dx
dv
x = v2
dx
1 dx
 2 dv =
v x
Integrating,
1 dx
v 2
dv = 
x
+c

Mohammad Abdul Halim, Assistant Professor in Mathematics


Page 52 of 249

1
 − = ln x + c
v
x
 − = ln x + ln c1 ; ln c1 = c ( say ) 
y
x
 − = ln c1 x
y
x
 = − ln c1 x
y
x
 = ln ( c1 x )
−1

y
x  1 
 = ln  
y  c1 x 
x
 y=
 1 
ln  
 c1 x 
(As desired)
𝑑𝑦 𝑦 𝑦
Problem 09: Solve: = +
𝑑𝑥 𝑥
cos 𝑥
Solution: Given that,
𝑑𝑦 𝑦 𝑦
= + cos ……………(i)
𝑑𝑥 𝑥 𝑥
Let 𝑦 = 𝑣𝑥
𝑑𝑦 𝑑𝑣
𝑜𝑟, = 𝑣 + 𝑥
𝑑𝑥 𝑑𝑥
From (i) we get,
𝑑𝑣
𝑣 + 𝑥 = 𝑣 + cos 𝑣
𝑑𝑥
𝑑𝑣
𝑜𝑟, 𝑥 𝑑𝑥 = cos 𝑣
Separating the variables we obtain,
𝑑𝑥 𝑑𝑣
𝑥
= cos 𝑣
𝑑𝑥
𝑜𝑟, 𝑥 = sec 𝑣𝑑𝑣
Now integrating,
𝑑𝑥
∫ 𝑥 = ∫ sec 𝑣𝑑𝑣
𝑜𝑟, ln 𝑥 = ln|sec 𝑣 + tan 𝑣| + ln 𝑐
𝑜𝑟, ln 𝑥 = ln{𝑐|sec 𝑣 + tan 𝑣|}
𝑜𝑟, 𝑥 = 𝑐|sec 𝑣 + tan 𝑣|
𝑦 𝑦
∴ 𝑥 = 𝑐 |𝑠𝑒𝑐 𝑥 + tan 𝑥 |
(As desired)
𝑑𝑦 𝑦 𝑦
Problem 10: Solve: = +
𝑑𝑥 𝑥
sec 𝑥
Solution: Given that,
𝑑𝑦 𝑦 𝑦
𝑑𝑥
= 𝑥 + sec 𝑥 …………..(i)
Let 𝑦 = 𝑣𝑥
𝑑𝑦 𝑑𝑣
𝑜𝑟, 𝑑𝑥 = 𝑣 + 𝑥 𝑑𝑥

Mohammad Abdul Halim, Assistant Professor in Mathematics


Page 53 of 249

From (i) we get,


𝑑𝑣
𝑣 + 𝑥 𝑑𝑥 = 𝑣 + sec 𝑣
𝑑𝑣
𝑜𝑟, 𝑥 = sec 𝑣
𝑑𝑥
Separating the variables we obtain,
𝑑𝑥 𝑑𝑣
𝑥
= sec 𝑣
𝑑𝑥
𝑜𝑟, 𝑥 = cos 𝑣𝑑𝑣
Now integrating,
𝑑𝑥
∫ 𝑥 = ∫ cos 𝑣𝑑𝑣
𝑜𝑟, ln 𝑥 = sin 𝑣 + ln 𝑐
𝑜𝑟, ln 𝑥 = ln 𝑒 sin 𝑣 + ln 𝑐
𝑜𝑟, ln 𝑥 = ln(𝑐. 𝑒 sin 𝑣 )
𝑜𝑟, 𝑥 = 𝑐. 𝑒 sin 𝑣
𝑦
𝑠𝑖𝑛( )
∴ 𝑥 = 𝑐. 𝑒 𝑥

(As desired)
𝑑𝑦 𝑦 𝑦
Problem 11: Solve: = + cot
𝑑𝑥 𝑥 𝑥
Solution: Given that,
𝑑𝑦 𝑦 𝑦
= + cot ……..(i)
𝑑𝑥 𝑥 𝑥
Let 𝑦 = 𝑣𝑥
𝑑𝑦 𝑑𝑣
𝑜𝑟, 𝑑𝑥 = 𝑣 + 𝑥 𝑑𝑥
From (i) we get,
𝑑𝑣
𝑣 + 𝑥 = 𝑣 + cot 𝑣
𝑑𝑥
𝑑𝑣
𝑜𝑟, 𝑥 = cot 𝑣
𝑑𝑥
Separating the variables we obtain,
𝑑𝑥 𝑑𝑣
𝑥
= cot 𝑣
𝑑𝑥
𝑜𝑟, 𝑥
= tan 𝑣

Now integrating,
𝑑𝑥
∫ 𝑥 = ∫ tan 𝑣𝑑𝑣
𝑜𝑟, ln 𝑥 = − ln|cos 𝑣| + ln 𝑐
𝑐
𝑜𝑟, ln 𝑥 = ln |cos 𝑣|
𝑐
𝑜𝑟, 𝑥 = |cos 𝑣|
𝑐
∴𝑥= 𝑦
|cos( )|
𝑥
(As desired)
𝑑𝑦 𝑦 𝑦
Problem 12: Solve: 𝑑𝑥 = 𝑥 + 𝑐𝑜𝑠𝑒𝑐 𝑥
Solution: Given that,
𝑑𝑦 𝑦 𝑦
𝑑𝑥
= 𝑥 + 𝑐𝑜𝑠𝑒𝑐 𝑥 ……….(i)
Let 𝑦 = 𝑣𝑥
𝑑𝑦 𝑑𝑣
𝑜𝑟, 𝑑𝑥 = 𝑣 + 𝑥 𝑑𝑥
From (i) we get,

Mohammad Abdul Halim, Assistant Professor in Mathematics


Page 54 of 249

𝑑𝑣
𝑣 + 𝑥 𝑑𝑥 = 𝑣 + cosec 𝑣
𝑑𝑣
𝑜𝑟, 𝑥 𝑑𝑥 = 𝑐𝑜𝑠𝑒𝑐 𝑣
Separating the variables we obtain,
𝑑𝑥 𝑑𝑣
𝑥
= 𝑐𝑜𝑠𝑒𝑐 𝑣
𝑑𝑥
𝑜𝑟, 𝑥 = sin 𝑣𝑑𝑣
Now integrating,
𝑑𝑥
∫ 𝑥 = ∫ 𝑠𝑖𝑛𝑣𝑑𝑣
𝑜𝑟, ln 𝑥 = − cos 𝑣 + ln 𝑐
𝑜𝑟, ln 𝑥 = ln 𝑒 − cos 𝑣 + ln 𝑐
𝑜𝑟, ln 𝑥 = ln(𝑐. 𝑒 − cos 𝑣 )
𝑜𝑟, 𝑥 = 𝑐. 𝑒 − cos 𝑣
𝑦
− cos( )
∴ 𝑥 = 𝑐. 𝑒 𝑥

(As desired)
𝑑𝑦 𝑥+𝑦
Problem 13: Solve: = 𝑑𝑥 𝑥
Solution: Given that,
𝑑𝑦 𝑥+𝑦
𝑑𝑥
= 𝑥 ……………(i)
Let 𝑦 = 𝑣𝑥
𝑑𝑦 𝑑𝑣
𝑜𝑟, = 𝑣 + 𝑥
𝑑𝑥 𝑑𝑥
From (i) we get,
𝑑𝑣 𝑥+𝑣𝑥
𝑣 + 𝑥 𝑑𝑥 = 𝑥
𝑑𝑣 𝑥(1+𝑣)
𝑜𝑟, 𝑣 + 𝑥 =
𝑑𝑥 𝑥
𝑑𝑣
𝑜𝑟, 𝑣 + 𝑥 𝑑𝑥 =1+𝑣
𝑑𝑣
𝑜𝑟, 𝑥 =1
𝑑𝑥
Separating the variables we obtain,
𝑑𝑥
𝑥
= 𝑑𝑣
Now integrating,
𝑑𝑥
∫ 𝑥 = ∫ 𝑑𝑣
𝑜𝑟, ln 𝑥 = 𝑣 + ln 𝑐
𝑜𝑟, ln 𝑥 = ln 𝑒 𝑣 + ln 𝑐
𝑜𝑟, ln 𝑥 = ln(𝑐. 𝑒 𝑣 )
𝑜𝑟, 𝑥 = 𝑐. 𝑒 𝑣
𝑦
∴ 𝑥 = 𝑐. 𝑒 𝑥

(As desired)
𝑑𝑦 2𝑥+𝑦
Problem 14: Solve: = 𝑑𝑥 2𝑥
Solution: Given that,
𝑑𝑦 2𝑥+𝑦
𝑑𝑥
= 2𝑥 ……………(i)
Let 𝑦 = 𝑣𝑥
𝑑𝑦 𝑑𝑣
𝑜𝑟, 𝑑𝑥 = 𝑣 + 𝑥 𝑑𝑥
From (i) we get,
𝑑𝑣 2𝑥+𝑣𝑥
𝑣 + 𝑥 𝑑𝑥 = 2𝑥

Mohammad Abdul Halim, Assistant Professor in Mathematics


Page 55 of 249

𝑑𝑣 𝑥(2+𝑣)
𝑜𝑟, 𝑣 + 𝑥 𝑑𝑥 = 2𝑥
𝑑𝑣 2+𝑣
𝑜𝑟, 𝑣 + 𝑥 =
𝑑𝑥 2
𝑑𝑣 2+𝑣
𝑜𝑟, 𝑥 𝑑𝑥 = 2 − 𝑣
𝑑𝑣 2−𝑣
𝑜𝑟, 𝑥 𝑑𝑥 = 2
Separating the variables we obtain,
𝑑𝑥 2
= 𝑑𝑣
𝑥 2−𝑣
Now integrating,
𝑑𝑥 𝑑𝑣
∫ 𝑥 = 2 ∫ 2−𝑣
𝑜𝑟, ln 𝑥 = −2 ln(2 − 𝑣) + ln 𝑐
𝑜𝑟, ln 𝑥 + 2 ln(2 − 𝑣) = ln 𝑐
𝑜𝑟, ln 𝑥 + ln(2 − 𝑣)2 = ln 𝑐
𝑜𝑟, ln{𝑥. (2 − 𝑣)2 } = ln 𝑐
𝑦 2
𝑜𝑟, 𝑥. (2 − ) = 𝑐
𝑥
(As desired)
𝑑𝑦 𝑥+2𝑦
Problem 15 : Solve: = 𝑑𝑥 2𝑥
Solution: Given that,
𝑑𝑦 𝑥+2𝑦
𝑑𝑥
= 2𝑥 ………….(i)
Let 𝑦 = 𝑣𝑥
𝑑𝑦 𝑑𝑣
𝑜𝑟, = 𝑣 + 𝑥
𝑑𝑥 𝑑𝑥
From (i) we get,
𝑑𝑣 𝑥+2𝑣𝑥
𝑣+𝑥 =
𝑑𝑥 2𝑥
𝑑𝑣 𝑥(1+2𝑣)
𝑜𝑟, 𝑣 + 𝑥 𝑑𝑥 = 2𝑥
𝑑𝑣 1+2𝑣
𝑜𝑟, 𝑥 𝑑𝑥 = 2
− 𝑣
𝑑𝑣 1
𝑜𝑟, 𝑥 𝑑𝑥 = 2
Separating the variables we obtain,
𝑑𝑥
𝑥
= 2𝑑𝑣
Now integrating,
𝑑𝑥
∫ 𝑥 = 2 ∫ 𝑑𝑣
𝑜𝑟, ln 𝑥 = 2𝑣 + ln 𝑐
𝑜𝑟, ln 𝑥 = ln 𝑒 2𝑣 + ln 𝑐
𝑜𝑟, ln 𝑥 = ln(𝑐𝑒 2𝑣 )
𝑜𝑟, 𝑥 = 𝑐𝑒 2𝑣
𝑦
∴ 𝑥 = 𝑐𝑒 2𝑥 (As desired)
𝑑𝑦 𝑥+𝑦
Problem 16: Solve: 𝑑𝑥 = 2𝑥
Solution: Given that,
𝑑𝑦 𝑥+𝑦
= ………….(i)
𝑑𝑥 2𝑥
Let 𝑦 = 𝑣𝑥
𝑑𝑦 𝑑𝑣
𝑜𝑟, 𝑑𝑥 = 𝑣 + 𝑥 𝑑𝑥
From (i) we get,
𝑑𝑣 𝑥+𝑣𝑥
𝑣 + 𝑥 𝑑𝑥 = 2𝑥

Mohammad Abdul Halim, Assistant Professor in Mathematics


Page 56 of 249

𝑑𝑣 𝑥(1+𝑣)
𝑜𝑟, 𝑣 + 𝑥 𝑑𝑥 = 2𝑥
𝑑𝑣 1+𝑣
𝑜𝑟, 𝑥 = −𝑣
𝑑𝑥 2
𝑑𝑣 1−𝑣
𝑜𝑟, 𝑥 𝑑𝑥 = 2
Separating the variables we obtain,
𝑑𝑥 2
𝑥
= 1−𝑣 𝑑𝑣
Now integrating,
𝑑𝑥 2
∫ 𝑥 = ∫ 1−𝑣 𝑑𝑣
𝑜𝑟, ln 𝑥 = −2 ln(1 − 𝑣) + ln 𝑐
𝑜𝑟, ln 𝑥 + 2 ln(1 − 𝑣) = ln 𝑐
𝑜𝑟, ln{𝑥(1 − 𝑣)2 } = ln 𝑐
𝑜𝑟, 𝑥(1 − 𝑣)2 = 𝑐
𝑦 2
∴ 𝑥 (1 − ) = 𝑐
𝑥
(As desired)
dy x − xy + y
2 2
Problem 17: Solve = .
dx xy
Solution: Given differential equation is,
dy x 2 − xy + y 2
= (i)
dx xy
dy dv
Choosing y = vx such that =v+x .
dx dx
The equation (i) becomes,
dv x 2 − xvx + v 2 x 2
v+ x =
dx xvx
dv x − vx 2 + v 2 x 2
2
 v+ x =
dx vx 2
dv 1 − v + v 2
 v+x =
dx v
dv 1 − v + v 2
x = −v
dx v
dv 1 − v + v 2 − v 2
x =
dx v
dv 1 − v
x =
dx v
Separating the variables,
v 1
dv = dx
1− v x
Integrating,
v 1
 1 − v dv =  x dx

Mohammad Abdul Halim, Assistant Professor in Mathematics


Page 57 of 249

1−1+ v
 1− v
dv = ln x + ln c

1 − (1 − v )
 1 − v dv = ln x + ln c
 1 
  1 − v − 1 dv = ln x + ln c
1
 1 − v dv −  dv = ln x + ln c
−1
− dv −  dv = ln x + ln c
1− v
− ln 1 − v − v = ln x + ln c
ln 1 − v + v + ln x + ln c = 0
ln cx 1 − v + v = 0
Substituting the value of v we get,
y y
ln cx 1 − + =0
x x
x− y y
ln cx + =0
x x
x ln c ( x − y ) + y = 0
(As desired)
Short Questions
1. What do you mean by Homogeneous differential equation?
( )
2. Show that the differential equation x − 3 y dx + 2 xydy = 0 is homogeneous.
2 2

( )
3. Show that the degree of homogeneous equation y + x + y dx − xdy = 0 is 1.
2 2

Broad Questions
Type 01:

Solve the following differential equations:


dy y ( x − 2 y )
1. =
dx x ( x − 3 y )
dy y y 2
2. 2 = +
dx x x 2
dy 3xy
3. = 2
dx y + 3x 2
dy 5 x − y
4. =
dx 5x
dy x + y 4
4
5. =
dx xy 3

Mohammad Abdul Halim, Assistant Professor in Mathematics


Page 58 of 249

dy y
6. =
dx x + xy
dy 2 xy
7. = 2
dx y − x 2
8. ( 3x + y ) dy + ( x + 3 y ) dx = 0
2 2 2 2

9. y ( y − 2 x ) dx + x ( 2 y − x ) dy = 0
2 2 2 2

10. ( x + 2 xy − y ) dx + ( y + 2 xy − x ) dy = 0
2 2 2 2

11. ( 2 x + 2 xy + y ) dx + ( x + 2 xy ) dy = 0
2 2 2

12. ( x + 4 x y + 3x y ) dx + ( x + 2 x y + y ) dy = 0
4 3 2 2 4 3 4

13. ( x x + y − y ) dx + xydy = 0
2 2 2

14. ( x + y x + y ) dx − xy x + y dy = 0
3 2 2 2 2 2

 x
  x y
x

15. 1 + e  dx + 1 −  e dy = 0
y
   y
 
x
 x

16. ye y
dx +  y − xe y
 dy = 0
 
 y  dy  y 
17.  x sin  =  y sin − x 
 x  dx  x 
18. (2 xy − x dy + ydx = 0 )
dy xy 2 − x 2 y
19. =
dx x3
dy y 3 − 3x 2 y
20. =
dx x3 + 3xy 2
dy x2 y
21. = 3
dx x + y 3
dy y + x 2 + y 2
22. =
dx x
dy
23. x − y = x x2 + y 2
dx
dy
24. x = y + 2 y2 − x2
dx
dy
25. x = y ( ln y − ln x + 1)
dx

Mohammad Abdul Halim, Assistant Professor in Mathematics


Page 59 of 249

Type 02:
1. Show that the solution of the general homogeneous equation of the first order and first degree
dy  y dv y
= f   is ln x =  + C where = v .
dx x f (v) − v x

2. Solve the homogeneous equation (y+ )


x 2 + y 2 dx − xdy = 0 by selecting appropriate
transformation.
x2

(
Prove that the solution of x + y
dy
)
= xy is e 2 y = ky where k is arbitrary constant.
2
2 2
3.
dx
4. ( 2 2 2
) k
Reduces x x + y − y dx + xydy = 0 to x 2 + y 2 = x ln   where k is arbitrary constant.
x
 y   y  y 
5. Solve x cos   dy +  x sin   − y cos    dx = 0 .
x  x  x 
6. Show that y = vx transforms a homogeneous differential equation into a separable equation and
2
(
hence solve x x + y − y dx + xydy = 0 .
2 2
)
◊◊Non-Homogeneous Differential Equation:
dy a1 x + b1 y + c1 a b 
An equation of the form = ,  1  1  is called non-Homogeneous differential
dx a2 x + b2 y + c2  a2 b2 
equation.
Note:
A nonhomogeneous differential equation can be solved by reducing it to homogeneous form.
By substituting x = x + h and y = y + k so that dx = dx and dy = dy  where h and k are constants
choosing in such a way that a1h + b1k + c1 = 0 and a2 h + b2 k + c2 = 0 .Considering the transformation
given equation reduces to the following form,
a1 x + b1 y + a1h + b1k + c1
dy
=
dx
a2 x + b2 y  + a2 h + b2 k + c2
a1 x + b1 y
dy
=
dx
a2 x + b2 y
Which is homogeneous differential equation in x and y easily solvable by putting y = vx where v is a
function of x .

Mathematical Problems based on reducible to Homogeneous differential equation

Problem 01: Solve ( 6 x − 5 y + 4) dy + ( y − 2 x − 1) dx = 0 .


Solution: Given differential equation is,
( 6x − 5 y + 4) dy + ( y − 2x −1) dx = 0
Given equation be written as
dy
=−
( y − 2 x − 1)
dx (6x − 5 y + 4)

Mohammad Abdul Halim, Assistant Professor in Mathematics


Page 60 of 249

dy 2x − y +1  a1 b1 
= (i )  = 
dx 6 x − 5 y + 4  a2 b2 

dy dy
Substituting x = x + h and y = y + k so that = to reduce the equation homogeneous form.
dx dx
Choose h and k in such a way that
2h − k + 1 = 0 (ii )
And
6h − 5k + 4 = 0 (iii )
1 1
Solving equation (ii) and (iii) by scientific calculator we get h = − and k = .
4 2
1 1
Therefore x = x − and y = y  +
4 2
The equation (ii) becomes homogeneous on taking above transformation,
dy 2 x − y
= (iv)
dx 6 x − 5 y
dy  dv
Considering a transformation y = vx such that = v + x .
dx  dx
Then the equation (iv) becomes,
dv 2 x − vx
v + x =
dx 6 x − 5vx
dv 2−v
v + x =
dx 6 − 5v
dv 2−v
x = −v
dx 6 − 5v
dv 2 − v − 6v + 5v 2
x =
dx 6 − 5v
dv 5v − 7v + 2
2
x =
dx 6 − 5v
Separating variables,
6 − 5v 1
dv = dx
5v − 7v + 2
2
x
6 − 5v 1
dv = dx
5v − 5v − 2v + 2
2
x
6 − 5v 1
dv = dx
5v ( v − 1) − 2 ( v − 1) x
6 − 5v 1
dv = dx
( v − 1)( 5v − 2 ) x

Mohammad Abdul Halim, Assistant Professor in Mathematics


Page 61 of 249

 1 
 3 −5  1
 +  dv =  dx
 v − 1 ( 5v − 2 )  x
 
Integrating,
 1 
 3 −5  1
  v − 1 + ( 5v − 2 )  dv =  x dx
 
 
1
−5 1
 v 3− 1 dv +  ( 5v − 2) dv =  x dx
1 1 5 1

3 v −1
dv − 
5v − 2
dv =  dx
x
1
ln v − 1 − ln 5v − 2 = ln x + ln c
3
ln 3 v − 1 − ln 5v − 2 = ln x + ln c
3 v −1
ln − = ln cx
5v − 2
 1
y− 
 2
−1
 
3
1
x+ 
 4  1
= c x + 
 1  4
y− 
5
2
−2
 1
x+ 
 4
2 y −1
2 −1
3
4x +1
4 4x +1
=c
2 y −1 4
5 2 −2
4x +1
4
4y − 2
3 −1
4x +1 4x +1
=c
20 y − 10 4
−2
4x +1

Mohammad Abdul Halim, Assistant Professor in Mathematics


Page 62 of 249

4 y − 2 − 4x −1
3
4x +1 4x +1
=c
20 y − 10 − 8 x − 2 4
4x +1
4 y − 4x − 3
3
4x +1 4x +1
=c
20 y − 8 x − 12 4
4x +1
4 y − 4x − 3 4 x + 1 20 y − 8 x − 12
3 =c .
4x +1 4 4x +1
4 y − 4x − 3
3 = c ( 5 y − 2 x − 3)
4x +1
4 y − 4x − 3
= c 3 ( 5 y − 2 x − 3)
3

4x +1
( 4 y − 4 x − 3) = c3 ( 4 x + 1)( 5 y − 2 x − 3)
3

(As desired)
dy x + 2 y − 3
Problem 02: Solve the differential equation =
dx 2 x + y − 3
Solution: Given differential equation is,
dy x + 2 y − 3
= (i )
dx 2 x + y − 3
Put x = x + h and y = y + k where h , k are constants.
dy dy
 =
dx dx
Then the equation (i) becomes,
dy x + 2 y + ( h + 2k − 3)
= (ii )
dx 2 x + y + ( 2h + k − 3)
Now choose
h + 2k − 3 = 0 (iii)
2h + k − 3 = 0 (iv)
Solving equations (iii) and (iv) we get,
h = 1 and k = 1
With this substitution equation (ii) becomes,
dy x + 2 y
= (v )
dx 2 x + y
Which is a homogeneous equation in x and y .
So put, y = vx
dy dv
 = v + x
dx dx

Mohammad Abdul Halim, Assistant Professor in Mathematics


Page 63 of 249

From equation (5), we have


dv x + 2vx
v + x =
dx 2 x + vx
dv 1 + 2v
 v + x =
dx 2 + v
dv 1 + 2v
 x = −v
dx 2 + v
dv 1 + 2v − 2v − v 2
 x =
dx 2+v
dv 1 − v 2
 x =
dx 2 + v
2+v dx
 dv =
1− v 2
x
Integrating both sides, we get
2+v dx
 1− v 2
dv = 
x
2+v dx
 dv = 
(1 + v )(1 − v ) x
1 1 3 1  dx
  . + . dv =  
 2 1+ v 2 1− v  x
1 dv 3 dv dx
  +  =
2 1+ v 2 1− v x
1 3
 ln (1 + v ) − ln (1 − v ) = ln x + ln c
2 2
1
 ln 1 + v + ln = ln cx
(1 − v )
3

1+ v
 ln = ln cx
(1 − v )
3

1+ v
 = cx
(1 − v )
3

1+ v
 = Cx2 ; let , c 2 = C
(1 − v )
3

 1 + v = Cx2 (1 − v )
3

y y 
3

 1 + = Cx2 1 − 
x  x 
 x + y  = C ( x − y  )
3

 x − 1 + y − 1 = C ( x − 1 − y + 1)
3

Mohammad Abdul Halim, Assistant Professor in Mathematics


Page 64 of 249

 x + y − 2 = C ( x − y)
3
(As desired)
dy
Problem 03: Solve the differential equation ( 3 x − 7 y − 3) = (3 y − 7 x + 7 )
dx
Solution: Given differential equation is,
dy
( 3x − 7 y − 3) = (3 y − 7 x + 7 )
dx
dy ( 3 y − 7 x + 7 )
 = (i )
dx ( 3 x − 7 y − 3)
Put x = x + h and y = y + k where h , k are constants.
dy dy
 =
dx dx
Then the equation (i) becomes,
dy 3 y − 7 x + ( −7 h + 3k + 7 )
= (ii )
dx 3 x − 7 y + ( 3h − 7 k − 3)
Now choose
−7h + 3k + 7 = 0 (iii)
3h − 7k − 3 = 0 (iv)
Solving equations (iii) and (iv) we get,
h = 1 and k = 0
with this substitution equation (ii) becomes,
dy 3 y − 7 x
= (v )
dx 3x − 7 y
which is a homogeneous equation in x and y .
So put, y = vx
dy dv
 = v + x
dx dx
From equation (5), we have
dv 3vx − 7 x
v + x =
dx 3 x − 7vx
dv 3v − 7
 v + x =
dx 3 − 7v
dv 3v − 7
 x = −v
dx 3 − 7v
dv 3v − 7 − 3v + 7v 2
 x =
dx 3 − 7v
dv −7 + 7v 2
x  =
dx 3 − 7v

 x
dv −7 1 − v
=
( 2
)
dx 3 − 7v
3 − 7v dx
 dv = −7
1− v 2
x

Mohammad Abdul Halim, Assistant Professor in Mathematics


Page 65 of 249

Integrating both sides, we get


3 − 7v dx
 1− v 2
dv = −7 
x
3 − 7v dx
 dv = −7 
(1 + v )(1 − v ) x
 5 2  dx
  − dv = −7  
 1+ v 1− v  x
dv dv dx
 5 − 2 = −7 
1+ v 1− v x
 5ln (1 + v ) + 2ln (1 − v ) = −7ln x + ln c
 ln (1 + v ) + ln (1 − v ) = −7 ln x + ln c
5 2

 ln (1 + v ) (1 − v ) = ln cx−7
5 2

c
 (1 + v ) (1 − v ) =
5 2

x 7
y   y 
5 2
 c
 1 +  1 −  = 7 ; as y = vx
 x   x  x
 ( x  + y  ) ( x − y  ) = c
5 2

 ( x + y − 1) ( x − y − 1) = c ; as x = x + 1 and y = y  + 0
5 2

(As desired)
dy x+ y−2
Problem 04: Solve =
dx − x + y − 4
Solution:
Given that,
dy x+ y−2
= (i)
dx − x + y − 4

Put x = x + h and y = y + k where h , k are constants.


dy dy
 =
dx dx
Then the equation (i) becomes,
dy x + y  + ( h + k − 2 )
= (ii )
dx − x + y + ( −h + k − 4 )

Now choose
h+k −2=0 (iii )
−h + k − 4 = 0 (iv)
Solving equations (iii) and (iv) we get,
h = −1 , k = 3
With this substitution equation (ii) becomes,

Mohammad Abdul Halim, Assistant Professor in Mathematics


Page 66 of 249

dy x + y
= (v )
dx − x + y
Which is a homogeneous equation in x and y .
So put, y = vx
dy dv
 = v + x
dx dx
From equation (v), we have
dv x + vx
v + x =
dx v x − x

dv v + 1
Or, x = −v
dx v −1

v −1 d x
Or, dv =
1 + 2v − v 2
x

Or, −
1
2
( )
ln 1 + 2v − v 2 = ln x + ln c

( )

Or, ln 1 + 2v − v 2 2 = ln cx
1

 y  y 2  2
Or, 1 + 2 − 2  = cx
 x x 
1

 x2 + 2 xy − y2  2
Or,   = cx
 x 2 
1

1  x2 + 2 xy − y2  2
Or,   = cx
x−1  x 2 
1

( )

Or, x2 + 2 xy − y2 2 =c

( )
1
2 −2
( x + 1) + 2 ( x + 1)( y − 3) − ( y − 3)
2
Or, =c

Or, (x 2
+ 2 x + 1 + 2 xy − 6 x + 2 y − 6 − y 2 + 6 y − 9 = c −2 )
(
 x 2 + 2 xy − y 2 − 4 x + 8 y − 14 = c −2 )
(As desired)

Mohammad Abdul Halim, Assistant Professor in Mathematics


Page 67 of 249

dy x+2 y+3
Problem 05: Solve =
dx 2x + y + 3

Solution:
Given that,
dy x+2 y+3
= (i )
dx 2x + y + 3

Put x = x + h and y = y + k where h , k are constants.


dy dy
 =
dx dx
Then the equation (i) becomes,
d y x + 2 y  + ( h + 2 k + 3 )
= (ii )
d x 2 x + y  + ( 2h + k + 3 )

Now choose
h + 2k + 3 = 0 (iii )
2h + k + 3 = 0 (iv)
Solving equations (iii) and (iv) we get,
 h k 1 
h = −1 , k = − 1  6 − 3 = 6 − 3 = 1 − 4 

 Equation (ii) reduces to,


d y x + 2 y 
= (v )
d x 2 x + y 

Which is a homogeneous equation in x and y .


So put, y = vx
dy dv
 = v + x
dx dx
From equation (v), we have
dv x + 2v x
v + x =
dx 2 x + v x

dv 1 + 2v 1 + 2v − 2v − v 2
Or, x = −v =
dx 2 + v 2+v

2+v dx
Or, dv =
1 −v 2
x

 2 v  dx
Or,  + 2
dv =
1 − v
2
1 −v  x

Mohammad Abdul Halim, Assistant Professor in Mathematics


Page 68 of 249

 2 1 ( −2v )  dx
Or,  − 2 
dv =
1 −v
2 2
2 1 −v  x

1+ v
Or, 2
1
ln
2.1 1 − v
1
(
− ln 1 − v 2 = ln x + ln c
2
)
1+ v
Or, 2ln
1− v
(
− ln 1 − v 2 = 2ln cx )
2
1+ v
Or, ln
1− v
(
= ln c 2 x2 + ln 1 − v 2 )
2
 1+ v 
Or,   = c x 1 − v
2 2 2
( )
 1− v 

y 
2

 1 + x   y 
2
Or,  
= c x 1 − 2 
2 2

 1− y   x 
 
x 
2
 x + y 
Or,     = c x − y
2 2 2
( )
 x −y 

 x + y = c2 ( x − y ) 3

 x + 1 + y + 1 = c 2 (x + 1 − y − 1) 3

 x + y + 2 = c 2 (x − y ) 3 (As desired)

H.W:
dy
1. Solve ( x + y + 3) = y − x +1
dx
2. Solve ( 4 x + 3 y + 1) dx + ( 2 y + 3x + 1) dy = 0

Special Type 01:


dy a1 x + b1 y + c1  a1 b1 1 
An equation of the form = ,  = =  is called Non-Homogeneous differential
dx a2 x + b2 y + c2  a2 b2 m 
dy a1 x + b1 y + c1
equation. This equation may be written as = and it is solvable by putting
dx m ( a1 x + b1 y ) + c2
a1 x + b1 y = v .

Mohammad Abdul Halim, Assistant Professor in Mathematics


Page 69 of 249

dy
Problem 01: Solve the differential equation ( 2 x − 2 y + 5 ) = x− y+3
dx
Solution: Given differential equation is,
dy
( 2 x − 2 y + 5)= x− y+3
dx
dy x− y +3
 =
dx 2 x − 2 y + 5
dy x− y +3
 = … …. … (1)
dx 2 ( x − y ) + 5
Put x− y =v
dy dv dy dv
 1− =  = 1−
dx dx dx dx
Then the equation (1) becomes,
dv v + 3
1− =
dx 2v + 5
dv v+3
 = 1−
dx 2v + 5
dv 2v + 5 − v − 3
 =
dx 2v + 5
dv 2 + v
 =
dx 2v + 5
2v + 5
 dv = dx
2+v
Integrating both sides, we get
2v + 5
2+v
dv =  dx

2 (2 + v) +1
 dv =  dx
2+v
 1 
 2+ dv =  dx
 2+v 
 2v + ln ( 2 + v ) = x + c
 2 ( x − y ) + ln ( x − y + 2) = x + c
 x − 2 y + ln ( x − y + 2 ) = c
(As desired)
dy
Problem 02: Solve the differential equation ( 3 x − 2 y + 1) = 6x − 4 y + 3
dx
Solution: Given differential equation is,
dy
( 3x − 2 y + 1) = 6x − 4 y + 3
dx

Mohammad Abdul Halim, Assistant Professor in Mathematics


Page 70 of 249

dy 6 x − 4 y + 3
 =
dx 3x − 2 y + 1
dy 2 ( 3x − 2 y ) + 3
 = … …. … (1)
dx 3x − 2 y + 1
Put 3x − 2 y = v
dy dv dy 1  dv 
 3− 2 =  = 3− 
dx dx dx 2  dx 
Then the equation (1) becomes,
1 dv  2v + 3
3−  =
2 dx  v + 1
dv 4v + 6
 3− =
dx v +1
dv 4v + 6
 = 3−
dx v +1
dv 3v + 3 − 4v − 6
 =
dx v +1
dv −v − 3
 =
dx v +1
v + 1 dv
 = − dx
v + 3 dx
Integrating both sides, we get
v +1
 v + 3dv = − dx

( v + 3) − 2dv = −
v+3  dx
 2 
  1 − dv = −  dx
 v+3
 ( 3x − 2 y ) − 2ln ( 3x − 2 y + 3) = − x + c
 4 x − 2 y − 2ln ( 3x − 2 y + 3) = c
(As desired)
𝑑𝑦 3𝑥+2𝑦
Problem 03: Solve 𝑑𝑥
= 3𝑥+2𝑦+2

Solution: Given that,


𝑑𝑦 3𝑥+2𝑦
𝑑𝑥
= 3𝑥+2𝑦+2 …………..(i)

Let 𝑣 = 3𝑥 + 2𝑦
𝑑𝑣 𝑑𝑦
𝑜𝑟, 𝑑𝑥 = 3 + 2 𝑑𝑥
𝑑𝑦 1 𝑑𝑣
𝑜𝑟, 𝑑𝑥 = 2 (𝑑𝑥 − 3)

From (i) we get,

Mohammad Abdul Halim, Assistant Professor in Mathematics


Page 71 of 249

1 𝑑𝑣 𝑣
( − 3) =
2 𝑑𝑥 𝑣+2
𝑑𝑣 2𝑣
𝑜𝑟, 𝑑𝑥 − 3 = 𝑣+2
𝑑𝑣 2𝑣
𝑜𝑟, 𝑑𝑥 = 𝑣+2 + 3
𝑑𝑣 5𝑣+6
𝑜𝑟, 𝑑𝑥 = 𝑣+2

Separating the variables we obtain,


𝑣+2
𝑑𝑥 = 5𝑣+6 𝑑𝑣
1 5𝑣+10
𝑜𝑟, 𝑑𝑥 = 5 ( 5𝑣+6 ) 𝑑𝑣
(5𝑣+6)+4
𝑜𝑟, 5𝑑𝑥 = 𝑑𝑣
5𝑣+6
4
𝑜𝑟, 5𝑑𝑥 = (1 + 5𝑣+6) 𝑑𝑣

Now, integrating
4
5 ∫ 𝑑𝑥 = ∫ (1 + 5𝑣+6) 𝑑𝑣
4
𝑜𝑟, 5𝑥 = 𝑣 + 5 ln(5𝑣 + 6) + 𝑐
4
𝑜𝑟, 5𝑥 = 3𝑥 + 2𝑦 + 5 ln(15𝑥 + 10𝑦 + 6) + 𝑐
4
𝑜𝑟, 2𝑥 = 2𝑦 + ln(15𝑥 + 10𝑦 + 6) + 𝑐
5

∴ 10𝑥 = 10𝑦 + 4 ln(15𝑥 + 10𝑦 + 6) + 5𝑐


𝑑𝑦 4𝑥−10𝑦+3
Problem 04: Solve 𝑑𝑥
= 2𝑥−5𝑦+5

Solution: Given that,


𝑑𝑦 4𝑥−10𝑦+3
𝑑𝑥
= 2𝑥−5𝑦+5

𝑑𝑦 2(2𝑥−5𝑦)+3
𝑜𝑟, = …….(i)
𝑑𝑥 2𝑥−5𝑦+5

Let 𝑣 = 2𝑥 − 5𝑦
𝑑𝑣 𝑑𝑦
𝑜𝑟, 𝑑𝑥 = 2 − 5 𝑑𝑥
𝑑𝑦 1 𝑑𝑣
𝑜𝑟, 𝑑𝑥 = 5 (2 − 𝑑𝑥)

From (i) we get,


1 𝑑𝑣 2𝑣+3
5
(2 − 𝑑𝑥) = 𝑣+5
𝑑𝑣 10𝑣+15
𝑜𝑟, 2 − 𝑑𝑥 = 𝑣+5

Mohammad Abdul Halim, Assistant Professor in Mathematics


Page 72 of 249

10𝑣+15 𝑑𝑣
𝑜𝑟, 2 − 𝑣+5
= 𝑑𝑥
2𝑣+10−10𝑣−15 𝑑𝑣
𝑜𝑟, 𝑣+5
= 𝑑𝑥
−8𝑣−5 𝑑𝑣
𝑜𝑟, 𝑣+5
= 𝑑𝑥

Separating the variables we obtain,


𝑣+5
𝑜𝑟, −𝑑𝑥 = 8𝑣+5 𝑑𝑣
1 8𝑣+40
𝑜𝑟, −𝑑𝑥 = 8 ( 8𝑣+5 ) 𝑑𝑣
(8𝑣+5)+35
𝑜𝑟, −8𝑑𝑥 = 𝑑𝑣
8𝑣+5
35
𝑜𝑟, −8𝑑𝑥 = (1 + 8𝑣+5) 𝑑𝑣

Now, integrating
35
−8 ∫ 𝑑𝑥 = ∫ (1 + 8𝑣+5) 𝑑𝑣
35
𝑜𝑟, −8𝑥 = 𝑣 + 8
ln(8𝑣 + 5) + 𝑐
35
𝑜𝑟, −8𝑥 = 2𝑥 − 5𝑦 + 8
𝑙𝑛(16𝑥 − 40𝑦 + 5) + 𝑐
35
∴ 5𝑦 = 10𝑥 + 8
𝑙𝑛(16𝑥 − 40𝑦 + 5) + 𝑐
𝑑𝑦 4𝑥−10𝑦+3
Problem 05: Solve 𝑑𝑥
= 6𝑥−15𝑦+5

Solution: Given that,


𝑑𝑦 4𝑥−10𝑦+3
𝑑𝑥
= 6𝑥−15𝑦+5

𝑑𝑦 2(2𝑥−5𝑦)+3
𝑜𝑟, 𝑑𝑥 = 3(2𝑥−5𝑦)+5………….(i)

Let 𝑣 = 2𝑥 − 5𝑦
𝑑𝑣 𝑑𝑦
𝑜𝑟, 𝑑𝑥 = 2 − 5 𝑑𝑥
𝑑𝑦 𝑑𝑣
𝑜𝑟, 5 𝑑𝑥 = 2 − 𝑑𝑥
𝑑𝑦 1 𝑑𝑣
𝑜𝑟, 𝑑𝑥 = 5 (2 − 𝑑𝑥)

From (i) we get,


1 𝑑𝑣 2𝑣+3
5
(2 − 𝑑𝑥) = 3𝑣+5
𝑑𝑣 10𝑣+15
𝑜𝑟, 2 − 𝑑𝑥 = 3𝑣+5
10𝑣+15 𝑑𝑣
𝑜𝑟, 2 − 3𝑣+5
= 𝑑𝑥

Mohammad Abdul Halim, Assistant Professor in Mathematics


Page 73 of 249

6𝑣+10−10𝑣−15 𝑑𝑣
𝑜𝑟, 3𝑣+5
= 𝑑𝑥
−4𝑣−5 𝑑𝑣
𝑜𝑟, 3𝑣+5
= 𝑑𝑥

Separating variables we obtain,


3𝑣+5
𝑑𝑥 = − (4𝑣+5) 𝑑𝑣
(4𝑣+5)−𝑣
𝑜𝑟, −𝑑𝑥 = 𝑑𝑣
4𝑣+5
𝑣
𝑜𝑟, −𝑑𝑥 = (1 − 4𝑣+5) 𝑑𝑣
1 4𝑣
𝑜𝑟, −𝑑𝑥 = 𝑑𝑣 − ( ) 𝑑𝑣
4 4𝑣+5
1 4𝑣+5−5
𝑜𝑟, −𝑑𝑥 = 𝑑𝑣 − 4 ( 4𝑣+5
) 𝑑𝑣
1 5
𝑜𝑟, −𝑑𝑥 = 𝑑𝑣 − 4 (1 − 4𝑣+5) 𝑑𝑣

Now integrating,
1 5
− ∫ 𝑑𝑥 = ∫ 𝑑𝑣 − 4 ∫ (1 − 4𝑣+5) 𝑑𝑣
1 5
𝑜𝑟, −𝑥 = 𝑣 − 4 (𝑣 − 4 ln(4𝑣 + 5)) + 𝑐
1 5
𝑜𝑟, −𝑥 = 𝑣 − 𝑣 + ln(4𝑣 + 5) + 𝑐
4 16
5
𝑜𝑟, −4𝑥 = 4𝑣 − 𝑣 + ln(4𝑣 + 5) + 𝑐
4
5
𝑜𝑟, −4𝑥 = 3𝑣 + 4 ln(4𝑣 + 5) + 𝑐
5
𝑜𝑟, −4𝑥 = 6𝑥 − 15𝑦 + ln(8𝑥 − 20𝑦 + 5) + 𝑐
4
5
∴ 15𝑦 = 10𝑥 + 4 ln(8𝑥 − 20𝑦 + 5) + 𝑐

H.W:
𝑑𝑦 2𝑥−5𝑦+3
(i) 𝑑𝑥
= 4𝑥−10𝑦+5
𝑑𝑦 3𝑥−2𝑦+3
(ii) 𝑑𝑥
= 6𝑥−4𝑦+4
𝑑𝑦 6𝑥−4𝑦+3
(iii) 𝑑𝑥
= 3𝑥−2𝑦+1

Special Type 02:


dy a1 x + b1 y + c1
An equation of the form = , ( b1 = − a2 ) is called Non-Homogeneous differential
dx a2 x + b2 y + c2
equation. This equation may be written as

Mohammad Abdul Halim, Assistant Professor in Mathematics


Page 74 of 249

dy a x + b1 y + c1
= 1
dx −b1 x + b2 y + c2
 ( a1 x + b1 y + c1 ) dx = ( −b1x + b2 y + c2 ) dy
 a1 xdx + b1 ydx + c1dx = −b1 xdy + b2 ydy + c2 dy
 a1 xdx + b1 ydx + c1dx + b1 xdy − b2 ydy − c2 dy = 0
 a1 xdx + b1 ydx + b1 xdy + c1dx − b2 ydy − c2 dy = 0
 a1 xdx + b1d ( xy ) + c1dx − b2 ydy − c2dy = 0
By integrating we can solve it.
dy 2 x − y + 1
Problem 01: Solve = .
dx x − 2 y + 1
Solution:
Given differential equation is
dy 2 x − y + 1
=
dx x − 2 y + 1
Here coefficient of y in numerator is equal to coefficient of x in the denominator with sign changed.
Rearranging the given differential equation’s
( x − 2 y + 1) dy = ( 2x − y + 1) dx
xdy − 2 ydy + dy = 2 xdx − ydx + dx
xdy − 2 ydy + dy = 2 xdx − ydx + dx
xdy + ydx − 2 ydy + dy − 2 xdx − dx = 0
d ( xy ) − 2 ydy + dy − 2 xdx − dx = 0
Now, integrating both-sides we get,
xy − y 2 + y − x 2 − x = c
(As desired)
dy 2 x − y + 1
Problem 02: Solve = .
dx x + 2 y − 3
Solution:
Given differential equation is
dy 2 x − y + 1
=
dx x + 2 y − 3
Here coefficient of y in numerator is equal to coefficient of x in the denominator with sign changed.
Rearranging the given differential equation’s
( x + 2 y − 3) dy = ( 2x − y + 1) dx
xdy + 2 ydy − 3dy = 2 xdx − ydx + dx
xdy + ydx + 2 ydy − 3dy − 2 xdx − dx = 0
d ( xy ) + 2 ydy − 3dy − 2 xdx − dx = 0
Now, integrating both-sides we get,
xy + y 2 − 3 y − x 2 − x = c (As desired)
dy
◊◊ Linear Differential Equation: A differential equation of the form + Py = Q where P and Q
dx
function of x only or constant is called linear differential equation of first order and first degree.

Mohammad Abdul Halim, Assistant Professor in Mathematics


Page 75 of 249

Method of Solving Linear Differential Equation:


dy
Assume a linear differential equation + Py = Q (i )
dx
Multiplying both-sides of above equation by e 
Pd x
we find
e + Pye  = Q e
P d x dy P d x Pd x

dx
d   P dx   Pd x
ye = Q e
dx  
Integrating with respect to x and adding an arbitrary constant C we get the desired solution.
ye =  Q e
P dx Pd x
dx + C
This is the required solution of the linear differential equation.
Note:
Integrating Factor:
An algebraic factor by multiplying which a differential equation be solvable or integrable.
On other hand Integrating Factor is a factor which makes an expression readily (directly) integrable after
multiplying by it.
Or
A differential equation is multiplied by a factor (function of x,y) and it becomes integrable, such type of
factor is called integrating factor. It is shortly denoted by I.F.
dy
1. An integrating factor of the equation + Py = Q where P and Q function of x only or constant
dx
is I .F = e 
Pd x
.
dx
2. An integrating factor of the equation + Px = Q where P and Q function of y only or constant
dy
is I .F = e 
Pd y
.
dy
3. The solution of + Py = Q is as like y  I .F =  ( Q  I .F ) dx + C .
dx
Mathematical Problems based on linear differential equation
dy
Problem 01: Solve + xy = x .
dx
Solution:
Given differential equation is
dy
+ xy = x (i )
dx
Here P = x and Q = x
x2
Integrating factor, I.F= e 
xd x
=e2
x2
2
Multiplying the above equation by e , we get
x2 x x 2 2
dy
e 2
+ xye = xe 2
2
dx

Mohammad Abdul Halim, Assistant Professor in Mathematics


Page 76 of 249

d  x2 
2 2
x

 ye  = xe 2
dx  
Integrating, we get
x2 x2
ye 2
=  xe dx 2

x2 1
Let = z such that .2 xdx = dz  xdx = dz
2 2
Now,
x2
ye 2
=  e z dz
x2 x2
ye 2
= e +C 2

x2

y = 1 + Ce 2

(As desired)
Problem 02: Solve (1 − x 2 )
dy
− xy = 1.
dx
Solution: Given differential equation is,

(1 − x ) dy
2

dx
− xy = 1 (i )
Equation (i) can be written as,

(1 − x ) dy
2

dx
− xy = 1
dy x 1
 − y= (ii )
dx (1 − x )
2
(1 − x 2 )
−x
 1− x2 dx
Integrating factor, I.F = e
1
(
ln 1− x 2 )
= e2
1

=e
( ln 1− x2 ) 2

1
= (1 − x 2 ) 2
= 1 − x2
Multiplying the above equation (ii) by 1 − x 2 , we get
dy x 1 − x 2 1 − x2
1− x − 2
y=
dx (1 − x 2 ) (1 − x2 )
dy x 1
 1 − x2 − y=
dx 1− x 2
1 − x2

d
dx
(
y 1 − x2 =
1
1 − x2
)

Mohammad Abdul Halim, Assistant Professor in Mathematics


Page 77 of 249

Integrating both sides, we get


dx
y 1 − x2 =  +c
1 − x2
 y 1 − x 2 = sin −1 ( x ) + c (As desired)
dy
Problem 03: Solve x + 2 y = x 2 log x .
dx
Solution: Given differential equation is,
dy
x + 2 y = x 2 log x (i )
dx
Equation (i) can be written as,
dy
x + 2 y = x 2 log x
dx
dy 2
 + y = x log x (ii )
dx x
2
 x dx
Integrating factor,I.F = e
= e 2ln x
= eln x
2

= x2
Multiply both sides of equation (ii) by x 2 , we get
dy
x2 + 2 xy = x 3 log x
dx
 ( x 2 y ) = x 3 log x
d
dx
Integrating both sides, we get
x 2 y =  x 3 log x + c
d 
 x 2 y = log x  x 3dx −   ( log x )  x 3dx dx + c
 dx 
4 4
x 1 x
 x 2 y = log x −  . dx + c
4 x 4
4
x 1
 x 2 y = log x −  x3dx + c
4 4
4
x 1 x4
 x 2 y = log x − . + c
4 4 4
4
x x4
 x 2 y = log x − + c
4 16
2 2
x x
 y = log x − + cx − 2
4 16
(As desired)
dy
Problem 04: Solve the differential equation + 2 y tan x = sin x .
dx

Mohammad Abdul Halim, Assistant Professor in Mathematics


Page 78 of 249

Solution: Given differential equation is,


dy
+ 2 y tan x = sin x … … … (1)
dx
This is a linear equation of first order.
I.F = e 
2 tan xdx

= e2ln(sec x)
ln ( sec2 x )
=e
= sec 2 x
Multiply both sides of equation (1) by sec 2 x , we get
dy
sec 2 x + 2 y sec 2 x tan x = sin x sec 2 x
dx
 ( y sec 2 x ) = sec x tan x
d
dx
Integrating both sides, we get
y sec 2 x =  sec x tan xdx + c
 y sec2 x = sec x + c
1 c
y= +
sec x sec 2 x
 y = cos x + c cos 2 x
which is the required solution.
dy 1
Problem 05: Solve the differential equation x − 2 y = x 2 + sin 2 .
dx x
Solution: Given differential equation is,
dy 1
x − 2 y = x 2 + sin 2 … … … (1)
dx x
The equation (1) can be written as,
dy 1
x − 2 y = x 2 + sin 2
dx x
dy 2 1 1
 − y = x + sin 2 … … … (2)
dx x x x
This is a linear equation of first order.
−2
= e x
dx
I.F
= e −2ln x
−2
= eln x
1
= 2
x
1
Multiply both sides of equation (2) by
, we get
x2
1 dy 2 1 1 1
2
− 3 y = + 3 sin 2
x dx x x x x

Mohammad Abdul Halim, Assistant Professor in Mathematics


Page 79 of 249

d  y  1 1 1
  2  = + 3 sin 2
dx  x  x x x
Integrating both sides, we get
y dx 1 1
2
=  +  3 sin 2 dx + c
x x x x
y 1 1
 2 = ln x +  3 sin 2 dx + c
x x x
y 1 1
 2 = ln x −  sin tdt + c ; putting 2 = t
x 2 x
y 1
 2 = ln x + cos t + c
x 2
y 1 1
 2 = ln x + cos 2 + c
x 2 x
2
x 1
 y = x 2 ln x + cos 2 + c x 2
2 x
which is the required solution.
dy
Problem 06: Solve the differential equation − 2 y cos x = −2sin 2 x .
dx
Solution: Given differential equation is,
dy
− 2 y cos x = −2sin 2 x … … … (1)
dx
This is a linear equation of first order.
I.F = e 
−2cos xdx

= e −2sin x
Multiply both sides of equation (1) by e −2sin x , we get
dy
e −2sin x − 2 ye −2sin x cos x = −2e −2sin x sin 2 x
dx

d
dx
( )
ye −2sin x = −2e −2sin x sin 2 x
Integrating both sides, we get
ye −2sin x = −2 e −2sin x sin 2 xdx + c
 ye −2sin x = −2  e −2sin x .2sin x cos xdx + c
 ye −2sin x = −  tet dt + c ; putting − 2sin x = t
 d  
 ye−2sin x = − t  et dt −   ( t )  et dt dt  + c
  dt  
 ye −2sin x
= −te +  e dt + c
t t

 ye−2sin x = −tet + et + c
 ye−2sin x = −et ( t − 1) + c
 ye−2sin x = −e−2sin x ( −2sin x − 1) + c

Mohammad Abdul Halim, Assistant Professor in Mathematics


Page 80 of 249

 y = 2sin x + 1 + c e2sin x
which is the required solution.
Problem 07: Solve the differential equation 1 + y 2 dx + x − tan −1 y dy = 0 . ( ) ( )
Solution: Given differential equation is,
(1 + y ) dx + ( x − tan y ) dy = 0 … … … (1)
2 −1

The equation (1) can be written as,


(1 + y ) dx + ( x − tan y ) dy = 0
2 −1

 (1 + y ) dx = − ( x − tan y ) dy
2 −1

 (1 + y ) = − x + tan y
dx 2 −1

dy
dx x tan −1 y
 =− +
dy 1+ y2 1+ y2
dx x tan −1 y
 + = … … … (2)
dy 1 + y 2 1 + y 2
This is a linear equation of first order.
1
 1+ y2 dy
I.F = e
−1
= e tan y

−1
Multiply both sides of equation (2) by e tan y
, we get
dx−1 x tan −1 y tan −1 y tan−1 y
etan + y
e = e
dy 1 + y 2 1+ y2


d
dy
−1
xetan y = (
tan −1 y tan −1 y
1+ y2
e )
Integrating both sides, we get
tan −1 y tan−1 y
=
−1
xetan y
e dy + c
1+ y2
=  tet dt + c
−1
 xetan y
; putting tan −1 y = t
= et ( t − 1) + c
−1
 xe tan y

( tan y − 1) + c
−1 −1
−1
 xetan y
= etan y

 x = ( tan −1 y − 1) + ce − tan −1 y

which is the required solution.


Problem 08: Solve the differential equation x + 2 y 3 ( ) dy
dx
=y.
Solution: Given differential equation is,

( x + 2 y ) dy
dx
=y 3
… … … (1)

Mohammad Abdul Halim, Assistant Professor in Mathematics


Page 81 of 249

dx x + 2 y 3
The equation (1) can be written as, =
dy y
dx x
 = + 2 y2
dy y
dx x
 − = 2 y 2 … … … (2)
dy y
This is a linear equation of first order.
−1
 y dy
I.F = e
= e− ln y
−1
= eln y
1
=
y
1
Multiply both sides of equation (2) by , we get
y
1 dx x
 − = 2y
y dy y 2
d x
   = 2y
dy  y 
Integrating both sides, we get
x
= 2 ydy + c
y
x y2
 = 2. + c
y 2
 x = y + cy
3

which is the required solution.


𝑑𝑦
Problem 09: Solve: cos 2 𝑥 𝑑𝑥 + 𝑦 = tan 𝑥

Solution: Given equation is


𝑑𝑦
cos2 𝑥 + 𝑦 = tan 𝑥
𝑑𝑥
𝑑𝑦 𝑦 tan 𝑥
𝑜𝑟, + =
𝑑𝑥 cos 𝑥 cos2 𝑥
2

𝑑𝑦
𝑜𝑟, + 𝑦 sec 2 𝑥 = sec 2 𝑥 tan 𝑥
𝑑𝑥
𝑑𝑦
Which is first order differential equation of the form 𝑑𝑥
+ 𝑃(𝑥)𝑦 = 𝑄(𝑥), where 𝑃 = sec 2 𝑥 and 𝑄 =
sec 2 𝑥 tan 𝑥.
2 𝑥𝑑𝑥
Now, 𝐼. 𝐹 = 𝑒 ∫ 𝑃𝑑𝑥 = 𝑒 ∫ sec = 𝑒 tan 𝑥

Mohammad Abdul Halim, Assistant Professor in Mathematics


Page 82 of 249

∴ Solution,

𝑦𝑒 ∫ 𝑃(𝑥)𝑑𝑥 = ∫ 𝑒 ∫ 𝑃(𝑥)𝑑𝑥 𝑄(𝑥)𝑑𝑥 + 𝑐

𝑜𝑟, 𝑦𝑒 tan 𝑥 = ∫ 𝑒 tan 𝑥 sec 2 𝑥 tan 𝑥 𝑑𝑥 + 𝑐

𝑜𝑟, 𝑦𝑒 tan 𝑥 = ∫ tan 𝑥 𝑒 tan 𝑥 𝑑(tan 𝑥)𝑑𝑥 + 𝑐

𝑜𝑟, 𝑦𝑒 tan 𝑥 = tan 𝑥 𝑒 tan 𝑥 − 𝑒 tan 𝑥 + 𝑐


𝑜𝑟, 𝑦 = 𝑒 − tan 𝑥 (tan 𝑥 𝑒 tan 𝑥 − 𝑒 tan 𝑥 ) + 𝑐𝑒 − tan 𝑥
𝑜𝑟, 𝑦 = tan 𝑥 − 1 + 𝑐𝑒 − tan 𝑥 . [where c arbitrary constant.] [Ans.]
𝒅𝒚
Problem 10: Solve: + 𝟐𝒚 = 𝟒𝒙
𝒅𝒙

Solution: Given that,


𝑑𝑦
+ 2𝑦 = 4𝑥………….(i)
𝑑𝑥

I.F=𝑒 ∫ 2𝑑𝑥 = 𝑒 2𝑥
Multiplying equation (i) by the I.F. 𝑒 2𝑥 we get,
𝑑𝑦
𝑒 2𝑥 + 2𝑦𝑒 2𝑥 = 4𝑥𝑒 2𝑥
𝑑𝑥
𝑑
𝑜𝑟, (𝑦𝑒 2𝑥 ) = 4𝑥𝑒 2𝑥
𝑑𝑥

Now integrating,
𝑦𝑒 2𝑥 = 4 ∫ 𝑥𝑒 2𝑥 𝑑𝑥
𝑒 2𝑥 𝑒 2𝑥
𝑜𝑟, 𝑦𝑒 2𝑥 = 4 [𝑥 2
− ∫1 2
𝑑𝑥]

𝑜𝑟, 𝑦𝑒 2𝑥 = 2𝑥𝑒 2𝑥 − 2 ∫ 𝑒 2𝑥 𝑑𝑥
𝑒 2𝑥
𝑜𝑟, 𝑦𝑒 2𝑥 = 2𝑥𝑒 2𝑥 − 2 +𝑐
2

𝑜𝑟, 𝑦𝑒 2𝑥 = 2𝑥𝑒 2𝑥 − 𝑒 2𝑥 + 𝑐
∴ 𝑦 = 2𝑥 − 1 + 𝑐𝑒 −2𝑥
𝒅𝒚
Problem 11: Solve + 𝒚 = 𝒆𝟓𝒙
𝒅𝒙

Solution: Given that,


𝑑𝑦
+ 𝑦 = 𝑒 5𝑥 ……………….(i)
𝑑𝑥

I.F. =𝑒 ∫ 1𝑑𝑥 = 𝑒 𝑥
Multiplying equation (i) by the I.F. 𝑒 𝑥 we get,
𝑑𝑦
𝑒 𝑥 𝑑𝑥 + 𝑦𝑒 𝑥 = 𝑒 5𝑥 . 𝑒 𝑥

Mohammad Abdul Halim, Assistant Professor in Mathematics


Page 83 of 249

𝑑
𝑜𝑟, 𝑑𝑥 (𝑦𝑒 𝑥 ) = 𝑒 6𝑥

Now integrating,
𝑦𝑒 𝑥 = ∫ 𝑒 6𝑥 𝑑𝑥
𝑒 6𝑥
𝑜𝑟, 𝑦𝑒 𝑥 = +𝑐
6

𝑒 5𝑥
∴𝑦= 6
+ 𝑐𝑒 −𝑥
𝒅𝒚
Problem 12: Solve 𝒅𝒙
− 𝒚 = 𝒆−𝟓𝒙

Solution: Given that,


𝑑𝑦
− 𝑦 = 𝑒 −5𝑥 ………………(i)
𝑑𝑥

I.F. =𝑒 ∫ −1𝑑𝑥 = 𝑒 −𝑥
Multiplying equation (i) by the I.F. 𝑒 −𝑥 we get,
𝑑𝑦
𝑒 −𝑥 𝑑𝑥 + 𝑦𝑒 −𝑥 = 𝑒 −5𝑥 . 𝑒 −𝑥
𝑑
𝑜𝑟, 𝑑𝑥 (𝑦𝑒 −𝑥 ) = 𝑒 −6𝑥

Now integrating,
𝑦𝑒 −𝑥 = ∫ 𝑒 −6𝑥 𝑑𝑥
𝑒 −6𝑥
𝑜𝑟, 𝑦𝑒 −𝑥 = − +𝑐
6

𝑒 −5𝑥
∴𝑦=− + 𝑐𝑒 𝑥
6
𝒅𝒚
Problem 13: Solve 𝒅𝒙
+ 𝒚 = 𝐜𝐨𝐬 𝒙

Solution: Given that,


𝑑𝑦
𝑑𝑥
+ 𝑦 = cos 𝑥………..(i)

I.F. =𝑒 ∫ 1𝑑𝑥 = 𝑒 𝑥
Multiplying equation (i) by the I.F. 𝑒 𝑥 we get,
𝑑𝑦
𝑒 𝑥 𝑑𝑥 + 𝑦𝑒 𝑥 = 𝑒 𝑥 cos 𝑥
𝑑
𝑜𝑟, 𝑑𝑥 (𝑦𝑒 𝑥 ) = 𝑒 𝑥 cos 𝑥

Now integrating,
𝑦𝑒 𝑥 = ∫ 𝑒 𝑥 cos 𝑥𝑑𝑥………………(ii)
Let 𝐼 = ∫ 𝑒 𝑥 cos 𝑥𝑑𝑥

Mohammad Abdul Halim, Assistant Professor in Mathematics


Page 84 of 249

= 𝑒 𝑥 sin 𝑥 − ∫ 𝑒 𝑥 sin 𝑥𝑑𝑥


= 𝑒 𝑥 sin 𝑥 − [𝑒 𝑥 (− cos 𝑥) − ∫ 𝑒 𝑥 (− cos 𝑥)𝑑𝑥]
= 𝑒 𝑥 sin 𝑥 + 𝑒 𝑥 cos 𝑥 − ∫ 𝑒 𝑥 cos 𝑥𝑑𝑥
= 𝑒 𝑥 (sin 𝑥 + cos 𝑥) − 𝐼
𝑜𝑟, 2𝐼 = 𝑒 𝑥 (sin 𝑥 + cos 𝑥)
𝑒𝑥
𝑜𝑟, 𝐼 = (sin 𝑥 + cos 𝑥)
2
𝑒𝑥
∴ ∫ 𝑒 𝑥 cos 𝑥𝑑𝑥 = 2
(sin 𝑥 + cos 𝑥)

From (ii) we get,


𝑒𝑥
𝑦𝑒 𝑥 = 2
(sin 𝑥 + cos 𝑥) + 𝑐
1
∴ 𝑦 = 2 (sin 𝑥 + cos 𝑥) + 𝑐𝑒 −𝑥
𝒅𝒚
Problem 14: Solve 𝒅𝒙
+ 𝒚 = 𝐬𝐢𝐧 𝒙

Solution: Given that,


𝑑𝑦
𝑑𝑥
+ 𝑦 = sin 𝑥……………….(i)

I.F. =𝑒 ∫ 1𝑑𝑥 = 𝑒 𝑥
Multiplying equation (i) by the I.F. 𝑒 𝑥 we get,
𝑑𝑦
𝑒𝑥 + 𝑦𝑒 𝑥 = 𝑒 𝑥 sin 𝑥
𝑑𝑥
𝑑
𝑜𝑟, (𝑦𝑒 𝑥 ) = 𝑒 𝑥 sin 𝑥
𝑑𝑥

Now integrating,
𝑦𝑒 𝑥 = ∫ 𝑒 𝑥 sin 𝑥𝑑𝑥…………..(ii)
Let 𝐼 = ∫ 𝑒 𝑥 sin 𝑥𝑑𝑥
= 𝑒 𝑥 (− cos 𝑥) − ∫ 𝑒 𝑥 (− cos 𝑥)𝑑𝑥
= −𝑒 𝑥 cos 𝑥 + ∫ 𝑒 𝑥 cos 𝑥𝑑𝑥
= −𝑒 𝑥 cos 𝑥 + 𝑒 𝑥 sin 𝑥 − ∫ 𝑒 𝑥 sin 𝑥𝑑𝑥
= 𝑒 𝑥 (sin 𝑥 − cos 𝑥) − 𝐼
𝑜𝑟, 2𝐼 = 𝑒 𝑥 (sin 𝑥 − cos 𝑥)
𝑒𝑥
𝑜𝑟, 𝐼 = (sin 𝑥 − cos 𝑥)
2
𝑒𝑥
∴ ∫ 𝑒 𝑥 sin 𝑥𝑑𝑥 = 2
(sin 𝑥 − cos 𝑥)

Mohammad Abdul Halim, Assistant Professor in Mathematics


Page 85 of 249

From (ii) we get,


𝑒𝑥
𝑦𝑒 𝑥 = 2
(sin 𝑥 − cos 𝑥) + 𝑐
1
∴ 𝑦 = 2 (sin 𝑥 − cos 𝑥) + 𝑐𝑒 −𝑥
𝒅𝒚 𝟐
Problem 15: Solve + 𝟐𝒙𝒚 = 𝟐𝒙𝒆𝒙
𝒅𝒙

Solution: Given that,


𝑑𝑦 2
𝑑𝑥
+ 2𝑥𝑦 = 2𝑥𝑒 𝑥 ………………(i)
𝑥2 2
I.F. =𝑒 ∫ 2𝑥𝑑𝑥 = 𝑒 2 2 = 𝑒 𝑥
2
Multiplying equation (i) by the I.F. 𝑒 𝑥 we get,
2 𝑑𝑦 2 2 2
𝑒𝑥 + 2𝑥𝑦𝑒 𝑥 = 2𝑥𝑒 𝑥 . 𝑒 𝑥
𝑑𝑥
𝑑 2 2
𝑜𝑟,
𝑑𝑥
(𝑦𝑒 𝑥 ) = 2𝑥𝑒 2𝑥

Now integrating,
2 2
𝑦𝑒 𝑥 = ∫ 2𝑥𝑒 2𝑥 𝑑𝑥 Let 𝑥 2 = 𝑧
𝑑𝑧
= ∫ 𝑒 2𝑧 𝑑𝑧 𝑜𝑟, 2𝑥 = 𝑑𝑥
𝑒 2𝑧
= 2
+𝑐 ∴ 2𝑥𝑑𝑥 = 𝑑𝑧
2
𝑒 2𝑥
= 2
+𝑐
2
𝑒𝑥 2
∴𝑦= 2
+ 𝑐𝑒 −𝑥
𝒅𝒚
Problem 16: Solve: = 𝟐𝒙𝒚 + 𝒙
𝒅𝒙

Solution: Given that,


𝑑𝑦
𝑑𝑥
= 2𝑥𝑦 + 𝑥
𝑑𝑦
𝑜𝑟, 𝑑𝑥 − 2𝑥𝑦 = 𝑥……………(i)
𝑥2 2
−2
I.F. =𝑒 ∫ −2𝑥𝑑𝑥 =𝑒 2 = 𝑒 −𝑥
2
Multiplying equation (i) by the I.F. 𝑒 −𝑥 we get,
2 𝑑𝑦 2 2
𝑒 −𝑥 𝑑𝑥
− 2𝑥𝑦𝑒 −𝑥 = 𝑥𝑒 −𝑥
𝑑 2 2
𝑜𝑟,
𝑑𝑥
(𝑦𝑒 −𝑥 ) = 𝑥𝑒 −𝑥

Now integrating,

Mohammad Abdul Halim, Assistant Professor in Mathematics


Page 86 of 249

2 2
𝑦𝑒 −𝑥 = ∫ 𝑥𝑒 −𝑥 𝑑𝑥 Let 𝑥 2 = 𝑧
𝑑𝑧 𝑑𝑧
= ∫ 𝑒 −𝑧 2
𝑜𝑟, 2𝑥 = 𝑑𝑥
𝑒 −𝑧 𝑑𝑧
=− 2
+𝑐 ∴ 𝑥𝑑𝑥 = 2
2
𝑒 −𝑥
=− +𝑐
2
1 2
∴ 𝑦 = − 2 + 𝑐𝑒 𝑥

𝒅𝒚 𝟐
Problem 17: Solve: (𝟏 + 𝒙𝟐 ) 𝒅𝒙 − 𝟐𝒙𝒚 = (𝟏 + 𝒙𝟐 )

Solution: Given that,


𝑑𝑦
(1 + 𝑥 2 ) − 2𝑥𝑦 = (1 + 𝑥 2 )2
𝑑𝑥
𝑑𝑦 2𝑥
𝑜𝑟, − 𝑦 = 1 + 𝑥 2 …………….(i)
𝑑𝑥 1+𝑥 2
2𝑥
− 𝑑𝑥 2) 2 )−1 1
I.F. =𝑒 ∫ 1+𝑥2 = 𝑒 − ln(1+𝑥 = 𝑒 ln(1+𝑥 = (1 + 𝑥 2 )−1 = 1+𝑥 2
1
Multiplying equation (i) by the I.F. 1+𝑥2 we get,
1 𝑑𝑦 2𝑥
− (1+𝑥2 )2 𝑦 = 1
1+𝑥 2 𝑑𝑥

𝑑 1
𝑜𝑟, (𝑦 ) =1
𝑑𝑥 1+𝑥 2

Now integrating,
1
𝑦 1+𝑥2 = ∫ 1𝑑𝑥
1
𝑜𝑟, 𝑦 1+𝑥2 = 𝑥 + 𝑐

∴ 𝑦 = (𝑥 + 𝑐)(1 + 𝑥 2 )
𝒅𝒚
Problem 18: Solve 𝒅𝒙
+ 𝒚𝒄𝒐𝒕 𝒙 = 𝐜𝐨𝐭 𝒙

Solution: Given that,


𝑑𝑦
𝑑𝑥
+ 𝑦𝑐𝑜𝑡 𝑥 = cot 𝑥……………(i)

I.F. = 𝑒 ∫ cot 𝑥𝑑𝑥 = 𝑒 ln sin 𝑥 = sin 𝑥


Multiplying equation (i) by the I.F. sin 𝑥 we get,
𝑑𝑦
𝑠𝑖𝑛𝑥 𝑑𝑥 + 𝑦𝑐𝑜𝑡 𝑥. sin 𝑥 = cot 𝑥. sin 𝑥

Mohammad Abdul Halim, Assistant Professor in Mathematics


Page 87 of 249

𝑑
𝑜𝑟, 𝑑𝑥 (𝑦𝑠𝑖𝑛 𝑥) = cot 𝑥. sin 𝑥

Now integrating,
𝑦𝑠𝑖𝑛 𝑥 = ∫ cot 𝑥. sin 𝑥 𝑑𝑥
cos 𝑥
𝑜𝑟, 𝑦𝑠𝑖𝑛 𝑥 = ∫ sin 𝑥 sin 𝑥 𝑑𝑥

𝑜𝑟, 𝑦𝑠𝑖𝑛 𝑥 = ∫ cos 𝑥𝑑𝑥


𝑜𝑟, 𝑦𝑠𝑖𝑛 𝑥 = sin 𝑥 + 𝑐
∴ 𝑦 = 1 + 𝑐. 𝑐𝑜𝑠𝑒𝑐 𝑥
𝒅𝒚
Problem 19: Solve 𝒅𝒙
+ 𝒚𝒄𝒐𝒕 𝒙 = 𝐬𝐞𝐜 𝒙

Solution: Given that,


𝑑𝑦
𝑑𝑥
+ 𝑦𝑐𝑜𝑡 𝑥 = sec 𝑥…………..(i)

I.F. =𝑒 ∫ cot 𝑥𝑑𝑥 = 𝑒 ln sin 𝑥 = sin 𝑥


Multiplying equation (i) by the I.F. sin 𝑥 we get,
𝑑𝑦
𝑠𝑖𝑛𝑥 𝑑𝑥 + 𝑦𝑐𝑜𝑡 𝑥. sin 𝑥 = sec 𝑥. sin 𝑥
𝑑
𝑜𝑟, 𝑑𝑥 (𝑦𝑠𝑖𝑛 𝑥) = sec 𝑥. sin 𝑥

Now integrating,
𝑦𝑠𝑖𝑛 𝑥 = ∫ sec 𝑥. sin 𝑥 𝑑𝑥
1
𝑜𝑟, 𝑦𝑠𝑖𝑛 𝑥 = ∫ cos 𝑥 sin 𝑥𝑑𝑥

𝑜𝑟, 𝑦𝑠𝑖𝑛 𝑥 = ∫ tan 𝑥𝑑𝑥


𝑜𝑟, 𝑦𝑠𝑖𝑛 𝑥 = − ln|cos 𝑥| + 𝑐
∴ 𝑦 = [− ln|cos 𝑥| + 𝑐]𝑐𝑜𝑠𝑒𝑐 𝑥
𝒅𝒚
Problem 20: Solve 𝒅𝒙
+ 𝒚𝒄𝒐𝒕 𝒙 = 𝐭𝐚𝐧 𝒙

Solution: Given that,


𝑑𝑦
𝑑𝑥
+ 𝑦𝑐𝑜𝑡 𝑥 = tan 𝑥………………(i)

I.F. = 𝑒 ∫ cot 𝑥𝑑𝑥 = 𝑒 ln sin 𝑥 = sin 𝑥


Multiplying equation (i) by the I.F. sin 𝑥 we get,
𝑑𝑦
𝑠𝑖𝑛𝑥 𝑑𝑥 + 𝑦𝑐𝑜𝑡 𝑥. sin 𝑥 = tan 𝑥. sin 𝑥
𝑑
𝑜𝑟, 𝑑𝑥 (𝑦𝑠𝑖𝑛 𝑥) = tan 𝑥. sin 𝑥

Now integrating,

Mohammad Abdul Halim, Assistant Professor in Mathematics


Page 88 of 249

𝑦𝑠𝑖𝑛 𝑥 = ∫ tan 𝑥. sin 𝑥 𝑑𝑥


sin 𝑥
𝑜𝑟, 𝑦𝑠𝑖𝑛 𝑥 = ∫ cos 𝑥 sin 𝑥𝑑𝑥

𝑠𝑖𝑛2 𝑥
𝑜𝑟, 𝑦𝑠𝑖𝑛 𝑥 = ∫ 𝑑𝑥
cos 𝑥

1−𝑐𝑜𝑠2 𝑥
𝑜𝑟, 𝑦𝑠𝑖𝑛 𝑥 = ∫ cos 𝑥
𝑑𝑥

𝑜𝑟, 𝑦𝑠𝑖𝑛𝑥 = ∫(sec 𝑥 − cos 𝑥)𝑑𝑥


𝑜𝑟, 𝑦𝑠𝑖𝑛 𝑥 = ln|sec 𝑥 + tan 𝑥| − sin 𝑥 + 𝑐
∴ 𝑦 = ln|sec 𝑥 + tan 𝑥|. 𝑐𝑜𝑠𝑒𝑐 𝑥 − 1 + 𝑐. 𝑐𝑜𝑠𝑒𝑐 𝑥
Problems for Solution
Solve the following equations
𝑑𝑦
(i) 𝑑𝑥
− 3𝑦 = cos 𝑥
𝑑𝑦 1
(ii) =− 𝑦+𝑥
𝑑𝑥 𝑥
𝑑𝑦 1
(iii) + 𝑦 = cos 𝑥
𝑑𝑥 𝑥
𝑑𝑦 2
(iv) 𝑑𝑥
+ 𝑥
𝑦 = 𝑒𝑥
𝑑𝑦
(v) + 𝑦𝑐𝑜𝑡 𝑥 = 𝑐𝑜𝑠𝑒𝑐 𝑥
𝑑𝑥
𝑑𝑦
(vi) + 𝑦𝑠𝑒𝑐 𝑥 = sin 𝑥
𝑑𝑥
𝑑𝑦
(vii) 𝑑𝑥
+ 𝑦𝑠𝑒𝑐 𝑥 = cos 𝑥
(viii) cos 𝑥 𝑑𝑦 + (𝑦 sin 𝑥 − 1)𝑑𝑥 = 0
𝑑𝑦
(ix) + 𝑦 sec 2 𝑥 = sin 𝑥 sec 3 𝑥
𝑑𝑥
𝑑𝑦
(x) (𝑥 + 𝑦 + 1) =1
𝑑𝑥
𝑑𝑦
(xi) 𝑑𝑥
+ 𝑦 tan 𝑥 − sec 𝑥 = 0
𝑑𝑦
(xii) + 𝑥𝑦 = 𝑥
𝑑𝑥
(xiii) (1 + 𝑦 2 )𝑑𝑥 = (tan−1 𝑦 − 𝑥)𝑑𝑦
(xiv) (1 + x ) dy
2

dx
+ y = tan −1
x
dy y
(xv) + = sin x
dx x
dy
x ( x − 1) − y = x 2 ( x − 1)
2
(xvi)
dx
dy
(xvii) − y sin x = sin 2 x
dx
(xviii) ( x 2 − 1)
dy
+ 2 y = ( x + 1)
2

dx
(xix) y log ydx + ( x − log y ) dy = 0
(xx) dx + xdy = e− y log ydy

Mohammad Abdul Halim, Assistant Professor in Mathematics


Page 89 of 249

dy
(xxi) − y cos ecx = sin x
dx
dy
(xxii) + 3 y = 3 x 2 e −3 x
dx
dy
+ 2 xy = 2 xe x
2
(xxiii)
dx
dy
(xxiv) + sec 2 y = sec 2 x tan x
dx
dy
(xxv) x + ( x + 1) y = x 3
dx
dy
(xxvi) + x sin 2 y = x 3 cos 2 y
dx
◊◊Equation reducible to linear differential equation (Bernoulli’s Equation):
dy
A differential equation of the form + Py = Q y n where P and Q function of x only or constant is called
dx
Bernoulli’s Equation. The method of solution was discovered by Leibnitz.
Method of Solving Bernoulli’s Equation:
dy
Bernoulli’s equation is + Py = Q y n (i )
dx
Dividing both-sides of above equation (i) by y n , we find
dy
y−n + Py 1− n = Q (ii )
dx
dy dv dy 1 dv
Putting y 1− n = v such that (1 − n ) y −n =  y −n = . .
dx dx dx (1 − n ) dx
The equation (ii) reduces to the following form,
1 dv
. + Pv = Q
(1 − n ) dx
dv
 + P (1 − n ) v = Q (1 − n )
dx
Which is a linear equation in v and x , can be solved as linear differential equation.
Mathematical Problems based on Bernoulli’s equation
dy 2 y3
Problem 01: Solve + y= 3.
dx x x
Solution:
Given differential equation is
dy 2 y3
+ y= 3 (i )
dx x x
Multiplying the above equation (i) by y −3 , we get
dy 2 −2 1
y −3 + y = 3 (ii )
dx x x

Mohammad Abdul Halim, Assistant Professor in Mathematics


Page 90 of 249

dy dv dy 1 dv
Putting y −2 = v so that −2 y −3 =  y −3 =− . .
dx dx dx 2 dx
Equation (ii) becomes,
1 dv 2 1
− . + v= 3
2 dx x x
dv 4 2
− v=− 3 (iii )
dx x x
Above equation is a linear equation in v and x.
4 2
Here, P = − and Q = − 3
x x
4 1
Now, integrating factor, I.F = e = e x = e  x = e−4ln x = eln x = x −4
P dx − dx −4 dx −4

Multiplying the equation (iii) by x −4 , we get


dv 2
x −4 − 4 x −3 v = − 7
dx x
d 2
vx −4  = − 7
dx x
Integrating both-sides, we get
2
vx −4 = −  dx
x7
vx −4 = −2  x −7 dx
 x −6 
vx −4 = −2   + C
 −6 
1
vx −4 = 6 + C
3x
3v x = 1 + 3C x6
2

3 x 2 = y 2 + 3C y 2 x6
(As desired)
dy
Problem 02: Solve the differential equation = x3 y 3 − xy .
dx
Solution: The differential equation is,
dy
= x3 y 3 − xy … … … (1)
dx
Equation (1) can be written as,
dy
= x3 y 3 − xy
dx
dy
 + xy = x 3 y 3 … … … (2)
dx
This is a Bernoulli’s equation.
Dividing the equation (2) by y 3 we get
dy
y −3 + xy −2 = x 3 … … … (3)
dx

Mohammad Abdul Halim, Assistant Professor in Mathematics


Page 91 of 249

put v = y −2
dv dy dy 1 dv
 = −2 y −3  y −3 =−
dx dx dx 2 dx
Now the equation (3) becomes,
1 dv
− + xv = x 3
2 dx
dv
 − 2 xv = −2 x 3 … … … (4)
dx
This is a linear equation.
I.F = e 
−2 xdx

= e− x
2

Multiply both sides of equation (4) by e − x we get


2

dv
e− x − 2 xve − x = −2 x 3e − x
2 2 2

dx

d
dx
2

(
ve − x = −2 x 3e − x
2

)
Integrating both sides we get
ve− x = −2 x3e− x dx + c
2 2

 ve− x = − tet dt + c ; putting − x 2 = t


2

 ve − x = −et ( t − 1) + c
2

 ve− x = −e− x ( − x2 − 1) + c
2 2

 v = x 2 + 1 + ce x
2

 y −2 = x2 + 1 + ce x
2

(
 x 2 + 1 + ce x y 2 = 1
2

)
which is the solution.
dy 2 y3
Problem 03: Solve the differential equation + y= 3.
dx x x
Solution: The differential equation is,
dy 2 y3
+ y = 3 … … … (1)
dx x x
This is a Bernoulli’s equation.
Dividing the equation (1) by y 3 we get
dy 2 −2 1
y −3 + y = 3 … … … (2)
dx x x
put v = y −2
dv dy dy 1 dv
 = −2 y −3  y −3 =−
dx dx dx 2 dx
Now the equation (2) becomes,

Mohammad Abdul Halim, Assistant Professor in Mathematics


Page 92 of 249

1 dv 2 1
− + v= 3
2 dx x x
dv 4 2
 − v = − 3 … … … (3)
dx x x
This is a linear equation.
4
I.F = e
 − x dx
dx
=e x
−4

= e − 4ln x
−4
= eln x
1
= 4
x
1
Multiply both sides of equation (4) by we get
x4
1 dv 4 2
4
− 5v=− 7
x dx x x
d  v  2
  4=− 7
dx  x  x
Integrating both sides we get
v dx
4
= −2  7 + c
x x
v 1 1
 4 = +c
x 3 x6
1 1
 y −2 = 2
+ cx 4
3x
1 1 1
 2 = + cx 4
y 3 x2
which is the required solution.

(
Problem 04: Solve the differential equation x 2 y 3 + xy ) dy
dx
= 1.
Solution: The differential equation is,

(x 2
y 3 + xy )
dy
dx
= 1 … … … (1)
Equation (1) can be written as,

(x 2
y 3 + xy )
dy
dx
=1
dy 1
 = 2 3
dx ( x y + xy )
dx
 = x 2 y 3 + xy
dy

Mohammad Abdul Halim, Assistant Professor in Mathematics


Page 93 of 249

dx
 − xy = x 2 y 3 … … … (2)
dy
This is a Bernoulli’s equation.
Dividing the equation (2) by x 2 we get
dx y
x −2 − = y 3 … … … (3)
dy x
1
put v = −
x
dv dx dx dv
 = x −2  x −2 =
dy dy dy dy
Now the equation (3) becomes,
dv
+ yv = y 3 … … … (4)
dy
This is a linear equation.
I.F = e 
ydy

y2
=e 2

y2
2
Multiply both sides of equation (4) by e we get
y2 y2 y2
dv
e 2
+ yve 2
= y 3e 2
dy

d  y 
2 2
y
  ve 2  = − y e 2
3

dy  
Integrating both sides we get
y2 y2
ve 2
=  y e dy + c
3 2

y2
y2
 ve 2
= 2 tet dt + c ; putting =t
2
y2
 ve 2
= 2et ( t − 1) + c

 y2 
y2 y2

 ve = 2e  − 1 + c
2
2

 2 
2
y
1 −
 − = y 2 − 2 + ce 2
x

Mohammad Abdul Halim, Assistant Professor in Mathematics


Page 94 of 249

2
y
1 −
 = 2 − y 2 − ce 2
x
which is the required solution.
dy y y 3
Problem 05: Solve the differential equation 2 − = .
dx x x 3
Solution: The differential equation is,
dy y y 3
2 − = … … … (1)
dx x x 3
The equation (1) can be written as,
dy y y3
− = … … … (2)
dx 2 x 2 x 3
This is a Bernoulli’s equation.
Dividing the equation (2) by y 3 we get
dy 1 −2 1
y −3 − y = 3 … … … (3)
dx 2 x 2x
put v = y −2
dv dy dy 1 dv
 = −2 y −3  y −3 =−
dx dx dx 2 dx
Now the equation (2) becomes,

dv v 1
 + = − 3 … … … (3)
dx x x
This is a linear equation.
4
= e x
− dx
I.F
dx
=e x
−4

= e − 4ln x
−4
= eln x
1
= 4
x
1
Multiply both sides of equation (4) by we get
x4
1 dv 4 2
4
− 5v=− 7
x dx x x
d  v  2
  4=− 7
dx  x  x
Integrating both sides we get
v dx
4
= −2  7 + c
x x

Mohammad Abdul Halim, Assistant Professor in Mathematics


Page 95 of 249

v 1 1
 = +c
x4 3 x6
1 1
 y −2 = + cx 4
3 x2
1 1 1
 2 = 2
+ cx 4
y 3x
which is the required solution.
dy
Problem 06: Solve the differential equation sec 2 y + 2 x tan y = x 3 .
dx
Solution: The differential equation is,
dy
sec 2 y + 2 x tan y = x 3 … … … (1)
dx
This is a Bernoulli’s equation.
put v = tan y
dv dy
 = sec 2 y
dx dx
Now the equation (1) becomes,
dv
+ 2 xv = x 3 … … … (2)
dx
This is a linear equation.
I.F = e 
2xdx

= ex
2

2
Multiply both sides of equation (2) by e x we get
dv
+ 2 xe x v = x 3e x
2 2 2
ex
dx

d
dx
( )
ve x = x 3e x
2 2

Integrating both sides we get


ve x =  x3e x dx + c
2 2

1
 ve x =  tet dt + c ; putting x 2 = t
2

2
 ve x = ( tet − et ) + c
2 1
2
et
 ve = ( t − 1) + c
2
x

2
2
ex
 ve =
x2

2
( x 2 − 1) + c
 v = ( x 2 − 1) + ce − x
1 2

2
 tan y = ( x 2 − 1) + ce − x .
1 2

Mohammad Abdul Halim, Assistant Professor in Mathematics


Page 96 of 249

Which is the required solution.


𝑑𝑦
Problem 07: Solve 𝑥 𝑑𝑥 + 𝑦 = 𝑥 2 𝑦 2 .
𝑑𝑦
Solution: Given the equation 𝑥 + 𝑦 = 𝑥 2 𝑦 2
𝑑𝑥
We first rewrite the equation as
𝑑𝑦 𝑦
𝑑𝑥
+ 𝑥 = 𝑥𝑦 2 [𝐵𝑦 𝑑𝑖𝑣𝑖𝑑𝑖𝑛𝑔 𝑏𝑦 𝑥]
𝑑𝑦 𝑑𝑢
With 𝑛 = 2, we next substitute 𝑦 = 𝑢1−𝑛 , 𝑜𝑟 𝑦 = 𝑢1−2 , 𝑜𝑟 𝑦 = 𝑢−1 and 𝑑𝑥
= −𝑢−2 𝑑𝑥 in the given
equation,

𝑑𝑢 𝑢−1
−𝑢−2 + = 𝑥𝑢−2
𝑑𝑥 −1+2𝑥
𝑑𝑢 𝑢
𝑜𝑟, − = −𝑥 [𝐷𝑖𝑣𝑖𝑑𝑒 𝑏𝑦 −𝑢−2 ]
𝑑𝑥 𝑥
𝑑𝑢 1
𝑜𝑟, − 𝑢 = −𝑥
𝑑𝑥 𝑥
𝑑𝑢 1
Which is first order differential equation of the form 𝑑𝑥
+ 𝑃(𝑥)𝑢 = 𝑄(𝑥), where 𝑃 = − 𝑥 and 𝑄 =
−𝑥.
1 −1
Now, 𝐼. 𝐹 = 𝑒 ∫ 𝑃𝑑𝑥 = 𝑒 ∫ −𝑥𝑑𝑥 = 𝑒 − ln 𝑥 = 𝑒 ln 𝑥 = 𝑥 −1
∴ Solution,
𝑢𝑒 ∫ 𝑃(𝑥)𝑑𝑥 = ∫ 𝑒 ∫ 𝑃(𝑥)𝑑𝑥 𝑄(𝑥)𝑑𝑥 + 𝑐
𝑜𝑟, 𝑢𝑥 −1 = ∫ 𝑥 −1 ∙ (−𝑥)𝑑𝑥 + 𝑐

𝑜𝑟, 𝑢𝑥 −1 = − ∫ 𝑑𝑥 + 𝑐
𝑜𝑟, 𝑢𝑥 −1 = −𝑥 + 𝑐
𝑜𝑟, 𝑢 = −𝑥 2 + 𝑐𝑥
𝑜𝑟, 𝑦 −1 = −𝑥 2 + 𝑐𝑥 [𝑆𝑖𝑛𝑐𝑒 𝑢 = 𝑦 −1 ]
1
𝑜𝑟, 𝑦 = 𝑐𝑥−𝑥2 . [where c arbitrary constant.] [Ans.]
𝒅𝒚 𝟐 𝟐 𝟑
Problem 08: Solve: + 𝟑𝒙 𝒚 = 𝒙 𝒚
𝒅𝒙
Solution: In this equation, 𝑃(𝑥) = 3𝑥 2 , 𝑄(𝑥) = 𝑥 2 , 𝑛 = 3
Let us transform the above equation by the following transformation
1
𝑧 = 𝑦1−𝑛 = 𝑦1−3 = 𝑦 −2 = 2
𝑦
After the transformation the transformed equation stands,
𝑑𝑧
+ (1 − 3)3𝑥 2 𝑧 = (1 − 3)𝑥 2
𝑑𝑥
𝑑𝑧
𝑜𝑟, 𝑑𝑥 − 6𝑥 2 𝑧 = −2𝑥 2 ,…………….(i)
which is a linear differential equation in z.
2 𝑥3 3
Now, I.F.=𝑒 ∫ −6𝑥 𝑑𝑥 = 𝑒 −6 3 = 𝑒 −2𝑥
3
Now, multiplying the equation (i) by the I.F. 𝑒 −2𝑥 ,
3 𝑑𝑧 3 3
𝑒 −2𝑥 − 6𝑥 2 𝑧𝑒 −2𝑥 = −2𝑥 2 𝑒 −2𝑥
𝑑𝑥
𝑑 3 3
𝑜𝑟, (𝑧𝑒 −2𝑥 ) = −2𝑥 2 𝑒 −2𝑥
𝑑𝑥
Now integrating,
3 3
𝑧𝑒 −2𝑥 = −2 ∫ 𝑥 2 𝑒 −2𝑥 𝑑𝑥 Let 𝑥 3 = 𝑡

Mohammad Abdul Halim, Assistant Professor in Mathematics


Page 97 of 249

𝑑𝑡 𝑑𝑡
= −2 ∫ 𝑒 −2𝑡 3
𝑜𝑟, 3𝑥 2 = 𝑑𝑥
2 −2𝑡 𝑑𝑡
= −3∫𝑒 𝑑𝑡 𝑜𝑟, 𝑥 2 𝑑𝑥 = 3
2 𝑒 −2𝑡
= − 3 ( −2 ) + 𝑐
3 1 3
𝑧𝑒 −2𝑥 = 𝑒 −2𝑥 + 𝑐
3
1 3
𝑜𝑟, 𝑧 = 3 + 𝑐𝑒 2𝑥
1 1 3
∴ 𝑦2 = 3 + 𝑐𝑒 2𝑥

𝒅𝒚
Problem 09: Solve 𝒅𝒙
+ 𝟐𝒙𝒚 = 𝒙𝒚𝟐

Solution: In this equation, 𝑃(𝑥) = 2𝑥, 𝑄(𝑥) = 𝑥, 𝑛 = 2


Let us transform the above equation by the following transformation
1
𝑧 = 𝑦1−𝑛 = 𝑦1−2 = 𝑦 −1 =
𝑦

After the transformation the transformed equation stands,


𝑑𝑧
+ (1 − 2)2𝑥𝑧 = (1 − 2)𝑥
𝑑𝑥
𝑑𝑧
𝑜𝑟, − 2𝑥𝑧 = −𝑥, …………………(i)
𝑑𝑥

which is a linear differential equation in z.


𝑥2 2
Now, I.F. =𝑒 ∫ −2𝑥𝑑𝑥 = 𝑒 −2 2 = 𝑒 −𝑥
2
Now, multiplying the equation (i) by the I.F. 𝑒 −𝑥 ,
2 𝑑𝑧 2 2
𝑒 −𝑥 − 2𝑥𝑧𝑒 −𝑥 = −𝑥𝑒 −𝑥
𝑑𝑥
𝑑 2 2
𝑜𝑟, 𝑑𝑥 (𝑧𝑒 −𝑥 ) = −𝑥𝑒 −𝑥

Now integrating,
2 2
𝑧𝑒 −𝑥 = − ∫ 𝑥𝑒 −𝑥 𝑑𝑥 Let 𝑥 2 = 𝑡
𝑑𝑡 𝑑𝑡 𝑑𝑡
= − ∫ 𝑒 −𝑡 2
𝑜𝑟, 2𝑥 = 𝑑𝑥 𝑜𝑟, 𝑥𝑑𝑥 = 2
1
= − 2 ∫ 𝑒 −𝑡 𝑑𝑡
1
= 2 𝑒 −𝑡 + 𝑐
2 1 2
𝑧𝑒 −𝑥 = 2 𝑒 −𝑥 + 𝑐

Mohammad Abdul Halim, Assistant Professor in Mathematics


Page 98 of 249

1 2
𝑜𝑟, 𝑧 = 2 + 𝑐𝑒 𝑥
1 1 2
∴ 𝑦 = 2 + 𝑐𝑒 𝑥

𝒅𝒚 𝟏
Problem 10: Solve + 𝒚 = 𝒙𝒚𝟐
𝒅𝒙 𝒙
1
Solution: In this equation, 𝑃(𝑥) = 𝑥 , 𝑄(𝑥) = 𝑥, 𝑛 = 2
Let us transform the above equation by the following transformation
1
𝑧 = 𝑦1−𝑛 = 𝑦1−2 = 𝑦 −1 =
𝑦

After the transformation the transformed equation stands,


𝑑𝑧 1
𝑑𝑥
+ (1 − 2) 𝑥 𝑧 = (1 − 2)𝑥
𝑑𝑧 1
𝑜𝑟, − 𝑧 = −𝑥, ………………(i)
𝑑𝑥 𝑥
which is a linear differential equation in z.
1 −1 1
Now, I.F. =𝑒 − ∫𝑥𝑑𝑥 = 𝑒 − ln 𝑥 = 𝑒 ln 𝑥 =
𝑥
1
Now, multiplying the equation (i) by the I.F. 𝑥,
1 𝑑𝑧 1
− 𝑧 = −1
𝑥 𝑑𝑥 𝑥 2
𝑑 1
𝑜𝑟, 𝑑𝑥 (𝑧 𝑥) = −1

Now integrating,
1
𝑧
= − ∫ 𝑑𝑥
𝑥
1
𝑜𝑟, 𝑧 = −𝑥 + 𝑐
𝑥
𝑜𝑟, 𝑧 = −𝑥 2 + 𝑐𝑥
1
∴ = −𝑥 2 + 𝑐𝑥
𝑦
𝒅𝒚 𝟏
Problem 11: Solve 𝒅𝒙
+ 𝒙𝒚 = 𝒙√𝒚
Solution: Given that,
𝑑𝑦 1
𝑑𝑥
+ 𝑥 𝑦 = 𝑥 √𝑦
1
𝑑𝑦 1
𝑜𝑟, 𝑑𝑥 + 𝑥 𝑦 = 𝑥𝑦 2
1 1
In this equation, 𝑃(𝑥) = 𝑥 , 𝑄(𝑥) = 𝑥, 𝑛 = 2
Let us transform the above equation by the following transformation
1 1
𝑧 = 𝑦1−𝑛 = 𝑦1−2 = 𝑦 2
After the transformation the transformed equation stands,
𝑑𝑧 1 1 1
+ (1 − ) 𝑧 = (1 − ) 𝑥
𝑑𝑥 2 𝑥 2
𝑑𝑧 1 1
𝑜𝑟, + 𝑧 = 𝑥, ……………..(i)
𝑑𝑥 2𝑥 2

which is a linear differential equation in z.


1 1 1
Now, I.F. =𝑒 ∫2𝑥𝑑𝑥 = 𝑒 2 ln 𝑥 = 𝑥 2

Mohammad Abdul Halim, Assistant Professor in Mathematics


Page 99 of 249

1
Now, multiplying the equation (i) by the I.F. 𝑥 2 ,
1 𝑑𝑧 1 1 1 1
𝑥2 + 𝑧𝑥 2 = 𝑥. 𝑥 2
𝑑𝑥1 2𝑥 3 2
𝑑 1
𝑜𝑟, 𝑑𝑥 (𝑧𝑥 2 ) = 2 𝑥 2
Now integrating,
1 1 3
𝑧𝑥 2 = ∫ 𝑥 2 𝑑𝑥
2
3
1
= 2 ∫ 𝑥 2 𝑑𝑥
5
1 𝑥2
= 2 5
+𝑐
2
1 5
1
𝑧𝑥 2 = 5 𝑥 2 + 𝑐
1
1
𝑜𝑟, 𝑧 = 𝑥 2 + 𝑐𝑥 −2
5
1 1
1
∴ 𝑦 2 = 𝑥 2 + 𝑐𝑥 −2
5
Problems for Solution
𝑑𝑦
(i) 𝑑𝑥
− 𝑦 = 𝑥 3 3√𝑦
𝑑𝑦
(ii) 𝑦 − 𝑦2 = 𝑒 𝑥
𝑑𝑥
𝑑𝑦
(iii) + 𝑦 = 𝑦2𝑒 𝑥
𝑑𝑥
𝑑𝑦 𝑦 𝑦2
(iv) 𝑑𝑥
+ 𝑥
= 𝑥2
𝑑𝑦 𝑦 𝑦
(v) 𝑑𝑥
+ 𝑥 ln 𝑦 = 𝑥 2 (ln 𝑦)2
𝑑𝑦
(vi) 𝑥 𝑑𝑥 + 𝑦 = 𝑦 2 ln 𝑥
𝑑𝑦
(vii) (1 − 𝑥 2 ) + 𝑥𝑦 = 𝑥𝑦 2
𝑑𝑥
dy
(viii) + y = y 3 sin x
dx
dy
(ix) + y = y 2e x
dx
dy
(x) + xy = xy 2
dx
dy
(xi) − y = xy 2
dx
dy y
(xii) + =x y
dx x
dy 1
(xiii) + sin 2 y = x 3 cos 2 y
dx x
dy
(xiv) y − y2 = ex
dx
dy y y 2
(xv) 2 − = 2
dx x x
dy
(xvi) cos x + y sin x + 2 y 3 = 0
dx

Mohammad Abdul Halim, Assistant Professor in Mathematics


Page 100 of 249

dy
(xvii) + xy = x 3 y 3
dx
dy
(xviii) + y = y 2e x
dx

Special Type 1:
dy
Method of Solution of the equation f ( y ) + Pf ( y ) = Q where P and Q function of x only or
dx
constant.
Solution:
dy
Equation under solution is f ( y ) + Pf ( y ) = Q (i )
dx
dy dv
Let f ( y ) = v so that f ( y )= then the equation (i) becomes
dx dx
dv
+ Pv = Q
dx
Which is linear equation in v and x be solvable as linear equation.
◊◊Exact differential equation:
A differential equation Mdx + Ndy = 0 where both M and N are function of x and y is said to be exact
when there exist a function F(x,y) such that Mdx + Ndy = dF that is Mdx + Ndy becomes a perfect
differential.
Method of solving exact differential equation:
(i) Integrate M with respect to x regarding y as constant.
(ii) Find out those terms in N which are free from x and integrate them with respect to y .
(iii) Add the two expressions so obtained and equate the sum to an arbitrary constant.
(iv) Finally we obtained the desired solution like as


y as const .
Mdx + 
free from x
N dy = C

► State and prove the necessary and sufficient condition for M dx + N dy = 0 to be exact.
Assume an exact differential equation is M dx + N dy = 0 .
Therefore M dx + N dy = du (i ) where u is a function of x , y .
u u
But we have du ( x , y ) = dx + dy (ii )
x y
Comparing equation (i) and (ii) we get
u u
M= and N =
x y
M 2 u N 2 u
 = and =
 y  y x  x  x y
2 u 2 u M  N
Since = we can write = ;
 y x  x y y x
Which is the necessary condition.

Mohammad Abdul Halim, Assistant Professor in Mathematics


Page 101 of 249

M  N
Conversely if = , then we show that M dx + N dy = 0 is an exact equation.
y x
Let  M dx =u



( M dx )=  u
x x
u
i.e. M =
x
M 2u
=
y  y x
N   u 
=  
x x  y
  u 
 N −  =0
x  y 
Integrating with respect to x regarding y as constant
u
N− = f ( y ) Where f ( y ) is a function of y only.
y
u
N= + f (y)
y
u  u 
Now, M dx + N dy = dx +  + f ( y ) dy
x  y 
u u
= dx + dy + f ( y ) dy
x y
= du + d  f ( y ) dy
= d  u +  f ( y ) dy 
Which shows that M dx + N dy = 0 is an exact equation.
Mathematical Problem based on exact differential equation
Problem 01: Solve (𝒙𝟐 − 𝟒𝒙𝒚 − 𝟐𝒚𝟐 )𝒅𝒙 + (𝒚𝟐 − 𝟒𝒙𝒚 − 𝟐𝒙𝟐 )𝒅𝒚 = 𝟎
Solution: Here, 𝑀 = 𝑥 2 − 4𝑥𝑦 − 2𝑦 2 and 𝑁 = 𝑦 2 − 4𝑥𝑦 − 2𝑥 2
𝜕𝑀 𝜕𝑁
∴ 𝜕𝑦 = −4𝑥 − 4𝑦 and 𝜕𝑥 = −4𝑦 − 4𝑥 = −4𝑥 − 4𝑦
𝜕𝑀 𝜕𝑁
Since = so, the equation is exact.
𝜕𝑦 𝜕𝑥
Therefore, the general solution is
 M dx +
y Cons tan t
 (terms of N not containing x ) dy = C
∫(𝑥 2 − 4𝑥𝑦 − 2𝑦 2 )𝑑𝑥 + ∫ 𝑦 2 𝑑𝑦 = 𝐶
𝑥3 𝑥2 𝑦3
𝑜𝑟, 3
− 4𝑦 2
− 2𝑦 2 𝑥 + 3
= 𝐶
𝑥 3 𝑦 3
2
𝑜𝑟, 3 − 2𝑥 𝑦 − 2𝑥𝑦 2 + 3 =
𝐶
3 2 2 3
𝑜𝑟, 𝑥 − 6𝑥 𝑦 − 6𝑥𝑦 + 𝑦 = 3𝐶
∴ 𝑥 3 + 𝑦 3 − 6𝑥𝑦(𝑥 + 𝑦) = 𝐶 ′

Mohammad Abdul Halim, Assistant Professor in Mathematics


Page 102 of 249

Problem 02: Solve (𝒙 + 𝟐𝒙𝒚𝟑 )𝒅𝒙 + (𝟏 + 𝟑𝒙𝟐 𝒚𝟐 )𝒅𝒚 = 𝟎


Solution: Here, 𝑀 = 𝑥 + 2𝑥𝑦 3 and 𝑁 = 1 + 3𝑥 2 𝑦 2
𝜕𝑀 𝜕𝑁
∴ 𝜕𝑦 = 6𝑥𝑦 2 and 𝜕𝑥 = 6𝑥𝑦 2
𝜕𝑀 𝜕𝑁
Since 𝜕𝑦
= 𝜕𝑥
so, the equation is exact.
Now, ∫ 𝑀𝑑𝑥 = ∫(𝑥 + 2𝑥𝑦 3 )𝑑𝑥 [ y is constant]
𝑥2 𝑥2
= 2
+ 2𝑦 3 2
𝑥2 2 3
= +𝑥 𝑦 2
And ∫(𝑇𝑒𝑟𝑚𝑠 𝑖𝑛 𝑁 𝑖𝑛𝑑𝑒𝑝𝑒𝑛𝑑𝑒𝑛𝑡 𝑜𝑓 𝑥)𝑑𝑦 = ∫ 1𝑑𝑦
=𝑦
Therefore, the required solution is
𝑥2
+ 𝑥2𝑦3 + 𝑦 = 𝐶
2
Problem 03: Solve (𝟑𝒙𝟐 𝒚𝟐 + 𝟐𝒙𝒚𝟑 + 𝒙𝒄𝒐𝒔𝒙)𝒅𝒙 + (𝟐𝒙𝟑 𝒚 + 𝟑𝒙𝟐 𝒚𝟐 − 𝐥𝐧 𝒚)𝒅𝒚 = 𝟎
Solution: Here, 𝑀 = 3𝑥 2 𝑦 2 + 2𝑥𝑦 3 + 𝑥𝑐𝑜𝑠𝑥
And 𝑁 = 2𝑥 3 𝑦 + 3𝑥 2 𝑦 2 − ln 𝑦
𝜕𝑀 𝜕𝑁
∴ 𝜕𝑦 = 6𝑥 2 𝑦 + 6𝑥𝑦 2 and 𝜕𝑥 = 6𝑥 2 𝑦 + 6𝑥𝑦 2
𝜕𝑀 𝜕𝑁
Since = so, the equation is exact.
𝜕𝑦 𝜕𝑥
Now, ∫ 𝑀𝑑𝑥 = ∫(3𝑥 2 𝑦 2 + 2𝑥𝑦 3 + 𝑥𝑐𝑜𝑠𝑥)𝑑𝑥 [y as constant]
𝑥3 𝑥2 𝑑𝑥
= 3𝑦 2 3
+ 2𝑦 3
+ [𝑥 ∫ cos 𝑥𝑑𝑥 − ∫ (𝑑𝑥 ∫ 𝑐𝑜𝑠𝑥𝑑𝑥) 𝑑𝑥]
2
3 2 2 3
= 𝑥 𝑦 + 𝑥 𝑦 + [𝑥𝑠𝑖𝑛 𝑥 − ∫ sin 𝑥𝑑𝑥]
= 𝑥 3 𝑦 2 + 𝑥 2 𝑦 3 + 𝑥𝑠𝑖𝑛 𝑥 + cos 𝑥
And ∫(𝑇𝑒𝑟𝑚𝑠 𝑖𝑛 𝑁 𝑖𝑛𝑑𝑒𝑝𝑒𝑛𝑑𝑒𝑛𝑡 𝑜𝑓 𝑥)𝑑𝑦 = ∫ − ln 𝑦𝑑𝑦
= −(𝑦𝑙𝑛 𝑦 − 𝑦)
= 𝑦 − 𝑦𝑙𝑛 𝑦
Therefore, the required solution is
𝑥 3 𝑦 2 + 𝑥 2 𝑦 3 + 𝑥𝑠𝑖𝑛 𝑥 + cos 𝑥 + 𝑦 − 𝑦𝑙𝑛 𝑦 = 𝐶
Problem 04: Solve (𝟒𝒙𝒚 − 𝟑𝒚𝟐 + 𝒙𝒆−𝟐𝒙 )𝒅𝒙 + (𝟐𝒙𝟐 − 𝟔𝒙𝒚 − 𝐥𝐧 𝒚)𝒅𝒚 = 𝟎
Solution: Here, 𝑀 = 4𝑥𝑦 − 3𝑦 2 + 𝑥𝑒 −2𝑥 and 𝑁 = 2𝑥 2 − 6𝑥𝑦 − ln 𝑦
𝜕𝑀 𝜕𝑁
∴ 𝜕𝑦 = 4𝑥 − 6𝑦 and 𝜕𝑥 = 4𝑥 − 6𝑦
𝜕𝑀 𝜕𝑁
Since = so, the equation is exact.
𝜕𝑦 𝜕𝑥
Now, ∫ 𝑀𝑑𝑥 = ∫(4𝑥𝑦 − 3𝑦 2 + 𝑥𝑒 −2𝑥 )𝑑𝑥 [y as constant]
𝑥2 𝑑𝑥
= 4𝑦 2
− 3𝑦 2 𝑥 + [𝑥 ∫ 𝑒 −2𝑥 𝑑𝑥 − ∫ (𝑑𝑥 ∫ 𝑒 −2𝑥 𝑑𝑥) 𝑑𝑥]
𝑒 −2𝑥 𝑒 −2𝑥
= 2𝑥 𝑦 − 3𝑥𝑦 2 + [𝑥
2
−2
− ∫ −2 𝑑𝑥]
1 1
= 2𝑥 2 𝑦 − 3𝑥𝑦 2 − 2
𝑥𝑒 −2𝑥 + 2 ∫ 𝑒 −2𝑥 𝑑𝑥
1 1 𝑒 −2𝑥
= 2𝑥 2 𝑦 − 3𝑥𝑦 2 − 2
𝑥𝑒 −2𝑥
+ 2 −2
1 1 −2𝑥
= 2𝑥 2 𝑦 − 3𝑥𝑦 2 − 2
𝑥𝑒 −2𝑥
− 4
𝑒
And ∫(𝑇𝑒𝑟𝑚𝑠 𝑖𝑛 𝑁 𝑖𝑛𝑑𝑒𝑝𝑒𝑛𝑑𝑒𝑛𝑡 𝑜𝑓 𝑥)𝑑𝑦 = ∫ − ln 𝑦𝑑𝑦
= −(𝑦𝑙𝑛 𝑦 − 𝑦)
= 𝑦 − 𝑦𝑙𝑛 𝑦
Therefore, the required solution is

Mohammad Abdul Halim, Assistant Professor in Mathematics


Page 103 of 249

1 1
2𝑥 2 𝑦 − 3𝑥𝑦 2 − 𝑥𝑒 −2𝑥 − 𝑒 −2𝑥 + 𝑦 − 𝑦 ln 𝑦 = 𝐶
2 4
 x  x  x
Problem 05: Solve: 1 + e
y
 dx + e y
1 −  dy = 0 .
   y
x
Here, M = 1 + e y

 x x
N= e 1 −  y

 y
M x  x x x N x  1   x  xy  1 
=e y   = − 2 e y
and =e y
 −  + 1 −  e  
y y y y x  y  y  y
x  1 1 x 
= e y  − + − 2 
 y y y 
x x
=− 2 e y
y
M N
 = the given equation is exact.
y x
Therefore, the general solution is
 M dx +  (terms of N not containing x ) dy = C
y Cons tant

 x 
 1 + e  dx +  0 dy = c
y

 
x
e y
x+ =c
1
y
( )
Problem 06: Solve: x 2 − 2 xy + 3 y 2 dx + 4 y 3 + 6 xy − x 2 dy = 0 . ( )
Here, M = x 2 − 2 xy + 3 y 2
N = 4 y 3 + 6 xy − x 2
M N
= − 2 x + 6 y and = 6 y − 2x
y x
M N
 = the given equation is exact.
y x
Therefore, the general solution is
 M dx +  (terms of N not containing x ) dy = C
y Cons tant

 (x − 2xy + 3 y ) dx +  4 y dy = c
2 2 3

x3
− x 2 y + 3 xy 2 + y 4 = c .
3
Problem-07: Solve ( y 4 + 4 x 3 y + 3 x ) dx + ( x 4 + 4 xy 3 + y + 1) dy = 0 .

Mohammad Abdul Halim, Assistant Professor in Mathematics


Page 104 of 249

Solution: Given that,


(y 4
+ 4 x 3 y + 3 x ) dx + ( x 4 + 4 xy 3 + y + 1) dy = 0 … … … (1)
where, M = y 4 + 4 x3 y + 3x and N = x 4 + 4 xy 3 + y + 1
M
 = 4 y 3 + 4 x3
y
N
= 4 x3 + 4 y 3
x
M N
since, = so the equation (1) is an exact differential equation.
y x
Integrating M with respect to x we get
3 2
xy 4 + x 4 y + x
2
In N, terms free from x are y + 1 whose integral with respect to y is
1 2
y +y
2
Therefore the general solution is
3 2 1 2
xy 4 + x 4 y +
x + y + y=c.
2 2
Problem-08: Solve ( x − 2 xy + 3 y 2 ) dx + ( 4 y 3 + 6 xy − x 2 ) dy = 0 .
2

Solution: Given that,


(x 2
− 2 xy + 3 y 2 ) dx + ( 4 y 3 + 6 xy − x 2 ) dy = 0 … … … (1)
where, M = x 2 − 2 xy + 3 y 2 and N = 4 y 3 + 6 xy − x 2
M
 = −2 x + 6 y
y
N
= 6 y − 2x
x
M N
since, = so the equation (1) is an exact differential equation.
y x
Integrating M with respect to x we get
1 3
x − x 2 y + 3 xy 2
3
In N, terms free from x is 4 y 3 whose integral with respect to y is
y4
Therefore the general solution is
1 3
x − x 2 y + 3 xy 2 + y 4 = c .
3
Problem-09: Solve ( 2 x 3 + 3 y ) dx + ( 3 x + y − 1) dy = 0 .
Solution: Given that,
( 2x 3
+ 3 y ) dx + ( 3 x + y − 1) dy = 0 … … … (1)

Mohammad Abdul Halim, Assistant Professor in Mathematics


Page 105 of 249

where, M = 2 x3 + 3 y and N = 3 x + y − 1
M
 =3
y
N
=3
x
M N
since, = so the equation (1) is an exact differential equation.
y x
Integrating M with respect to x we get
1 4
x + 3 xy
2
In N, terms free from x are y − 1 whose integral with respect to y is
1 2
y −y
2
Therefore the general solution is
1 4 1
x + 3 xy + y 2 − y = c .
2 2
Problem – 10: ( ) (
Solve: x 2 − 2 xy + 3 y 2 dx + 4 y 3 + 6 xy − x 2 dy = 0 .)
Solution:
Here, M = x 2 − 2 xy + 3 y 2
N = 4 y 3 + 6 xy − x 2
M N
= − 2 x + 6 y and = 6 y − 2x
y x
M N
 = the given equation is exact.
y x

Therefore, the general solution is


 M dx +  (terms of N not containing x ) dy = C
y Cons tant

 (x − 2xy + 3 y ) dx +  4 y dy = c
2 2 3

x3
− x 2 y + 3 xy 2 + y 4 = c .
3
 x  x  x
Problem – 11: Solve 1 + e
y
 dx + e y 1 −  dy = 0 .
   y
Solution:
x
Here, M = 1 + e y

 x
x
N= e 1 − 
y

 y
M x  x x x N x  1  x x 1
=e y   = − 2 e y
and =e y
 −  + 1 −  e y
 
y y y y x  y  y  y

Mohammad Abdul Halim, Assistant Professor in Mathematics


Page 106 of 249

x  1 1 x 
=e y − + − 
 y y y2 
 
x xy
=−
e
y2
M N
 = the given equation is exact.
y x
Therefore, the general solution is
 M dx +  (terms of N not containing x ) dy = C
y Cons tant

 x 
 1 + e  dx +  0 dy = c
y

 
x
e y
x+ =c
1
y
Problems for Solution
(i) 𝑥𝑑𝑦 + (𝑦 + 𝑥𝑠𝑖𝑛2𝑥)𝑑𝑥 = 0
(ii) 2𝑥𝑠𝑖𝑛 𝑦𝑑𝑥 + 𝑥 2 cos 𝑦𝑑𝑦 = 0
(iii) (𝑦 4 + 4𝑥 3 𝑦 + 3𝑥)𝑑𝑥 + (𝑥 4 + 4𝑥𝑦 3 + 𝑦 + 1)𝑑𝑦 = 0
(iv) (𝑥 − 2𝑒 −2𝑦 )𝑑𝑦 + (𝑦 + 𝑥𝑠𝑖𝑛 2𝑥)𝑑𝑥 = 0
(v) (x 2
) ( )
− 4 xy − 2 y 2 dx + y 2 − 4 xy + 2 x 2 dy = 0
(vi) (x 3
+ 3xy 2 ) dx + ( y + 3x y ) dy = 0
3 2

(vii) ( 2 y − x −1) dy + ( 2x − y + 1) dx = 0
(viii) ( 2x + 3 y − 6) dy − ( 6x − 2 y − 7 ) dx = 0
(ix) ( e + 1) cos xdx + e sin xdy = 0
y y

(x) ( 2 x + 3 y ) dx + ( 3x + y − 1) dy = 0
3

(xi) ( x − 2xy − y ) dx − ( x + y ) dy = 0
2 2 2

(xii) ( 3x y − 6 x ) dx − ( x + 2 y ) dy = 0
2 3

(xiii) ( 3x + 4 xy ) dx + ( 2 x + 2 y ) dy = 0
2 2

(xiv) ( x − 2e ) dy + ( y + x sin x ) dx = 0
y

(xv) cos x tan y + cos ( x + y )  dx + sin x sec 2 y + cos ( x + y )  dy = 0


(xvi) (x 2
− 4 xy − 2 y 2 ) dx + ( y 2 − 4 xy − 2 x 2 ) dy = 0
◊◊Equation reducible to exact differential equation
A differential equation M ( x, y ) dx + N ( x, y ) dy = 0 is not an exact differential equation if
M N
 .
y x

Mohammad Abdul Halim, Assistant Professor in Mathematics


Page 107 of 249

But it can be reduced to an exact differential equation by multiplying a function of x and y, which is called
an integrating factor.
Rules for finding integrating factor: Let the differential equation is,
M ( x, y ) dx + N ( x, y ) dy = 0 ... … … (1)
M N

y x
1. If = f ( x ) then the integrating factor is  = e f ( x)dx .
N
M N

y x
2. If = g ( y ) then the integrating factor is  = e g ( y )dy .
M
3. If M and N are both homogeneous function in x, y of degree n, then the integrating factor is
1
= ; where, Mx + Ny  0 .
Mx + Ny
4. If the equation (1) is of the form, yf ( xy ) dx + xg ( xy ) dy = 0 then the integrating factor is
1
= ; where, Mx − Ny  0
Mx − Ny
M y
NOTE: 1. If Mx + Ny = 0 , then = − and the equation reduces to ydx − xdy = 0 ,
N x
which can be easily solved.
M y
2. If Mx − Ny = 0 , then = and the equation reduces to ydx + xdy = 0 ,
N x
which can be easily solved.
Mathematical problem based on reducible exact differential equation:
( )
Problem-01: Solve x 2 + y 2 + x dx + xydy = 0 .
Solution: Given that,
(x 2
+ y 2 + x ) dx + xydy = 0 … … … (1)
where, M = x 2 + y 2 + x and N = xy
M
 = 2y
y
N
=y
x
M N
since,  so the equation (1) is not an exact differential equation.
y x
M N

y x 2 y − y 1
However, = =
N xy x
1
= e x
dx
Hence, I.F
= e ln x
=x

Mohammad Abdul Halim, Assistant Professor in Mathematics


Page 108 of 249

Multiplying by I.F, the equation (1) becomes,


( )
x 3 + xy 2 + x 2 dx + x 2 ydy = 0 … … … (2)
which is exact now.
Let, M ' = x3 + xy 2 + x 2 and N ' = x 2 y
Integrating M ' with respect to x we get
1 4 1 2 2 1 3
x + x y + x
4 2 3
In N ' , there is no term free from x.
Therefore the general solution is
1 4 1 2 2 1 3
x + x y + x =c.
4 2 3
Problem-02: Solve ( x + y 2 + 2 x ) dx + 2 ydy = 0 .
2

Solution: Given that,


(x 2
+ y 2 + 2 x ) dx + 2 ydy = 0 … … … (1)
where, M = x 2 + y 2 + 2 x and N = 2 y
M
 = 2y
y
N
=0
x
M N
since,  so the equation (1) is not an exact differential equation.
y x
M N

y x 2 y − y
However, = =1
N xy
Hence, I.F = e 
dx

= ex
Multiplying by I.F, the equation (1) becomes,
( e x x 2 + e x y 2 + 2 xe x ) dx + xye x dy = 0 … … … (2)
which is exact now.
Let, M ' = e x x 2 + e x y 2 + 2 xe x and N ' = x 2 ye x
Integrating M ' with respect to x we get
ex x2 + ex y 2
In N ' , there is no term free from x.
Therefore the general solution is
ex x2 + ex y 2 = c .
 1 3 1 2
y + x  dx + ( x + xy 2 ) dy = 0 .
1
Problem-03: Solve  y +
 3 2  4
Solution: Given that,

Mohammad Abdul Halim, Assistant Professor in Mathematics


Page 109 of 249

 1 3 1 2
 y + y + x  dx + ( x + xy ) dy = 0 … … … (1)
1 2

 3 2  4
where, M = y + y 3 + x 2 and N = ( x + xy 2 )
1 1 1
3 2 4
M
 = 1+ y2
y
N 1
= (1 + y 2 )
x 4
M N
since,  so the equation (1) is not an exact differential equation.
y x
M N

y x
1
1+ y2 − 1+ y2
4
( )
However, =
N 1
4
(
x + xy 2 )
1
(
4 + 4 y2 −1 − y2 )
=4
1
4
(
x 1+ y2 )
=
(
3 + 3y2 )
( )
x 1+ y2
3 (1 + y ) 2

=
x (1 + y ) 2

3
=
x
3
= e x
dx
Hence, I.F
= e3ln x
= eln x
3

= x3
Multiplying by I.F, the equation (1) becomes,
 3 1 3 3 1 5
 x y + x y + x  dx + ( x + x y ) dy = 0 … … … (2)
1 4 4 2

 3 2  4
which is exact now.
x y + x and N ' = ( x 4 + x 4 y 2 )
1 3 3 1 5 1
Let, M ' = x 3 y +
3 2 4
Integrating M ' with respect to x we get
1 4 1 1
x y + x4 y3 + x6
4 12 12
In N ' , there is no term free from x.
Therefore the general solution is

Mohammad Abdul Halim, Assistant Professor in Mathematics


Page 110 of 249

1 4 1 1
x y + x4 y3 + x6 = c .
4 12 12
Problem-04: Solve ( x + y 4 ) dx − xy 3dy = 0 .
4

Solution: Given that,


(x 4
+ y 4 ) dx − xy 3dy = 0 … … … (1)
where, M = x 4 + y 4 and N = − xy 3
M
 = 4 y3
y
N
= − y3
x
M N
since,  so the equation (1) is not an exact differential equation.
y x
But the equation (1) is a homogeneous differential equation.
1
Hence, I.F =
Mx + Ny
1
=
x + xy 4 − xy 4
5

1
= 5
x
Multiplying by I.F, the equation (1) becomes,
 1 y4  y3
 + 5 
dx − dy = 0 … … … (2)
x x  x4
which is exact now.
1 y4 y3
Let, M ' = + 5 and N ' = − 4
x x x
'
Integrating M with respect to x we get
y4
ln x − 4
4x
'
In N , there is no term free from x.
Therefore the general solution is
y4
ln x − = c.
4 x4
Problem-05: Solve y ( xy + 2 x 2 y 2 ) dx + x ( xy − x 2 y 2 ) dy = 0 .
Solution: Given that,
y ( xy + 2 x 2 y 2 ) dx + x ( xy − x 2 y 2 ) dy = 0 … … … (1)
( ) (
where, M = y xy + 2 x 2 y 2 and N = x xy − x 2 y 2 )
M
 = 2 xy + 6 x 2 y 2
y

Mohammad Abdul Halim, Assistant Professor in Mathematics


Page 111 of 249

N
= 2 xy − 6 x 2 y 2
x
M N
since,  so the equation (1) is not an exact differential equation.
y x
But the equation (1) is of the form, yf ( xy ) dx + xg ( xy ) dy = 0 .
1
Hence, I.F =
Mx − Ny
1
=
x y + 2 x y − x 2 y 2 + x3 y 3
2 2 3 3

1
= 3 3
3x y

Multiplying by I.F, the equation (1) becomes,


 1 2  1 1 
 2 +  dx +  2
−  dy = 0 … … … (2)
 3x y 3x   3xy 3 y 
which is exact now.
1 2 1 1
Let, M ' = 2
+ and N ' = 2

3x y 3x 3xy 3 y
'
Integrating M with respect to x we get
1 2
− + ln x
3 xy 3
1
In N ' , term free from x is − , whose integral with respect to y is,
3y
1
− ln y
3
Therefore the general solution is
1 2 1
− + ln x − ln y = ln c
3xy 3 3
1
 − + ln x 2 − ln y = 3ln c
xy
1 x2
− + ln = ln c3
xy y
1
x 2 − xy
 e = c3
y
1
x2
 = Ce xy ; where, C = c 3 .
y

Mohammad Abdul Halim, Assistant Professor in Mathematics


Page 112 of 249

f ( x ) dx M N 
1. Show that e 
1
is an integrating factor of Mdx + N dy = 0 if  −  is a function of
N  y x 
x only.

Let P is an integrating factor of Mdx + N dy = 0 then


PMdx + PN dy = 0 is an exact equation.



(PM ) =  (P N )
y x
M P N P
Or, P +M = P +N
y y x x
 M N  P P
Or, P  −  = N −M
 y x  x y
 M N  P P
Or, P  −  = N [ Since P is a function of x only , = 0 .]
  y  x   x  y
 M N  dP
Or, P  −  = N
 y x  dx
1  M N  dP
Or,  −  dx =
N  y x  P
 M N 
 = f (x ) , is a function of x only, then
1
 −
 x 
Now
N  y
= f ( x ) dx
dP
P
log P =  f (x ) dx
P = e  f ( x ) dx
1 M N 
i.e. e  f ( x ) dx is an integrating factor of Mdx + N dy = 0 if  −  is a function of x only.
N   y  x 
− g ( y ) dy 1 M N 
2. Show that e  is an integrating factor of Mdx + N dy = 0 if  −  is a function
M   y  x 
of y only.
Let P is an integrating factor of Mdx + N dy = 0 then
PMdx + PN dy = 0 is an exact equation.



(PM ) =  (P N )
y x
M P N P
Or, P +M = P +N
y y x x

Mohammad Abdul Halim, Assistant Professor in Mathematics


Page 113 of 249

 M N  P P
Or, P  −  = N −M
 y x  x y
 M N  P P
P  −  = −M [ Since P is a function of y only , = 0 .]
 x 
Or,
 y y x
 M N  dP
P  −  = −M
 x 
Or,
 y dy
 M N 
1 dP
Or,  −  dy = −
 y
M x  P
 M N 
 = f ( y ) , is a function of y only, then
1
 −
 x 
Now
M  y

= − f ( y ) dy
dP

P
 log P = −  f ( y ) dy
 P = e −  f ( y ) dy
− g ( y ) dy 1 M N 
i.e. e  is an integrating factor of Mdx + N dy = 0 if  −  is a function of y only.
M  y x 

3. If M (x, y ) dx + N (x, y ) dy = 0 is homogeneous and Mx + Ny  0 then


1
is an integrating
M x+ N y
factor.
4. If the equation can be written in the form y f (xy) dx + x g (xy) dy = 0 , f (xy)  g (xy) then
1 1
= is an integrating factor.
xy  f ( xy) − g ( xy) Mx − Ny
( )
Problem – 06: Is the differential equation x 2 + y 2 + 2 x dx + 2 y dy = 0 exact? If not, then make
the differential equation exact and hence solve the equation.
Solution:
Here, M = x 2 + y 2 + 2 x
N =2y
M N
= 2 y and =0
y x
M N
  the given equation is not exact.
y x
1   M  N  2y
However,  −  = =1
N   y  x  2y
Integrating Factor = e 
1 dx
= ex
Multiplying by e x the given equation becomes,
( )
e x x 2 + y 2 + 2 x dx + 2 y e x dy = 0 , which is exact.

Mohammad Abdul Halim, Assistant Professor in Mathematics


Page 114 of 249

Therefore, the general solution is


 e (x + y + 2x)dx +  0 dy = c
x 2 2

( )
  e x x 2 + 2 x dx + y 2  e x dx = c
(
 e x x 2 + y2 = c )
Problem – 07: Is the differential equation (1 + xy) y dx + (1− xy) x dy = 0 exact? If not, then make
the differential equation exact and hence solve the equation.
Solution:
Here, M = y + xy 2
N = x − x2 y
M N
= 1+ 2 xy and = 1 − 2 xy
y x
M N
  the given equation is not exact. But it is of the form y f (xy) dx + x g (xy) dy = 0
y x
Now, Mx − Ny = xy + x 2 y 2 − xy + x 2 y 2 = 2 x 2 y 2  0
1 1
Integrating Factor = = 2 2
Mx − Ny 2 x y
1
Multiplying by the given equation becomes,
2x2 y2
1
2 2
(1 + xy) y dx + 12 2 (1 − xy) x dy = 0
2x y 2x y
 1 1   1 1 
  2 +  dx +  2 −  dy = 0 , which is exact.
x y x   xy y 
Therefore, the general solution is
 1  1  1 
  2  dx +   −
 +  dy = c
x y  x  y 
1 1 1 1
  2 dx +  dx −  dy = c
y x x y
1
− + log x − log y = c
xy
Problem – 08: Calculate the integrating factor of
(xy sin xy + cos xy) y dx + (xy sin xy − cos xy) x dy = 0 and hence solve it.
Solution:
The given equation is of the form y f (xy) dx + x g (xy) dy = 0
1 1 1
Integrating Factor = = 2 2 =
Mx − Ny x y Sinxy + xy Cos xy − x y Sinxy + xyCosxy 2 xyCosxy
2 2

1
Multiplying by the given equation becomes,
2 xyCosxy

Mohammad Abdul Halim, Assistant Professor in Mathematics


Page 115 of 249

1
(xy sin xy + cos xy) y dx + 1 (xy sin xy − cos xy) x dy = 0
2 xyCosxy 2 xyCosxy
1 1 1 1
  y tan xy +  dx +  x tan xy −  dy = 0 , which is exact.
2 x 2 y
Therefore, the general solution is
1  1 1  1
 2  y tan xy + x  dx + 2   − y  dy = c 
   
y  log Cos xy  1 1
  −  + log x − log y = c 
2 y  2 2
 log x = c + log y + log (Cosxy)
 log x = c + log ( y Cosxy)
 x = e c e log ( y Cosxy )
 x = c y Cos xy
(
Problem – 09: Solve: y 2 dx + x 2 − xy − y 2 dy = 0 . )
Solution:
The given equation is homogeneous and
Mx + Ny = xy 2 + x 2 y − xy 2 − y 3 = y x 2 − y 2  0 ( )
1 1
Integrating Factor = =
Mx + Ny y x − y 22
( )
1
Multiplying by
(
y x − y2
2
)
the given equation becomes,

y x 2 − y 2 − xy
+ dy = 0
x2 − y2
dx
(
y x2 − y2 )
1 1 1  1 xy 
  −  dx +  −  dy = 0
2 x − y x+ y   y y x 2
− y 2 
 ( )
1 1 1  1 x 
  −  dx +  − 2  dy = 0 , which is exact.
2 
2 x − y x+ y  y x −y 
Therefore, the general solution is
1 1 1   1
 2  x − y − x + y  dx +   y  dy = log c
   
1 x− y 
 log   + log y = log c
2 x+ y 
 (x − y ) y 2 = (x + y ) c
Exercise:
1. ( )
x 2 + y 2 + 1 dx − 2 xydy = 0
2. (y 4
+ 2 y ) dx + ( xy 3 + 2 y 4 − 4 x ) dy = 0

Mohammad Abdul Halim, Assistant Professor in Mathematics


Page 116 of 249

3. y 2 dx + ( x 2 − xy − y 2 ) dy = 0
4. y ( x 2 y 2 + 2 ) dx + x ( 2 − 2 x 2 y 2 ) dy = 0
5. ( x y − 2 xy ) dx − ( x − 3x y ) dy = 0
2 2 3 2

6. y dx − ( x − xy − y ) dy = 0
2 2 2

7. y(1 + xy)dx + x (1 − xy ) dy = 0
8. ( x 2 + y 2 + x)dx + xydy = 0
9. (
( y 4 + 2 y )dx + xy 3 + 2 y 4 − 4 x dy = 0 )
10. (3 x y + 2 xy )dx + 2 x y
2 4 3 3
( − x ) dy = 02

(
11. (2 xy + y )dx + 2 y − x dy = 0
2 3
)
12. ( xy
3
+ y )dx + 2 ( xy 2
+ x + y 4 dy = 0 )
13. ( x + y + 2 x)dx + 2 ydy = 0
2 2

3 4 3
( )
14. ( x + xy )dx + 2 y dy = 0

15. ( y
4
+ 2 y )dx + ( xy + 2 y 3 4
− 4 x dy = 0)
16. (12 y + 4 y + 6 x )dx + 3 x + xy dy = 0
3 2 2
( )
17. y ln ydx + ( x − ln y ) dy = 0
18. y ( xy + 2 x y )dx + x xy − x y dy = 0
2 2 2 2
( )
(
19. x − 2 y3
) dx + 2 xy dy = 0
2

20. (y + 2 y ) dx + (xy + 2 y − 4 x ) dy = 0
4 3 4

21. (x + y ) dx − xy dy = 0
4 4 3

22. (y − 2 x y ) dx + (x − 2 xy ) dy = 0
4 3 4 3

23. y (2 xy + 1) dx + x (1 + 2 xy − x y ) dy = 0 3 3

A pplication of first order and first-degree ODE

In this chapter we pointed out that differential equations originate from the mathematical
formulation from a physical problem and then we have an attempt to interpret the solution
in terms of the quantities involved in the original problem.

Mohammad Abdul Halim, Assistant Professor in Mathematics


Page 117 of 249

Worked Out Problems

Problem-01: The population of bacteria in a culture grows at a rate that is proportional to the number at
present. Initially, it has p0 number of bacteria, and after 1 hr the number of bacteria is measured to be
3
p0 .
2
(a). What is the number of bacteria after t hr?
(b). Determine the necessary time for number of bacteria to triple.
Solution: Let, p (t ) be the number of bacteria at any time t hr.Since, the rate of growth of bacteria is
proportional to the number of bacteria p (t ) ,
dp dp
So  p  = kp (1) where, k is a proportional constant.
dt dt
According to the question we have,
3
p ( 0 ) = p0 & p (1) = p0 (2)
2
Now from Eq. (1) we can write,
dp dp
= kdt  = k  dt  ln p = kt + ln c  ln p = ln ekt + ln c  ln p = ln cekt
p p
 p = cekt (3)
From Eq. (2), we have
p0 = ce k .0  c = p0
From Eq. above we have,
3
p0 = p0 e k
2
3
or , e k =
2
3
or , k = ln  
2
 k = 0.4055
Using the values of c & k in Eq.(4) we have,
p = p0 e0.4055t (4)
This is the number of bacteria after t hr.
Again, let after t = t1 hr the number of bacteria will be triple. i.e, p ( t1 ) = 3 p0 .
Now from Eq. (4) we have,
3 p0 = p0 e0.4055t1
or , e0.4055t1 = 3
or , 0.4055t1 = ln 3
ln 3
or , t1 =
0.4055
or , t1 = 2.71 hr
This is the required time. (ans.)

Mohammad Abdul Halim, Assistant Professor in Mathematics


Page 118 of 249

Problem-02: The population of bacteria in a culture grows at a rate that is proportional to the number at
present. Initially, there are 600 bacteria, and after 3 hr there are 10,000 bacteria.
(a). What is the number of bacteria after t hr?
(b). What is the number of bacteria after 5 hr?
(c). When will the number of bacteria reach 24,000?
Solution: Let, p (t ) be the number of bacteria at any time t hr.
Since, the rate of growth of bacteria is proportional to the number of bacteria p (t ) ,
dp
So  p
dt
dp
or , = kp ... ... ...(1)
dt
where, k is a proportional constant.
According to the question we have,
p ( 0) = 600 ... ... ... (2)
p ( 3) = 10,000 ... ... ... (3)
Now from Eq. (1) we can write,
dp
= kdt
p
dp
or ,  = k  dt
p
or , ln p = kt + ln c
or , ln p = ln ekt + ln c
or , ln p = ln cekt
 p = cekt ... ... ...(4)
From Eq. (2) & Eq.(4) we have,
600 = cek .0
 c = 600
From Eq. (3) & Eq.(4) we have,
10, 000 = 600e3k
10, 000
or , e3k =
600
or , e =16.667
3k

or, 3k = ln (16.667 )
1
or , k = ln (16.667 )
3
 k = 0.9378
Using the values of c & k in Eq.(4) we have,
p = 600e0.9378t ... ... ...(5)
This is the number of bacteria after t hr.
Again, For t = 5 we get from Eq. (5),
p = 600e0.93785

Mohammad Abdul Halim, Assistant Professor in Mathematics


Page 119 of 249

 p = 65246.63
This is the number of bacteria after 5 hr.
Again, let after t = t1 hr the number of bacteria will be 24,000. i.e, p ( t1 ) = 24,000 .
Now from Eq. (4) we have,
24, 000 = 600e0.9378t1
or , e0.9378t1 = 40
or, 0.9378t1 = ln ( 40 )
ln ( 40 )
or , t1 =
0.9378
or , t1 = 3.93 hr
This is the required time. (ans.)
Problem-03: Radioactive substances decay at a rate that is proportional to the amount present. The half-
life of a substance is the time required for a given amount to be reduced by one-half. The half-life of cesium-
137 is 30 years. Suppose we have 100 mg sample.
(a). Find the mass that remains after t years.
(b). How much of the sample remains after 100 years?
(c). After how long will only 1 mg remain?
Solution: Let, p (t ) be the amount of cesium-137 present in the sample at any time t hr.
Since, the rate of decay of cesium-137 is proportional to the amount p (t ) ,
dp
So  p
dt
dp
or , = kp ... ... ...(1)
dt
where, k is a proportional constant.
According to the question we have,
p ( 0) = 100 ... ... ... (2)
p ( 30) = 50 ... ... ... (3)
Now from Eq. (1) we can write,
dp
= kdt
p
dp
or ,  = k  dt
p
or , ln p = kt + ln c
or , ln p = ln ekt + ln c
or , ln p = ln cekt
 p = cekt ... ... ...(4)
From Eq. (2) & Eq.(4) we have,
100 = cek .0
 c = 100
From Eq. (3) & Eq.(4) we have,
50 = 100e30 k

Mohammad Abdul Halim, Assistant Professor in Mathematics


Page 120 of 249

1
or , e30 k =
2
or , 30k = ln ( 0.5)
1
or , k = ln ( 0.5 )
30
 k = −0.0231
Using the values of c & k in Eq.(4) we have,
p = 100e−0.0231t ... ... ...(5)
This is the number of bacteria after t hr.
Again, For t = 100 years, we get from Eq. (5),
p = 100e−0.0231100
 p = 9.926mg
This amount of sample will remain after 100 years.
Again, let after t = t1 years the amount of sample will remain 1mg. i.e, p ( t1 ) = 1 .
Now from Eq. (4) we have,
1 = 100e−0.0231t1
1
or , e −0.0231t1 =
40
or, − 0.0231T = ln ( 0.025)
ln ( 0.025)
or , t1 = −
0.0231
or , t1 = 159.7 years
This is the required time. (ans.)
Problem-04: When a cake is removed from an oven, it’s temperature is measured at 3000 F . Three
minutes later it’s temperature is 2000 F . How long will it take for the cake to cool off to a room
temperature of 700 F ?
(a). Give a relation that gives the temperature of the cake after t mins.
(b). How long will it take for the cake to cool off to 750 F ?
Solution: Let, T (t ) be the amount of temperature of the cake at any time t mins.
From Newton’s Law of Cooling we know that the temperature of a body drops at a rate that is proportional
to the difference between the temperature of the body and the temperature of the surrounding medium.
dT
So  (T − Tm )
dt
dT
or , = k (T − Tm ) ... ... ...(1)
dt
where, k is a proportional constant.
According to the question we have,
T ( 0) = 300 ... ... ... (2)
T ( 3) = 200 ... ... ... (3)
Tm = 70 ... ... ... (4)

Mohammad Abdul Halim, Assistant Professor in Mathematics


Page 121 of 249

Now from Eq. (1) & Eq. (4) we can write,


dT
= k (T − 70 )
dt
dT
or , = kdt
T − 70
dT
or ,  = k  dt
T − 70
or, ln (T − 70 ) = kt + ln c
or, ln (T − 70) = ln ekt + ln c
or , ln (T − 70 ) = ln cekt
or , T − 70 = cekt
 T = 70 + cekt ... ... ... ... (5)
From Eq. (2) & Eq.(5) we have,
300 = 70 + ce k .0
 c = 230
From Eq. (3) & Eq.(5) we have,
200 = 70 + 230e3k
13
or , e3k =
23
 13 
or , 3k = ln  
 23 
1  13 
or , k = ln  
3  23 
 k = −0.19018
Using the values of c & k in Eq.(5) we have,
T = 70 + 230e−0.19018t ... ... ...(5)
This is the amount of temperature of the cake after t mins..
Again, let after t = t1 mins. the amount of temperature of the cake will be 750 F . i.e, T ( t1 ) = 75 .
Now from Eq. (5) we have,
75 = 70 + 230e −0.19018t1
5
or , e −0.19018t1 =
230
 5 
or , − 0.19018t1 = ln  
 230 
1  5 
or , t1 = − ln  
0.19018  230 
or , t1 = 20.13 mins.
This is the required time. (ans.)

Mohammad Abdul Halim, Assistant Professor in Mathematics


Page 122 of 249

Supplementary Questions
1. The population of a certain community increases at a rate that is proportional to the number of
people present at any time. If the population has doubled in 30 years, how long will it take to
triple? initially p ( 0 ) = p0 
2. If a small metal bar, whose initial temperature is 20 0 C , is dropped into a container of boiling
water. How long will it take for the bar to reach 90 0 C , if it is known that temperature increases
 dT 
2 0 C in 1second? T ( 0 ) = 200 , T (1) = 220 , T
 dt 
3. The population of a community is known to increase at a rate proportional to the number of people
present at time t .If the population has doubled in 50 years, how long will it take to triple?
4. Find the time required for a sum of money to double itself at 5% per annum compounded
continuously.
5. In a certain bacteria culture, the rate of increase in the number of bacteria is proportional to the
number present. If the number double in 4 hours, how many will be present in 12 yours? If the
number 400 in 3 hours and 2000 in 10 hours, then find the number initially present?
6. The population of a certain city increases at a rate proportional to the number of its inhabitants at
any time.
a) If the population doubled in 40 years, in how many years will it triple?
b) If the population is 1000 in 3 years, what was the initial population? What will be the population
in 10 years?
7. In a certain bacteria culture, the rate of increase in the number of bacteria is proportional to the
number present.
a) If the number triples in 5 hours, how many will be present in 10 hours.
b) When will be the number present be 10 times the number initially present?

Mohammad Abdul Halim, Assistant Professor in Mathematics


Page 123 of 249

HIGHER ORDER ORDINARY DIFFERENTIAL EQUATIONS WITH FIRST DEGREE

Linear ODE with first degree and higher order with constant coefficient
In this chapter we discuss about the solution of linear higher order ordinary differential equations by using
particular integrals and complementary functions.
General solution (G.S) = Complementary Function (C.F) + Particular Integral (P.I)
Chapter Contents:
➢ Linear higher order ordinary differential equation
➢ Solution of Cauchy Euler Equation
Learning Outcomes: Learners be able to know...
➢ How to determine the general solution of homogeneous linear DE
➢ How to solve the homogeneous linear differential equations with constant coefficients
➢ The solution procedure of Cauchy Euler
Cauchy Euler Equation:
A differential equation of the following form
dny d n −1 y d n−2 y d n −3 y
+ p + p + p + + pn y = X
dx n −1 dx n − 2 dx n −3
1 2 3
dx n
where p1 , p2 , p3 , pn are function of x or constant and X are also function of x or constant is called
a linear differential equation of order n. when p1 , p2 , p3 , pn all constants are then the equation is
called linear differential equation of order n with constant coefficients and otherwise it is called linear
differential equation of order n with variable coefficients (This equation is also known as Cauchy Euler
Equation of order n). When X = 0 then the equation is called homogeneous linear differential equation of
order n and when X a function of x then it is called non-homogeneous linear differential equation of order
n.
Simplest form of the above equation:
Consider a linear differential equation of order n as follows
dny d n −1 y d n−2 y d n −3 y
+ p + p + p + + pn y = X
dx n −1 dx n − 2 dx n −3
1 2 3
dx n
d
Replacing the differential operator by the equivalent operator D the above equation becomes
dx
D n y + p1 D n −1 y + p2 D n − 2 y + p3 D n −3 y + + pn y = X
(D n
+ p1 D n −1 + p2 D n − 2 + p3 D n −3 + + pn ) y = X
f ( D ) y = X [ say ]

Mohammad Abdul Halim, Assistant Professor in Mathematics


Page 124 of 249

Homogeneous linear differential equation of order n:


A differential equation of the following form
dny d n −1 y d n−2 y d n −3 y
+ p + p + p + + pn y = 0 (i )
dx n −1 dx n − 2 dx n −3
1 2 3
dx n
where p1 , p2 , p3 , pn are function of x or constant, is called Homogeneous linear differential
equation of order n.
Theorem: let f1 , f 2 , f3 , f n be the solution of the differential equation (i ) then the linear
combination y = c1 f1 + c2 f 2 + c3 f3 + + cn f n , where c1 , c2 , c3 , , cn arbitrary constants, is
called the general solution (G.S) of the differential equation (i ) .
Auxiliary Equation:
Consider a linear differential equation of order n as follows
dny d n −1 y d n−2 y d n −3 y
+ p + p + p + + pn y = 0 (i )
dx n −1 dx n − 2 dx n −3
1 2 3
dx n
d
Replacing the differential operator by the equivalent operator D the above equation becomes
dx
D n y + p1 D n −1 y + p2 D n − 2 y + p3 D n −3 y + + pn y = 0
(D n
+ p1 D n −1 + p2 D n − 2 + p3 D n −3 + + pn ) y = 0
f ( D) y = 0 (ii )
where f ( D ) = D + p1D
n n −1
+ p2 D n−2
+ p3 D n −3
+ + pn ( Differential operator)
Assume y = e mx be a trial solution of the equation (i ) and if we put it in the left-hand side of equation (i )
the equation (i ) takes the form as follows:
d n (e m x ) d n −1 ( e m x ) d n−2 ( e m x ) d n −3 ( e m x )
n
+ p1 n −1
+ p2 n−2
+ p3 n −3
+ + pn ( e m x ) = 0
dx dx dx dx
m  e + p1. m  e + p2 m  e + p3 .m n −3  e m x +
n mx n −1 mx n−2 mx
+ pn  e m x = 0
(m n
+ p1. m n −1 + p2 m n − 2 + p3 .m n −3 + + pn )  e m x = 0
Here for all values of x , e m x  0 .
So , m n + p1. m n −1 + p2 m n − 2 + p3 .m n −3 + + pn = 0
Say f ( m) = m + p1. m n n −1
+ p2 m n−2
+ p3 .m n −3
+ + pn = 0
The equation f ( m) = mn + p1. mn−1 + p2 mn−2 + p3 .mn−3 + + pn = 0 is called the auxiliary
equation of the equation (i ) . Therefore, it is seen that form the above equation (ii ) we easily obtain
auxiliary equation from the differential operator f ( D ) by replacing/Substituting D by the symbol m .
Hence in future we shall write the auxiliary equation as f ( D ) = 0 and solve it for D .
In this section, we discuss about the solution of homogeneous linear differential equation with
constant coefficients:
Some formulas for the solution of homogeneous linear differential equation with constant coefficients
1. when all roots of the auxiliary equation are real and different or distinct: Say the roots are
m1  m2  m3  m4 then the solution of the differential equation is ,
y = c1e m 1x + c2e + c3e m 3 x + c4e
m2 x m4 x

Mohammad Abdul Halim, Assistant Professor in Mathematics


Page 125 of 249

2. when some roots of the auxiliary equation are real equal and some are different or distinct:
Say the roots are m1  m2  m3 = m (equal ) , m4  m5 then the solution of the differential
equation is,
y = ( c1 + c2 x + c3 x 2 ) e m x + c4e + c5e
m4 x m5 x

3. when some roots of the auxiliary equation are complex: Say the roots are   i  then the
solution of the differential equation is, y = e  x ( c1 cos  x + c2 sin  x )
4. when some roots of the auxiliary equation are repeated complex: Say the roots are   i  and
  i then the solution of the differential equation is ,
y = e  x ( c1 + c2 x ) cos  x + ( c3 + c4 x ) sin  x 
Solution Procedure:
1. Put the differential equation to the form f ( D ) y = 0
2. From auxiliary equation writing f ( D ) = 0
3. Find out the roots of the auxiliary equation for D using a numerical procedure if necessary since
D and m produce equivalent relation.
4. Find the contribution to the general solution from the distinct, real roots, any repeated roots and
any complex roots.
5. Finally the general solution is given by the formula y = yd + yr + yc where yd the contribution
from all distinct real roots is, y r is the contribution from all the repeated roots and yc is the
contribution from all the complex roots.
Mathematical problems
d2y dy
Problem 01: Solve 2
+ 3 + 2y = 0
dx dx
Solution:
Given homogeneous differential equation is
d2y dy
2
+ 3 + 2y = 0
dx dx
Using the differential operator D we write the given differential equation as
D 2 y + 3Dy + 2 y = 0
(D 2
+ 3D + 2 ) y = 0
Now the auxiliary equation of the given differential equation is
D 2 + 3D + 2 = 0
D2 + 2D + D + 2 = 0
D ( D + 2) + 1( D + 2) = 0
( D + 2)( D + 1) = 0
Therefore D + 2 = 0 and D + 1 = 0 that implies D = −2 and D = −1 .
It shows that from the above, roots of the auxiliary equation are different so the general solution/complete
solution of the given differential equation is
y = c1 e −2 x + c2 e − x (As desired)
d4y d3y d2y dy
Problem 02: Solve 4
− 3
− 9 2
− 11 − 4 y = 0
dx dx dx dx
Solution:Given homogeneous differential equation is

Mohammad Abdul Halim, Assistant Professor in Mathematics


Page 126 of 249

d4y d3y d2y dy


4
− 3
− 9 2
− 11 − 4 y = 0
dx dx dx dx
Using the differential operator D we write the given differential equation as
D 4 y − D3 y − 9D 2 y − 11Dy − 4 y = 0
(D 4
− D 3 − 9 D 2 − 11D − 4 ) y = 0
Now the auxiliary equation of the given differential equation is
D 4 − D3 − 9 D 2 − 11D − 4 = 0
D3 ( D + 1) − 2D2 ( D + 1) − 7 D ( D + 1) − 4 ( D + 1) = 0
( D + 1) ( D3 − 2 D 2 − 7 D − 4 ) = 0
( D + 1) D 2 ( D + 1) − 3D ( D + 1) − 4 ( D + 1) = 0
( D + 1) ( D2 − 3D − 4) = 0
2

( D + 1) ( D2 − 4D + D − 4) = 0
2

( D + 1) D( D − 4) + 1( D − 4) = 0
2

( D + 1) ( D + 1)( D + 4) = 0
2

( D + 1) ( D + 4) = 0
3

( D + 1) = 0 and D + 4 = 0
3
Therefore
( D + 1)( D + 1)( D + 1) = 0 D = −4
D = −1, D = −1, D = −1 D = −4
There are four roots of the auxiliary equation in which three are equal and one is different, so the general
solution of the given differential equation is
y = ( c1 + c2 x + c3 x 2 ) e − x + c4 e − 4 x (As desired)
d 4 y d 3 y dy
Problem 03: Solve − − +y=0
dx 4 dx3 dx
Solution:
Given homogeneous differential equation is
d 4 y d 3 y dy
− − +y=0
dx 4 dx3 dx
Using the differential operator D we write the given differential equation as
D 4 y − D3 y − Dy + y = 0
(D 4
− D 3 − D + 1) y = 0
Now the auxiliary equation of the given differential equation is
D 4 − D3 − D + 1 = 0
D3 ( D − 1) − 1( D − 1) = 0
( D − 1) ( D3 − 1) = 0
( D − 1)( D − 1) ( D 2 + D + 1) = 0

Mohammad Abdul Halim, Assistant Professor in Mathematics


Page 127 of 249

Therefore ( D −1) = 0 , ( D −1) = 0 and ( D 2 + D + 1) = 0


−1  1 − 4.1.1 1 3
D = 1 , D = 1 and D = =−  i
2.1 2 2
There are four roots of the auxiliary equation in which two are equal and two are complex, so the general
solution of the given differential equation is
1
− x  3 3 
y = ( c1 + c2 x ) e x + e 2
 c3 cos x + c4 sin x  (As desired)
 2 2 

Non-Homogeneous linear differential equation of order n:


A differential equation of the following form
dny d n −1 y d n−2 y d n −3 y
+ p + p + p + + pn y = X
dx n −1 dx n − 2 dx n −3
1 2 3
dx n
where p1 , p2 , p3 , pn are function of x or constant and X are also function of x or constant is called
a non-homogeneous linear differential equation of order n.
Simplest form of the above equation is f ( D) y = X [ say ] (i )

In this section we discuss about the solution of non-homogeneous linear differential equation with
constant coefficients:
Complementary function:
Complementary function is a general linear combination of n linearly independent solution of the
homogeneous linear differential equation with constant coefficients f ( D) y = 0 . We shall denote this by
yc .
Particular Integral:
Particular solution of the differential equation f ( D ) y = X involving no arbitrary constants is called
particular integral of f ( D ) y = X .we shall denote this by y p .
Calculation of particular Integral:
Generally particular integral comes from the particular solution of the deferential equation f ( D ) y = X by
1
the equation y = X.
f ( D)
1 1
Note that: The symbol stands for integration and also 2 stands for the operation of integration twice.
D D
Particular integral comes from particular solution of f ( D ) y = X so it does not belongs any arbitrary
constant.
Complete primitive/General solution:
The general solution/complete solution of non-homogeneous linear differential equation with constant
coefficients is the sum of complementary function ( C.F ) and particular integral ( P.I ) that is 𝑦 = 𝐶. 𝐹 +
𝑃. 𝐼 = 𝑦𝑐 + 𝑦𝑝 .
Some formulas for finding out the particular integral of the non-homogeneous linear differential equation
with constant coefficients:
1
 f ( D)
−1
➢ When X = x m or polynomial then expand or in ascending power of D and
f ( D)
operate on x m or polynomial .

Mohammad Abdul Halim, Assistant Professor in Mathematics


Page 128 of 249

1 1
➢ When X = e a x then (e a x ) = (e a x ), f (a)  0.
f ( D) f (a )
1 1 1
• If f ( a ) = 0 then (e a x ) = x (e a x ) or x (e a x )
f ( D) f (a) f ( D)
1 1 1
• Again if f ( a ) = 0 then (e a x ) = x 2 (e a x ) or x 2 (e a x )
f ( D) f (a) f ( D)
1 1
➢ When X = sin ax then (sin ax) = (sin ax) , f (−a 2 )  0.
2
f (D ) f (−a 2 )
1 1
• If f (−a 2 ) = 0 then (sin ax) = x (sin ax)
f (D )2
f (−a 2 )
1 1
• Again if f (−a 2 ) = 0 then (sin ax) = x 2 (sin ax)
2
f (D ) f (−a 2 )
Note: we substitute D 2 by − a 2 and this above formula is same for X = cos ax .
1 1
➢ When X = e a xV where V is a function of x then (e a xV ) = e a x . (V )
f ( D) f ( D + a)
➢ When X = x mV where V has the form cos ax or sin ax then x mV becomes x m cos ax or
x m sin ax that’s are the real and imaginary part of x me i a x = x m( cos ax + i sin ax ) respectively
and P.I can be easily calculated such as
1
f ( D)
( x m cos ax) = real part of
1
f ( D)
 x m( cos ax + i sin ax )

1
= real part of x meiax  ei = cos  + i sin  
f ( D)
1
f ( D)
( x msin ax) = Imaginary part of
1
f ( D)
 x m( cos ax + i sin ax )

1
= Imaginary part of x meiax  ei = cos  + i sin  
f ( D)
Important formulae:
𝑑 1
➢ 𝐷 = 𝑑𝑥 𝑎𝑛𝑑 𝐷 = ∫[ ]𝑑𝑥
➢ (1 + 𝑥)−1 = 1 − 𝑥 + 𝑥2 − 𝑥3 + ⋯ ⋯ ⋯ ⋯ ⋯ ⋯ ⋯ ⋯ ⋯ ⋯ + ∞
➢ (1 − 𝑥)−1 = 1 + 𝑥 + 𝑥2 + 𝑥3 + ⋯ ⋯ ⋯ ⋯ ⋯ ⋯ ⋯ ⋯ ⋯ ⋯ + ∞
➢ (1 + 𝑥)−2 = 1 + 2𝑥 + 3𝑥 2 + 4𝑥 3 + ⋯ ⋯ ⋯ ⋯ ⋯ ⋯ ⋯ ⋯ + ∞
➢ (1 − 𝑥)−2 = 1 − 2𝑥 + 3𝑥 2 − 4𝑥 3 + ⋯ ⋯ ⋯ ⋯ ⋯ ⋯ ⋯ ⋯ ∞
➢ (1 − 𝑥)−3 = 1 + 3𝑥 + 6𝑥 2 + 10𝑥 3 + ⋯ ⋯ ⋯ ⋯ ⋯ ⋯ ⋯ ⋯ ∞
Solution Procedure:
1. Put the differential equation to the form f ( D ) y = X
2. From auxiliary equation f ( D ) = 0 find complementary function yc .
1
3. Also find particular integral y p from the integral y = X.
f ( D)

Mohammad Abdul Halim, Assistant Professor in Mathematics


Page 129 of 249

4. Finally, the general solution of non-homogeneous equation is calculated by the formula


y = yc + y p where yc is complementary function and y p is particular integral.
Mathematical problems
Problem 01: Solve D y + 3Dy + 2 y = 4 x .
2

Solution: Given that,


D 2 y + 3Dy + 2 y = 4 x
(D 2
)
+ 3D + 2 y = 4 x
Therefore, the auxiliary equation is,
D 2 + 3D + 2 = 0
D 2 + 2D + D + 2 = 0
D( D + 2) + 1( D + 2) = 0
( D + 2)( D + 1) = 0
 ( D + 2) = 0 or ( D + 1) = 0
 D = −2 or D = −1
The complementary function is,
yc = c1e − x + c2 e −2 x
The particular integral is,
−1
1 1 1 1 3 
yp = 2 ( 4x) = ( 4x) = 1 +  D 2 + D   ( 4x )
D + 3D + 2   1 2 3  2 2 2 
2 1 +  D + D  
 2 2 
1  1  3 
2
 1 2 3  1 2 3  
= 1 −  D + D  +  D + D  −  ( 4 x ) = 4 x −  0 + .4  
 2
2 2  2 2  
 2  2 
= 2x − 3
Therefore, the general solution of equation (1) is,
y = yc + y p
= c1e − x + c2 e −2 x + 2 x + 3 , where c1 , c2 are arbitrary constants.
Problem-02: Solve D y + 5Dy + 4 y = 3 − 2 x .
2

Solution: Given that,


D 2 y + 5Dy + 4 y = 3 − 2 x
(D 2
)
+ 5D + 4 y = 3 − 2 x
The auxiliary equation is,
D 2 + 5D + 4 = 0
D 2 + 4D + D + 4 = 0
D( D + 4) + 1( D + 4) = 0
( D + 4)( D + 1) = 0
Consequently, D + 4 = 0 or D + 1 = 0
D=−4 or D = − 1
The complementary function is,
yc = c1e − x + c2 e −4 x
The particular integral is,

Mohammad Abdul Halim, Assistant Professor in Mathematics


Page 130 of 249

1 1
yp = (3 − 2x ) = (3 − 2x )
D + 5D + 4
2
  1 2 5 
4 1 +  D + D  
 4 4 
−1
1 1 5 
= 1 +  D 2 + D   (3 − 2x )
4 4 4 
1  1  5 
2
 1 2 5  1 2 5  
= 1 −  D + D  +  D + D  −  ( 3 − 2 x ) = 3 − 2 x −  0 −  
 4
4 4  4 4  
 4  2 
3 1 5 11 1
= − x+ = − x
4 2 8 8 2
Therefore, the general solution of equation is,
y = yc + y p
11 1
= c1e − x + c2 e −4 x + − x , where c1 , c2 are arbitrary constants.
8 2

Problem-03: Solve D y + 2 Dy + y = 2 x + x .
2 2

Solution: Given that,


D 2 y + 2Dy + y = 2 x + x 2
(D 2
)
+ 2D + 1 y = 2 x + x 2
The auxiliary equation is,
D 2 + 2D + 1 = 0
(D + 1)2 = 0  ( D + 1)( D + 1) = 0
Therefore D + 1 = 0 , D + 1 = 0  D = −1 and D = −1
The complementary function is,
yc = ( c1 + c2 x ) e− x
The particular integral is,

( ) ( ) ( ) ( )
1 1 −1
yp = 2 x + x 2
= 2 x + x 2
= 1 + D 2
+ 2 D  2 x + x2
D + 2D + 1 1 + ( D + 2D )  
2 2

= 1 − ( D 2 + 2 D ) + ( D 2 + 2 D ) −  ( 2 x + x2 )
2

 
=  2 x + x 2 − 2 + 2 ( 2 + 2 x ) + 8
= 2 x + x2 − 2 − 4 − 4 x + 8 = x2 − 2 x + 2
Therefore, the general solution of equation is,
y = yc + y p = ( c1 + c2 x ) e− x + x2 − 2 x + 2 , where c1 , c2 are arbitrary constants.
Problem-04: Solve D y − 6 Dy + 9 y = 1 + x + x .
2 2

Solution: Given that,


D 2 y − 6Dy + 9 y = 1 + x + x 2
(D 2
)
− 6D + 9 y = 1 + x + x 2
The auxiliary equation is,
D 2 − 6D + 9 = 0

Mohammad Abdul Halim, Assistant Professor in Mathematics


Page 131 of 249

( D − 3) 2 = 0  ( D − 3)( D − 3) = 0
D − 3 = 0 or D − 3 = 0  D = 3 , D = 3
The complementary function is,
yc = ( c1 + c2 x ) e3 x
The particular integral is,
yp =
1
D − 6D + 9
2 (1 + x + x2 ) =
1
  1 2 2 
1 + x + x2 ( )
9 1 +  D − D  
 9 3 
−1
1 1 2 
= 1 +  D 2 − D  
9 9 3 
(1 + x + x ) 2

1 1  2
2  1 2 
= 1 −  D2 − D  +  D2 − D  −  (1 + x + x )
2

9   9 3  9 3  
1 2 2  8 1  8 1
9 9 3  9 9 
2 2 4
= 1 + x + x 2 −  − (1 + 2 x )  +  = 1 + x + x 2 − + + x +  =
9 3 3 9  27
( 3x 2 + 7 x + 7 )

Therefore, the general solution of equation is,


y = yc + y p = ( c1 + c2 x ) e3 x +
1
27
( 3x 2 + 7 x + 7 ) , where c1 , c2 are arbitrary constants.
Problem-05: Solve D y − 2 D y + D y = x .
4 3 2 3

Solution: Given that,


D 4 y − 2D 3 y + D 2 y = x 3
(D 4
)
− 2D 3 + D 2 y = x 3
The auxiliary equation is,
D 4 − 2D 3 + D 2 = 0
D 2 ( D 2 − 2 D + 1) = 0
D 2 = 0 or ( D − 1) 2 = 0
D = 0 , D = 0 or D = 1 , D = 1
The complementary function is,
yc = c1 + c2 x + ( c3 + c4 x ) e x
The particular integral is,

2 (
x3 ) = 2 ( )
1 + ( D 2 − 2 D )  ( x 3 )
1 1 1 −1
yp = x 3
=
D − 2D + D
4 3
D 1 + D 2 − 2 D  ( )D 
2 

= 2  x3 − ( 6 x − 6 x 2 ) + ( −24 + 24 x )  = 2 ( x 3 − 6 x + 6 x 2 − 24 + 24 x )
1 1
D D
= 2 ( x 3 + 6 x 2 + 18 x − 24 ) =
1 1 5 1 4
x + x + 3x 3 − 12 x 2
D 20 2
Therefore, the general solution of equation is,
1 5 1 4
y = yc + y p = c1 + c2 x + ( c3 + c4 x ) e x + x + x + 3x 3 − 12 x 2
20 2

Mohammad Abdul Halim, Assistant Professor in Mathematics


Page 132 of 249

Problem 06: Solve D y − Dy − 2 y = e .


2 x

Solution: Given that,


D 2 y − Dy − 2 y = e x
(D 2
)
− D − 2 y = ex
The auxiliary equation is,
D2 − D − 2 = 0
D 2 − 2D + D − 2 = 0
D( D − 2) + 1( D − 2) = 0
( D − 2)( D + 1) = 0
That implies D = −1 and D = 2
The complementary function is,
yc = c1e − x + c2e 2 x
The particular integral is,
ex ex
yp = 2
D −D−2
1
( ex ) = 2
1 −1 − 2
=−
2
Therefore, the general solution of equation is,
ex
y = yc + y p = c1e− x + c2e2 x − , where c1 , c2 are arbitrary constants.
2
−3 x
Problem-07: Solve D y + 4 Dy + 3 y = e .
2

Solution: Given that,


D 2 y + 4 Dy + 3 y = e −3 x
(D 2
)
+ 4 D + 3 y = e −3 x
The auxiliary equation is,
D 2 + 4D + 3 = 0
D 2 + 3D + D + 3 = 0
D( D + 3) + 1( D + 3) = 0
( D + 3)( D + 1) = 0
That implies D = −1 & D = −3
The complementary function is,
yc = c1e − x + c2 e −3 x
The particular integral of (1) is,
xe −3 x
yp = 2
1
D + 4D + 3
(
e −3 x = ) x
2D + 4
( )
e −3 x =
2 ( −3) + 4
1
= − xe −3 x
2
Therefore, the general solution of equation is,
1
y = yc + y p = c1e − x + c2 e −3 x − xe −3 x , where c1 , c2 are arbitrary constants.
2
−x
Problem-08: Solve D y + y = 3 + e + 5e .
3 2x

Solution: Given that,


D 3 y + y = 3 + e − x + 5e 2 x
(D )
+ 1 y = 3 + e − x + 5e 2 x
3

The auxiliary equation is, D 3 + 1 = 0

Mohammad Abdul Halim, Assistant Professor in Mathematics


Page 133 of 249

(D + 1)(D 2 − D + 1) = 0
1  1 − 4.1.1 1  i 3 1 3
That implies D = −1 and D 2 − D + 1 = 0  D = = = i
2.1 2 2 2
The complementary function is,
x   3   3  
yc = c1e− x + e 2 c2 cos  x  + c3 sin  x  
  2   2  
The particular integral is,
yp =
1
D +1
(
3
3 + e − x + 5e 2 x ) = 3
1
D +1
( 3) + 3 ( e− x ) + 3 ( 5e2 x )
1
D +1
1
D +1
2x
xe− x 5e2 x
( ) x
( )
5e
( )
−1
= 1 + D 3 ( 3) + e −x
+ = 1 − D 3
+ D 6
− ( )
3 + +
23 + 1 3 ( −1)
2
3D 2 9
1 5
= 3 + xe − x + e 2 x
3 9
Therefore, the general solution of equation is,
x   3   3   1 5
y = yc + y p = c1e− x + e 2 c2 cos  x  + c3 sin  x   + 3 + xe − x + e 2 x (As desired)

  2   2   3 9
−x
Problem-09: Solve D y + 3D y + 3Dy + y = e .
3 2

Solution: Given that,


D 3 y + 3D 2 y + 3Dy + y = e − x
(D 3
)
+ 3D 2 + 3D + 1 y = e − x
The auxiliary equation is,
D 3 + 3D 2 + 3D + 1 = 0
( D + 1) 3 = 0
That implies D = −1 , D = −1 & D = −1
The complementary function is,
yc = ( c1 + c2 x + c3 x 2 ) e − x
The particular integral is,
x2 x3 − x
yp =
1
D 3 + 3D 2 + 3D + 1
( e −x
) =
x
3D 2 + 6 D + 3
( e −x
) =
6D + 6
e −x
=
6
e ( )
Therefore, the general solution of equation is,
x3 − x
(
y = yc + y p = c1 + c2 x + c3 x 2 e − x + ) 6
e , where c1 , c2 , c3 are arbitrary constants.

Problem-10: Solve 2 D y − 3D y + y = e + 1 .
3 2 x

Solution: Given that,


2 D 3 y − 3D 2 y + y = e x + 1
(2D 3
)
− 3D 2 + 1 y = e x + 1
The auxiliary equation is,
2 D 3 − 3D 2 + 1 = 0
2 D 2 (D − 1) − D(D − 1) − ( D − 1) = 0

Mohammad Abdul Halim, Assistant Professor in Mathematics


Page 134 of 249

(
( D − 1) 2 D 2 − D − 1 = 0 )
1  1 + 4.2.1 1  3 1
That implies D = 1 or 2 D 2 − D − 1 = 0  D = =  D = 1, D = −
2.2 4 2
The complementary function is,
x

yc = ( c1 + c2 x ) e x + c3e 2

The particular integral is,


yp =
1
2 D − 3D + 1
3 2 ( e x + 1)

=
x
6D − 6D
2 ( ex ) +
1
2 D − 3D 2 + 1
3

x2
=
12 D − 6
( )
ex + 1

x2
=
12 (1) − 6
( ex ) + 1

1 2 x
= x e +1
6
Therefore, the general solution of equation is,
x
− 1
y = yc + y p = ( c1 + c2 x ) e + c3e x 2
+ x 2e x + 1 , where c1 , c2 and c3 are arbitrary constants.
6
Problem-11: Solve D y + 4 y = sin 3x .
2

Solution: Given that,


D 2 y + 4 y = sin 3x
(D 2
)
+ 4 y = sin 3x
The auxiliary equation is,
D2 + 4 = 0
D 2 = − 4 = 4i 2
D =  2i
The complementary function is,
yc = c1 cos 2 x + c2 sin 2 x
The particular integral is,

1
yp = ( sin 3x )
D +4 2

1
= ( sin 3x )
−3 + 4
2

1
= ( sin 3x )
−9 + 4
1
= − sin 3 x
5

Mohammad Abdul Halim, Assistant Professor in Mathematics


Page 135 of 249

Therefore, the general solution of equation is,

y = yc + y p

1
= c1 cos 2 x + c2 sin 2 x − sin 3 x , where c1 and c2 are arbitrary constants.
5

Problem-12: Solve D y − 2Dy + 5 y = 10sin x .


2

Solution: Given that,


D 2 y − 2 Dy + 5 y = 10 sin x
(D 2
)
− 2D + 5 y = 10 sin x
The auxiliary equation is,
D 2 − 2D + 5 = 0
2  4 − 4.1.5 2  4i
D= = = 1  2i
2 .1 2
Therefore, D = 1 + 2i D = 1 − 2i .
The complementary function is,
yc = ( c1 cos 2 x + c2 sin 2 x ) e x
The particular integral is,
1 1
yp = (10sin x ) = 2 (10sin x )
D − 2D + 5
2
−1 − 2 D + 5
1 1 (2 + D)
= (10sin x ) = (10sin x ) = 2 2 (10sin x )
4 − 2D 2 (2 − D) 2 2 −D  
(2 + D)
= (10sin x )

2 22 − ( −12 ) 
1
= ( 20sin x + 10 cos x ) = 2sin x + cos x
10
Therefore, the general solution of equation is,
y = yc + y p
= ( c1 cos 2 x + c2 sin 2 x ) e x + 2sin x + cos x , where c1 and c2 are arbitrary constants.
Problem-13: Solve D y − 8Dy + 16 y = 5cos3x .
2

Solution: Given that,


D2 y − 8Dy + 16 y = 5cos3x
(D 2 − 8D + 16) y = 5 cos 3x
The auxiliary equation is,
D 2 − 8 D + 16 = 0
D 2 − 2.D.4 + 4 2 = 0
( D − 4 )2 =0
(D − 4)(D − 4) = 0
Therefore, D = 4 and D = 4

Mohammad Abdul Halim, Assistant Professor in Mathematics


Page 136 of 249

The complementary function is,


yc = ( c1 + c2 x ) e4 x
The particular integral is,
1 1
yp = ( 5cos 3x ) = 2 ( 5cos 3x )
D − 8 D + 16
2
−3 − 8 D + 16
1 ( 7 + 8D ) 5cos 3x = ( 7 + 8D ) 5cos 3x
( 5cos 3x ) = 2 2 ( ) ( )
=
7 − 8D 7 − ( 8D ) 49 − 64 D 2
1 1
= ( 35cos 3x − 120sin 3x ) = ( 35cos 3x − 120sin 3x )
49 − 64 ( −3 )
2
625
1
= ( 7 cos 3x − 24sin 3 x )
125
Therefore, the general solution of equation is,
y = yc + y p
1
= ( c1 + c2 x ) e 4 x +
( 7 cos 3x − 24sin 3x ) , where c1 , c2 are arbitrary constants.
125
Problem-14: Solve D y − 3Dy + 4 y = cos ( 3x + 5) .
2

Solution: Given that, D2 y − 3Dy + 4 y = cos ( 3x + 5)


(D 2
)
− 3D + 4 y = cos(3x + 5)
The auxiliary equation is,
D 2 − 3D + 4 = 0
3  9 − 16 3  i 7
D= =
2 2
The complementary function is,
  7   7   32x
yc = c1 cos  x  + c2 sin  x   e
  2   2  
The particular integral is,
1
yp = cos ( 4 x + 5 ) 
D − 3D + 4 
2

1
= 2  cos ( 4 x + 5 ) 
−4 − 3 D + 4 
1
= cos ( 4 x + 5 ) 
−12 − 3D 
1
=− cos ( 4 x + 5)
3( 4 + D) 

=−
(4 − D) cos ( 4 x + 5 ) 
3 ( 42 − D 2 )

=−
(4 − D) cos ( 4 x + 5) 

3 42 − ( −42 ) 
Mohammad Abdul Halim, Assistant Professor in Mathematics
Page 137 of 249

1
=−  4 cos ( 4 x + 5 ) + 4sin ( 4 x + 5 ) 
96 
1
=− cos ( 4 x + 5 ) + sin ( 4 x + 5 ) 
24 
Therefore, the general solution of equation is,
  7   7   32x 1
y = yc + y p = c1 cos  x  + c2 sin  x   e − cos ( 4 x + 5 ) + sin ( 4 x + 5 )  , where
  2   2   24
c1 , c2
are arbitrary constants.
Problem-15: Solve D y + y = sin 2 x sin x .
2

Solution: Given that, D y + y = sin 2 x sin x


2

D 2 y + y = sin 2 x sin x
(D 2
+ 1) y = sin 2 x sin x
The auxiliary equation is, D 2 + 1 = 0
D 2 = −1 = i 2
D =  i2 =  i
The complementary function is,
yc = c1 cos x + c2 sin x
The particular integral is,

( sin 2 x sin x ) = 2   2sin 2 x sin x 


1 1 1
yp =
D +1 2
D +1 2 

( cos x − cos 3 x ) =  2 ( cos x ) − 2 ( cos 3x ) 


1 1 1 1 1
=
2 D +1
2
2  D +1 D +1 
1 x
( cos x ) − 2 ( cos 3x )  =  2 ( cos x ) + cos 3x 
1 1 xD 1
= 
2  2D −3 + 1  2  2D 8 
1  xD 1  1 x 1  1
=  ( cos x ) + cos 3 x  =  ( − sin x ) + cos 3 x  = ( 4 x sin x + cos 3 x )
2  2 ( −1 )
2
8  2  −2 8  16

Therefore, the general solution of equation is,

1
y = yc + y p = c1 cos x + c2 sin x + ( 4 x sin x + cos 3x ) where c1 , c2 are arbitrary constants.
16
Problem-16: Solve D y + 4 y = sin x .
2 2

Solution: Given that, D y + 4 y = sin x


2 2

(D 2
+ 4 ) y = sin 2 x
The auxiliary equation is,
D2 + 4 = 0
D2 = 4i 2  D =  2i

Mohammad Abdul Halim, Assistant Professor in Mathematics


Page 138 of 249

The complementary function is,


yc = c1 cos 2 x + c2 sin 2 x
The particular integral is,
yp =
1
D +4
2 ( sin 2 x )
 
 
1 1 2  1 1 1 1 1 x 
= 2   2sin x  = (1 − cos 2 x ) =

− ( cos 2 x ) 
D +42  2 D +4 2 4  D  2D
2 2

 1 +  
  4  
1  1  D2  
−1
xD
=  1 +  (1 ) − ( cos 2 x ) 
2 4  4  2D2 

1  1  D2 D4  xD 
=  1 − + −  (1 ) − ( cos 2 x ) 
2 4 

4 16  2 ( −2 2
) 
1 1 x  11 x  1
=  + ( −2sin 2 x )  =  − sin 2 x  = (1 − x sin 2 x )
2 4 8  24 4  8
Therefore, the general solution of equation is,
1
y = yc + y p = c1 cos 2 x + c2 sin 2 x + (1 − x sin 2 x ) where, c1 , c2 are arbitrary constants.
8
d2y dy
Problem 17: Solve 2
− 2 + 5 y = 5cos x
dx dx
Solution:
Given non-homogeneous differential equation is
d2y dy
2
− 2 + 5 y = 5cos x
dx dx
Using the differential operator D we write the given differential equation as
D 2 y − 2 Dy + 5 y = 5cos x
(D 2
− 2 D + 5 ) y = 5cos x
Now the auxiliary equation of the given differential equation is
D2 − 2D + 5 = 0
2  4 − 4.1.5 2  4 − 20 2  −16 2  4i
D= = = = = 1  2i =   i
2.1 2 2 2
Therefore, the complementary function of the given differential equation is
yc = e1. x ( c1 cos 2 x + c2 sin 2 x )
= e x ( c1 cos 2 x + c2 sin 2 x )
Now particular Integral is
1 1 1
yp = ( 5cos x ) = 5. 2 ( cos x ) = 5. 2 ( cos x )
D − 2D + 5
2
D − 2D + 5 −1 − 2 D + 5

Mohammad Abdul Halim, Assistant Professor in Mathematics


Page 139 of 249

1 5 1 5 1 ( 2 + D )
= 5. ( cos x ) = . ( cos x ) = . ( cos x )
4 − 2D 2 2−D 2 (2 − D) (2 + D)
5 (2 + D) 5 (2 + D) 5 (2 + D)
= . ( cos x ) = . ( cos x ) = . ( cos x )
2 (4 − D )
2
2 ( 4 − (−1 ) )
2
2 5
1 1 1
= . ( 2 + D )( cos x ) = . ( 2 cos x + D cos x ) = . ( 2 cos x − sin x )
2 2 2
Therefore, the general solution of non-homogeneous equation is,
1
y = yc + y p = e x ( c1 cos 2 x + c2 sin 2 x ) + ( 2 cos x − sin x ) (As desired)
2
−x
Problem-18: Solve D y − 4 Dy − 5 y = xe .
2

−x
Solution: Given that, D y − 4 Dy − 5 y = xe
2

(D 2
− 4 D − 5 ) y = xe − x
The auxiliary equation is,
D2 − 4D − 5 = 0
D 2 − 5D + D − 5 = 0
D ( D − 5) + 1( D − 5) = 0
( D − 5)( D + 1) = 0
D = 5 , D = −1
The complementary function is,
yc = c1e − x + c2e5 x
The particular integral is,
yp = 2
1
( )
xe − x = e − x
1
( x)
D − 4D − 5 ( D − 1) − 4 ( D − 1) − 5
2

1 1
= e− x ( x ) = e− x 2 ( x)
D − 2D + 1 − 4D + 4 − 5
2
D − 6D

( )
1 1 1 1 −1
= − e− x ( x ) = − e− x 1 − D 6 ( x )
6 D 1− D
6 ( ) 6 D

= − e− x 1 + D + D
1 1  x
( )
2
( ) = − e
2 1 −x 1
+ x+ 1
6 D 6 36  6 D 6
1 − x  x2 x 
= − e  +  = − e− x ( x + 3x 2 )
1
6
6  2  36
Therefore, the general solution of equation (1) is,
e ( x + 3x 2 ) where c1 , c2 are arbitrary constants.
1 −x
y = yc + y p = c1e − x + c2 e5 x −
36
Problem-19: Solve D y − y = ( x + 3) e .
2 2x

Solution: Given that, D y − y = ( x + 3) e


2 2x

Mohammad Abdul Halim, Assistant Professor in Mathematics


Page 140 of 249

(D 2
− 1) y = ( x + 3) e 2 x
The auxiliary equation is,
D 2 − 1 = 0  D 2 = 1  D = 1
The complementary function is,
yc = c1e − x + c2e x
The particular integral is,
1 1 1
yp = ( x + 3) e 2 x = e 2 x ( x + 3) = e2 x 2 ( x + 3)
D −1 ( D + 2) −1 D + 4D + 4 − 1
2 2

1 1 1
= e2 x ( x + 3) = e 2 x ( x + 3)
D + 4D + 3
2
3  4 1 2 
1 +  3 D + 3 D  
  
−1
1  4 1 
= e2 x 1 +  D + D 2   ( x + 3)
3  3 3 
1  4 1  4 1 
2

= e2 x 1 −  D + D2  +  D + D 2  −  ( x + 3)
3   3 3  3 3  
1  4   1
= e2 x  x + 3 −  + 0  + 0 = e2 x ( 3x + 5 )
3  3   9
Therefore, the general solution of equation is,
1
y = yc + y p = c1e − x + c2e x + e 2 x ( 3x + 5 ) where, c1 , c2 are arbitrary constants.
9
Problem-20: Solve D y − 2 Dy + 4 y = e cos x .
3 x

Solution: Given that, D y − 2 Dy + 4 y = e cos x


3 x

(D 3
− 2 D + 4 ) y = e x cos x
The auxiliary equation is,
D3 − 2 D + 4 = 0
 D3 + 2 D 2 − 2 D 2 − 4 D + 2 D + 4 = 0
 D2 ( D + 2 ) − 2 D ( D + 2 ) + 2 ( D + 2 ) = 0
 ( D + 2) ( D2 − 2D + 2) = 0
 D + 2 = 0 or , D 2 − 2D + 2 = 0
2  4 − 4 2
 D = −2 or , D =
2
2  −4 2  4i 2 2  2i
= = = = 1 i
2 2 2
 D = −2,1 + i, 1 − i
The complementary function is,
yc = c1e−2 x + ( c2 cos x + c3 sin x ) e x
The particular integral is,

Mohammad Abdul Halim, Assistant Professor in Mathematics


Page 141 of 249

yp =
1
( e x cos x ) = e x
1
( cos x )
D − 2D + 4 ( D + 1) − 2 ( D + 1) + 4
3 3

1 1
= ex ( cos x ) = e x 3 ( cos x )
D + 3D + 3D + 1 − 2 D − 2 + 4
3 2
D + 3D 2 + D + 3
x x
= ex ( cos x ) = e x ( cos x )
3D + 6 D + 1 3 ( −1 ) + 6 D + 1
2 2

x 1 x
= ex ( cos x ) = − e x ( cos x )
−2 + 6 D 2 (1 − 3D )
1 x (1 + 3D ) 1 x (1 + 3D ) 1 x
= − ex ( cos x ) = − e x ( cos x ) = − xe ( cos x − 3sin x )
2 
1 − ( 3D )
2
 2 1 − 9 −12  ( ) 20
Therefore, the general solution of equation is,
1 x
y = yc + y p = c1e −2 x + ( c2 cos x + c3 sin x ) e x − xe ( cos x − 3sin x ) where c1 , c2 , c3 are
20
arbitrary
constants.
Problem-21: Solve D y − 2 Dy + 2 y = e sin x .
2 x

Solution: Given that,


D2 y − 2Dy + 2 y = e x sin x
(D 2
− 2 D + 2 ) y = e x sin x
The auxiliary equation is,
D2 − 2D + 2 = 0
2  4 − 4  2 2  −4 2  4i 2 2  2i
 D= = = = = 1 i
2 2 2 2
 D = 1 + i,1 − i
The complementary function is,
yc = ( c1 cos x + c2 sin x ) e x
The particular integral is,
yp =
1
( e x sin x ) = e x
1
( sin x )
D − 2D + 2 ( D + 1) − 2 ( D + 1) + 2
2 2

1 1 x
= ex ( sin x ) = e x 2 ( sin x ) = e x ( sin x )
D + 2D + 1 − 2D − 2 + 2
2
D +1 2D
xD xD 1
2 (
= ex sin x ) = e x ( sin x ) = − xe x ( cos x )
2D 2 ( −1 )
2
2
Therefore, the general solution of equation is,
1 x
y = yc + y p = ( c1 cos x + c2 sin x ) e x − xe ( cos x ) where c1 , c2 are arbitrary constants.
2
Problem-22: Solve D y + 3Dy + 2 y = e sin x .
2 2x

Solution: Given that, D y + 3Dy + 2 y = e sin x


2 2x

Mohammad Abdul Halim, Assistant Professor in Mathematics


Page 142 of 249

(D 2
+ 3D + 2 ) y = e 2 x sin x
The auxiliary equation is,
D 2 + 3D + 2 = 0
 D2 + 2D + D + 2 = 0
 D ( D + 2) + ( D + 2) = 0
 ( D + 1)( D + 2) = 0
 D = −1, − 2
The complementary function is,
yc = c1e − x + c2 e −2 x
The particular integral is,
yp = 2
1
( e 2 x sin x ) = e 2 x
1
( sin x )
D + 3D + 2 ( D + 2) + 3( D + 2) + 2
2

1 1
= e2 x ( sin x ) = e2 x 2 ( sin x )
D + 4 D + 4 + 3D + 6 + 2
2
D + 7 D + 12
1 1 (11 − 7 D ) sin x
( sin x ) = e2 x ( sin x ) = e2 x 2 ( )
= e2 x 2
−1 + 7 D + 12 11 + 7 D (11) − ( 7 D )
2

(11 − 7 D ) (11 − 7 D ) 1
= e2 x ( sin x ) = e2 x ( sin x ) = (11sin x − 7 cos x ) e2 x
121 − 49 D 121 − 49 ( −1 )
2 2
170
Therefore, the general solution of equation is,
1
y = yc + y p = c1e − x + c2e −2 x + (11sin x − 7 cos x ) e 2 x where, c1 , c2 are arbitrary constants.
170
Problem-23: Solve D y − 2 Dy + y = x sin x .
2

Solution: Given that,


D 2 y − 2 Dy + y = x sin x
(D 2
− 2 D + 1) y = x sin x
The auxiliary equation is,
D 2 − 2 D + 1 = 0  ( D − 1) = 0
2

 D = 1, D = 1
The complementary function is,
yc = ( c1 + c2 x ) e x
The particular integral is,
1
yp = ( x sin x )
D − 2D + 1
2

 
= Imaginary Part of  2
1
 D − 2D + 1
( xe ix ) 

  
( x ) 
1 1
= I . P. of eix ( x )  = I . P. of eix 2
 ( D + i ) − 2 ( D + i ) + 1   + − − − + 
2
D 2 Di 1 2 D 2i 1

Mohammad Abdul Halim, Assistant Professor in Mathematics


Page 143 of 249

 
  
( x )  = I . P. of  −
ix
1 e 1
= I . P. of eix 2 ( x )
 D + 2 Di − 2 D − 2i   2i 1 − 1 D 2 + 2 Di − 2 D 
 
 2i
( ) 



 ieix  1 2 
−1

= I .P. of  1 − (
D + 2Di − 2D  ( x ) )
 2  2i  
 ieix  1  
1 + ( D + 2 Di − 2 D ) + 2 ( D + 2 Di − 2 D ) +
1
 ( x )
2
= I .P. of  2 2

 2  2i 4i  
 ieix    ieix 
= I .P. of   x +
1
( 2i − 2 )  = I .P. of   x + (1 + i )
 2  2i   2 
i 
= I .P. of  ( cos x + i sin x ) ( x + 1) + i
2 
i 
= I .P. of  ( x + 1) cos x + i cos x + i ( x + 1) sin x − sin x
2 
1 
= I .P. of  i ( x + 1) cos x − cos x − ( x + 1) sin x − i sin x
2 
= ( x + 1) cos x − sin x
1
2
Therefore, the general solution of equation is,
y = yc + y p = ( c1 + c2 x ) e x +
1
2
( x + 1) cos x − sin x where, c1 , c2 are arbitrary constants.
Problem-24: Solve D y + Dy = x cos x .
2

Solution: Given that, D y + Dy = x cos x


2

(D 2
+ D ) y = x cos x
The auxiliary equation is,
D 2 + D = 0  D ( D + 1) = 0
 D = 0, D = −1
The complementary function is,
yc = c1 + c2 e − x
The particular integral is,

( x cos x ) = Re al Part of  2 ( 
xeix ) 
1 1
yp =
D +D 2
D + D 
 1   
= R . P. of eix ( x ) = R . P. of eix 2 1
( x ) 
 ( D + i ) + ( D + i )  D + 2iD + i + ( D + i )
2

2

  
( x )  = R . P. of eix 2
1 1
= R . P. of eix 2 ( x )
 D + 2iD − 1 + D + i   D + ( 2i + 1) D + ( i − 1) 

Mohammad Abdul Halim, Assistant Professor in Mathematics


Page 144 of 249

 
 

( x )
1 1
= R . P. of  e ix


( )
i − 1 
 1 ( 2i + 1) D  

1+  D2 + 
  ( i − 1) ( i − 1)  

   ( + )  
−1

1  1 2 i 1 
= R . P. of  eix 1 +  D2 + D   ( x ) 
 ( i − 1)   ( i − 1) ( i − 1)   
 
  
1 ix   1 ( 2i + 1)   1 ( 2i + 1) 
2

= R . P. of  e 1 −  D + D+ D + D −  ( x )
2 2

 ( i − 1)   ( i − 1) ( i − 1)   ( i − 1) ( i − 1)   
 
 1 ix  ( 2i + 1)   1 
= R . P. of  e x −   = R . P . of  e ix
 ( i − 1 ) x − ( 2i + 1)
 ( i − 1)  ( i − 1)   ( i − 1)
2


( cos x + i sin x )ix − x − 2i − 1
1
= R . P. of 
 −1 + 1 − 2i 
 1 
= R . P. of  − ( cos x + i sin x ) i ( x − 2 ) − ( x + 1)
 2i 
i 
= R . P. of  i ( x − 2 ) cos x − ( x + 1) cos x − ( x − 2 ) sin x − i ( x + 1) sin x
2 
= − ( x − 2 ) cos x + ( x + 1) sin x = ( 2 − x ) cos x + ( x + 1) sin x
1 1
2 2
Therefore, the general solution of equation is,
y = yc + y p = c1 + c2e − x +
1
2
( 2 − x ) cos x + ( x + 1) sin x where, c1 , c2 are arbitrary constants.

Problem-25: Solve D y − y = x sin x .


4

Solution: Given that, D y − y = x sin x


4

(D 4
− 1) y = x sin x
The auxiliary equation is,
D4 −1 = 0
 ( D 2 − 1)( D 2 + 1) = 0
 ( D + 1)( D − 1) ( D 2 − i 2 ) = 0
 ( D + 1)( D −1)( D + i )( D − i ) = 0
 D = −1, D = 1, D = i, D = −i
The complementary function is,
yc = c1e − x + c2 e x + c3 cos x + c4 sin x
The particular integral is,

Mohammad Abdul Halim, Assistant Professor in Mathematics


Page 145 of 249

1
yp = ( x sin x )
D −1
4

 1   ix 
= Imaginary Part of  4 ( xeix )  = I . P. of
1
e ( x ) 
 −   ( D + i ) − 1 
4
D 1
 ix 1 
= I . P. of e D 4 + 4 D 3i + 6 D 2i 2 + 4 Di 3 + i 4 − 1 ( x ) 
 ix 1 
= I . P. of e D 4 + 4 D 3i − 6 D 2 − 4 Di + 1 − 1 ( x ) 
 ix 1 
= I . P. of  e D 4 + 4 D 3i − 6 D 2 − 4 Di ( x ) 
 
 ix 
= I . P. of − e 1
( x )
 4 Di   1 3 3 i   
1 −  D + D + D  
2
 
   4i 2  
 ieix   1 3 3i  
−1

= I . P. of   
1 − D + D 2
+ D  ( x ) 
 4 D   4i 2  
 ieix  1 3 3i   1 3 3i 
2
 

= I . P. of  + + + + + +  +  ( x )
2 2
 
1 D D D   D D D
 4 D 
  4i 2   4 i 2   

 ieix  3i    ieix  x 2 3xi 
= I . P. of   x +   = I . P. of   + 
 4D  2   4  2 2 
i  x 2 3xi  
= I . P. of  ( cos x + i sin x )  + 
4  2 2 
1  x 2 3xi  
= I . P. of  ( i cos x − sin x )  + 
4  2 2 
 1  x2 3x x2 3xi 
= I . P. of   i cos x − cos x − sin x − sin x 
4  2 2 2 2 
1  x2 3x 
=  cos x − sin x 
4 2 2 
Therefore, the general solution of equation (1) is,
1  x2 3x 
y = yc + y p = c1e− x + c2e x + c3 cos x + c4 sin x +  cos x − sin x  where c1 , c2 are
4 2 2 
arbitrary
constants.
Problem-26: Solve D y − y = x cos x .
2 2

Solution: Given that, D y − y = x cos x


2 2

Mohammad Abdul Halim, Assistant Professor in Mathematics


Page 146 of 249

(D 2
− 1) y = x 2 cos x
The auxiliary equation is,

(D 2
− 1) = 0  ( D + 1)( D − 1) = 0
 D = 1, D = − 1
The complementary function is,
yc = c1e x + c2e − x
The particular integral is,
 1   ix 
yp =
1
( x 2 cos x ) = Re al Part of  2 ( x 2 eix )  = R . P. of e
1
( )
x 2

D −1  D −1   ( D + i ) − 1
2 2

 ix 2   ix 2 
e D 2 + 2iD + i 2 − 1 ( x )  = R . P. of e D 2 + 2iD − 1 − 1 ( x ) 
1 1
= R . P. of

 
 ix  
2 
 D 2 + 2iD − 2 ( )  ( )
1 1 1
= R . P. of e x = R . P . of  − e ix
x 2

 2 1 −  1 D 2 + iD  
   
2 
 1 ix   1 2 
−1

= R . P. of  − e 1 −  D + iD   ( x ) 
2

 2   2  
 1 ix   1 2  1 2 
2
 2 

= R . P. of  − e 1 +  D + iD  +  D + iD  +  ( x )
 2   2  2  
 
 1   1 
= R . P. of  − eix  x 2 + (1 + 2ix ) − 2 = R . P. of  − ( cos x + i sin x )  x 2 + 1 + 2ix − 2
 2   2 
 1
 2

= R . P. of  − ( cos x + i sin x ) ( x 2 − 1) + 2ix 


 1
 2
 
= R . P. of  − ( x 2 − 1) cos x + 2ix cos x + i ( x 2 − 1) sin x − 2 x sin x 


1
2
 1
2

= 2 x sin x − ( x 2 − 1) cos x = x sin x − ( x 2 − 1) cos x
Therefore, the general solution of equation is,
y = yc + y p = c1e x + c2e − x + x sin x −
2
( x − 1) cos x where, c1 , c2 are arbitrary constants.
1 2

Problem-27: Solve D y − y = xe sin x .


2 x

Solution: Given that, D y − y = xe sin x


2 x

(D 2
− 1) y = xe x sin x
The auxiliary equation is,

 ( D 2 − 1) = 0

Mohammad Abdul Halim, Assistant Professor in Mathematics


Page 147 of 249

 ( D + 1)( D − 1) = 0
 D = 1, D = −1
The complementary function is,
yc = c1e x + c2e − x
The particular integral is,
yp = 2
1
D −1
(
x 2 e x sin x )
y p = ex
1
(
x 2 sin x = e x 2 ) 1
x 2 sin x ( )
( D + 1) − 1 + + −
2
D 2 D 1 1
 
= ex
1
D + 2D 2 ( x 2 sin x ) = Imaginary Part of e x 2
 D + 2D
1
( x 2eix ) 

   1+i x 
= I . P. of e x eix
1
x 2
( )
 = I . P. of e( ) 2
1
( x 2 )
( D + i ) + 2 ( D + i )   D + 2 Di + i + 2 D + 2i 
2 2

 
= I . P. of e x eix 2
1
D + 2 (1 + i ) D − 1 + 2i
( x 2 )
 
 1 
= I . P. of  e x eix
1
x 2
 ( )
 ( 2i − 1) 
1 + D 2 + 2 (1 + i ) D 
 ( 2i + 1) x ix 
  ( )
−1
= I . P. of  − e e 1 + D 2 + 2 (1 + i ) D  x 2 
 5 
 ( 2i + 1) (1+i ) x   x2 
( )   ( )  ( )
2
= I . P. of  − e 1 − D 2
+ 2 1 + i D + D 2
+ 2 1 + i D −  
 5 
 ( 2i + 1) x 2 
= I . P. of  − e ( cos x + i sin x )  x 2 − 2 + 4 (1 + i ) x + 8 (1 + i )  
 
 5 
 ( 2i + 1) x 
= I . P. of  − e ( cos x + i sin x )  x 2 − 2 − 4 x − 4ix + 16i
 5 
 ( 2i + 1) x 
= I . P. of  − 
e ( cos x + i sin x ) ( x 2 − 4 x − 2 ) − 4i ( x − 4 ) 
 5 
 ( 2i + 1) x 
= I . P. of  − 
e ( cos x + i sin x ) ( x 2 − 4 x − 2 ) − 4i ( x − 4 )  
 5 
 ( 2i + 1) x 2 
= I . P. of −  
e ( x − 4 x − 2 ) cos x − 4i ( x − 4 ) cos x + i ( x 2 − 4 x − 2 ) sin x + 4 ( x − 4 ) sin x 
 5 
 ( 2i + 1) x 2 
= I . P. of  −  
e ( x − 4 x − 2 ) cos x − 4i ( x − 4 ) cos x + i ( x 2 − 4 x − 2 ) sin x + 4 ( x − 4 ) sin x 
 5 
( 2i + 1) e x

5
2 ( x − 4x − 2) cos x − 4i ( x − 4) cos x + i ( x − 4x − 2) sin x + 4 ( x − 4) sin x
2 2

Mohammad Abdul Halim, Assistant Professor in Mathematics


Page 148 of 249

Therefore, the general solution of equation is,


y = yc + y p = c1e x + c2e − x + x sin x −
2
( x − 1) cos x Where, c1 , c2 are arbitrary constants.
1 2

Supplementary Problems
1. Solve the following ODEs
No ODEs Answers
−x −4 x
1. 2
d y dy y = c1 e + c2 e
+5 + 4y = 0
dx 2 dx
2. 2
d y dy y = c1 e3 x + c2 e 4 x
2
− 7 + 12 y = 0
dx dx
3. d2y dy y = c1 e − a x + c2 e −b x
2
+ ( a + b ) + aby = 0
dx dx
4. d3y dy y = c1 e − x + c2 e −3 x + c3 e 4 x
3
− 13 − 12 y = 0
dx dx
5. 3
d y d2y dy y = c1e x + c2 e 2 x + c3e3 x
3
− 6 2
+ 11 − 6 y = 0
dx dx dx
6. d3y dy y = c1e − x + c2 e −3 x + c3e 4 x
3
− 13 − 12 y = 0
dx dx

2. Solve the Differential equations:


No ODEs Answers
1. 3
d y 2
d y dy y = ( c1 + c2 x ) e + c3 e −2 x
2x
−2 −4 + 8y = 0
dx3 dx 2 dx
2. D y − 4D y + 5Dy − 2 y = 0
3 2
y = ( c1 + c2 x ) e x + c3 e 2 x
3.
D + ( 2
3
) ( )
3 − 1 D2 + 3 − 2 3 D − 3 y = 0  y = c1 e x + ( c2 + c3 x ) e − 3x

4. d 3 y d 2 y dy y = ( c1 + c2 x ) e x + c3 e − x
− − +y=0
dx3 dx 2 dx
5. ( D − 1) ( D − 4)3 y = 0 y = ( c1 + c2 x ) e x + ( c3 + c4 x + c5 x 2 ) e 2 x
2

6. (D 4
− 2 D 3 − 3D 2 + 4 D + 4 ) y = 0 y = ( c1 + c2 x ) e2 x + ( c3 + c4 x ) e− x
7. (D 4
+ 2 D3 + D 2 ) y = 0 y = ( c1 + c2 x ) + ( c3 + c4 x ) e− x
8. d2y dy y = ( c1 + c2 x ) e2 x
2
− 4 + 4y = 0
dx dx
9. 4
d y d y 3
d2y dy y = ( c1 + c2 x + c3 x 2 ) e − x + c4e 4 x
4
− 3 − 9 2 − 11 − 4 y = 0
dx dx dx dx

3. Solve the following differential equations:


No ODEs Answers
1. (D + 5D + 6 y = 0
4 2
) ( ) (
y = c1 cos 3x + c2 sin 3x + c3 cos 2 x + c4 sin 2 x )

Mohammad Abdul Halim, Assistant Professor in Mathematics


Page 149 of 249

m
 m  −
m
2. d4y m  m m 
+ m4 y = 0 y=e 2
  c1 cos x + c2 sin x+e 2
 c3 cos x + c4 sin x
dx 4  2 2   2 2 
3. d4y y = c1eax + c1e− ax + ( c3 cos ax + c4 sin ax )
4
− a4 y = 0
dx
4. ( D 2 − 4 D + 1) y = 0 y = c1e
( 2+ 3 ) x
+ c1e
( 2− 3 ) x
5. (D 4
+ 8 D 2 + 16 ) y = 0 y = ( c1 + c2 x ) cos 2x + ( c3 + c4 x ) sin 2 x

(D + 1) ( D 2 + D + 1) y = 0
6. 2 x
2 − 3 3
y = ( c1 + c2 x ) cos x + ( c3 + c4 x ) sin x  + e 2 (c5 cos x + c6 sin x)
2 2
7. (D 4
+ D 2 + 1) y = 0 −
x
y = e 2 (c1 cos
3
x + c2 sin
3 x
x) + e 2 (c3 cos
3
x + c4 sin
3
x)
2 2 2 2
8. d2y dy
2
− 2 + 2y = 0 𝑦 = 𝑒 𝑥 (𝐴𝑐𝑜𝑠𝑥 + 𝐵𝑠𝑖𝑛𝑥)
dx dx
4. Solve the ODEs:
No ODEs Answers
1. 2
d y dy y = e ( c1 cos 2 x + c2 sin 2 x ) + ( 2sin x + cos x )
x

2
−2 + 5 y = 10sin x
dx dx
2. 2 1
d y
+ y = cos 2 x y = ( c1 cos x + c2 sin x ) − cos 2 x
dx 2 3
−  3  1
3. d 2 y dy x
3
+ + y = sin 2 x y = e 2  c1 cos x + c2 sin x  − ( 2cos 2 x + 3sin 2 x )
dx 2 dx 2 2
  13
(D + 1) y = cos 3x
4. 2 1
y = ( c1 + c2 x ) cos x + ( c3 + c4 x ) sin x +
2
cos 3x
64
5. (D 3
+ D 2 + D + 1) y = sin 2 x y = c1e − x + ( c2 cos x + c3 sin x ) +
1
( 2 cos 2 x − sin 2 x )
15
6. d4y d2y
+ 5 2 + 6y = 0 y = c1e 2ix
+ c2e− 2ix
+ c3e 3ix
+ c4e− 3ix

dx 4 dx
7. 4 3
d y d y dy  3 −
x
3 
− − +y=0 y = ( c1 + c2 x ) e + e c3 cos
x
x + c4 sin
2
x
dx 4 dx3 dx 2 2 

8. d4y d2y y = ( A + Bx ) cos x + (C + Dx ) sin x
+ 2 +y=0
dx 4 dx 2
9. d6y d4y d2y y = ( c1 + c2 x + c3 x 2 ) cos x + ( c4 + c5 x + c6 x 2 ) sin x
+3 4 +3 2 + y = 0
dx 6 dx dx
10. D y − 4D y + 4 y = 0
4 2
y = (c1 + c2 x)e 2x
+ (c3 + c4 x)e − 2x

11. D 4 y − 81y = 0 y = c1e 3 x + c1e −3 x + c3 cos 3x + c4 sin 3x


5. Solve the ODEs:
No ODEs Answers
1. D y − 2Dy + y = x 2
2
y = ( c1 + c2 x ) e + x + 4 x + 6
x 2

Mohammad Abdul Halim, Assistant Professor in Mathematics


Page 150 of 249

2. D2 y + 4 y = x2 + 3 x2 5
y = c1 cos 2 x + c2 sin 2 x + +
4 8
3. D 2 y + Dy − 2 y = 2 (1 + x − x 2 ) y = c1e x + c2 e −2 x + x 2
6. Solve the ODEs:
N ODEs Answers
o
1. D 2 y − 3Dy + 2 y = e3 x 1
y = c1e x + c2e 2 x + e3 x
2
D3 y − Dy = e x + e− x
( e + e− x )
2. x x
y = c1 + c2 e x + c3e − x +
2
3. D 2 y + 2 Dy + 2 y = 2e− x y = ( c1 cos x + c2 sin x ) e− x + 2e− x
4. D2 y + 4Dy + 4 y = e2 x + e−2 x e2 x 1 2 −2 x
y = ( c1 + c2 x ) e −2 x + + xe
16 2
7. Solve the ODEs:
1. D y + 3Dy + 2 y = cos 2 x
2 1
y = c1e − x + c2e −2 x + ( 3sin 2 x − cos 2 x )
20

2. D y − 5Dy + 6 y = 100sin 4 x
2
y = c1e3 x + c2e 2 x + 4 cos 4 x − 2sin 4 x

3. D y + 4 y = sin 2 x
2 x
y = c1 cos 2 x + c2 sin 2 x − cos 2 x
4

4. D y + 4 y = cos 2 x
2 x
y = c1 cos 2 x + c2 sin 2 x + sin 2 x
4

5. D y + y = sin x
2 x
y = c1 cos x + c2 sin x − cos x
2

6. D y + y = cos x
2 2 1 1
y = c1 cos x + c2 sin x + − cos 2 x
2 6

7. D y − 5Dy + 6 y = sin x + cos x


2 1
y = c1e3 x + c2 e 2 x + cos x
5

8. D y − y = sin ( 3x + 1)   −x
y = c1e x + c2 cos  3x  + c3 sin  3x   e 2
3

  2   2 
730 
+1 27 cos ( 3x + 1) − sin ( 3x + 1)

1
8. Solve the following ODE D 2 y + 4 y = x sin x Ans: y = c1 cos 2 x + c2 sin 2 x + ( 3x sin x − 2 cos x )
9

Mohammad Abdul Halim, Assistant Professor in Mathematics


Page 151 of 249

MCQs
1. What is the true for 𝐷 2 𝑦 + 4𝑦 = 𝑠𝑖𝑛𝑥 ?
a. Order 1,ODE b. Order 2 ,PDE c. Order 2,ODE d. linear PDE
2. What is the Auxiliary equation of D2 y + 4 y = sin x ?
a. D 2 + 4 b. m 2 + 4 c. m 2 − 4 d. a and b
3. What is the complementary function of D y − 4 y = sin x ?
2

a. 𝑦 = 𝑐1 𝑒 2𝑥 + 𝑐2 𝑒 −2𝑥 b. 𝑦 = 𝑐1 𝑒 2𝑥 + 𝑐2 𝑒 2𝑥 c. 𝑦 = (𝑐1 + 𝑥𝑐2 )𝑒 −2𝑥 d. 𝑦 = (𝑐1 + 𝑥𝑐2 )𝑒 2𝑥


4. Particular Integral of D2 y − 4 y = sin x is:
𝑠𝑖𝑛𝑥 𝑠𝑖𝑛5𝑥 𝑠𝑖𝑛𝑥 𝑐𝑜𝑠𝑥
a. 5
b. 5
c.− 5
d.− 5
5. Solution of D2 y − 4 y = sin x is:
𝑠𝑖𝑛5𝑥 𝑠𝑖𝑛5𝑥
a. 𝑦 = 𝑐1 𝑒 2𝑥 + 𝑐2 𝑒 −2𝑥 − 5
b. 𝑦 = 𝑐1 𝑒 2𝑥 + 𝑐2 𝑒 2𝑥 − 5
𝑐𝑜𝑠5𝑥 𝑠𝑖𝑛5𝑥
𝑐. 𝑦 = 𝑐1 𝑒 2𝑥 + 𝑐2 𝑒 −2𝑥 − 5
d. 𝑦 = (𝑐1 + 𝑥𝑐2 )𝑒 −2𝑥 − 5

Linear ODE with first degree and higher order with variable coefficients
In this chapter we discuss about the solution of linear higher order ordinary differential equations with
variable coefficients by using particular integrals and complementary functions but firstly converting it into
constant coefficients.
General solution (G.S) = Complementary Function (C.F) + Particular Integral (P.I)
Chapter Contents:
➢ Linear higher order ordinary differential equation with variable coefficients
➢ Solution of Homogeneous linear Equations
Learning Outcomes: Learners be able to know...
➢ How to determine the general solution of homogeneous linear DE
➢ How to solve the homogeneous linear differential equations reducing it to constant
coefficients form
Linear Differential Equations with variables Coefficients
n −1
dny n −1 d y
An equation of the form x n
n
+ P1 x n −1
+ + Pn y = Q where, P1 , P2 , Pn are
dx dx
constants and Q is function of x or constant, is called the linear differential equation with variables
coefficients or homogeneous linear equations.
Technique of conversion: If we put x = e or , z = ln x , then the above equation is transformed into an
z

equation with constant coefficients changing the independent variable from x to z as,
dz 1
=
dx x
dy dy dz dy 1 dy dy dy dy d
Now = .  = x = x = Dy as D =
dx dz dx dx x dz dx dz dx dz
d y d  dy 
2
d y d  1 dy 
2 2
d y 1 dy 1 d  dy 
Again, =   2 =    2 =− 2 +  
dx 2
dx  dx  dx dx  x dz  dx x dz x dx  dz 
d2y 1 dy 1 d 2 y dz d2y 1 dy 1 d 2 y 1 d2y 1 dy 1 d 2 y
 2 =− 2 + .  = − + .  = − +
dx x dz x dz 2 dx dx 2 x 2 dz x dz 2 x dx 2 x 2 dz x 2 dz 2

Mohammad Abdul Halim, Assistant Professor in Mathematics


Page 152 of 249

d 2 y d 2 y dy 2
2 d y d
 x2 2
= 2
−  x 2
= D ( D − 1) y as D =
dx dz dz dx dz
Similarly,
d3y
x3 = D ( D − 1)( D − 2 ) y
dx3
… … ……………………………
dny
x n
= D ( D − 1)( D − 2 ) ( D − n + 1) y
dx n

From we get,
 D ( D − 1)( D − 2 ) ( D − n + 1) + P1 D ( D − 1)( D − 2 ) ( D − n + 2 ) + + Pn y = Q
Then the above equation be in a linear differential equation with constant coefficients.
Mathematical Problems
d2y dy
Problem-01: Solve x 2 2
+ 9 x + 25 y = 0
dx dx
2
d y dy
Solution: Given that, x 2 2 + 9 x + 25 y = 0 (1)
dx dx
d
Putting x = e z and D = in equation (1) we get,
dz
D ( D −1) y + 9Dy + 25 y = 0
 D2 y − Dy + 9Dy + 25 y = 0
 D2 y + 8Dy + 25 y = 0
 ( D 2 + 8 D + 25 ) y = 0 ( 2)
Then the auxiliary equation of (2) is,
D 2 + 8D + 25 = 0
−8  64 − 100 −8  −36 −8  6i
 D= = = = −4  3i
2 2 2
 D = −4  3i
The general solution of (1) is,
y = ( c1 cos3z + c2 sin 3z ) e−4 z = c1 cos ( 3ln x ) + c2 sin ( 3ln x )  x
−4

1
= 4 c1 cos ( 3ln x ) + c2 sin ( 3ln x )  where c1 , c2 are arbitrary constants.
x
d2y
Problem-02: Solve x 2 2 + y = 3x 2
dx
d2y
Solution: Given that, x 2 2 + y = 3x 2 (1)
dx
d
Putting x = e z and D = in equation (1) we get,
dz
D ( D − 1) y + y = 3e2 z

Mohammad Abdul Halim, Assistant Professor in Mathematics


Page 153 of 249

 D 2 y − Dy + y = 3e2 z
 ( D 2 − D + 1) y = 3e 2 z ( 2)
Then the auxiliary equation of (2) is,
D2 − D + 1 = 0
1  1 − 4 1  −3 1  3i
 D= = =
2 2 2
1 3i
 D= 
2 2
The complementary function of (1) is,
  3   3  z 2   3   3   ln x 12
yc = c1 cos  z  + c2 sin  z   e = c1 cos  ln x  + c2 sin  ln x   e
  2   2     2   2  
  3   3 
= x c1 cos  ln x  + c2 sin  ln x  
  2   2  
The particular integral of (1) is,
yp =
1
D − D +1
2 ( 3e2 z )
= 2
1
2 − 2 +1
( 3e 2 z )

= e2 z
= x2
Therefore the general solution is,
  3   3 
y = yc + y p = x c1 cos  ln x  + c2 sin  ln x   + x 2 where c1 , c2 are arbitrary
  2   2  
constants.
d2y dy
Problem-03: Solve x 2 2
− 2x − 4 y = x4
dx dx
2
d y dy
Solution: Given that, x 2 2
− 2x − 4 y = x4 (1)
dx dx
d
Putting x = e z and D = in equation (1) we get,
dz
D ( D − 1) y − 2Dy − 4 y = e4 z
 D 2 y − 3Dy − 4 y = e4 z
 ( D 2 − 3D − 4 ) y = e 4 z ( 2)
Then the auxiliary equation of (2) is,
D 2 − 3D − 4 = 0
 D2 − 4D + D − 4 = 0
 ( D − 4) + ( D − 4) = 0

Mohammad Abdul Halim, Assistant Professor in Mathematics


Page 154 of 249

 ( D + 1)( D − 4) = 0
 D = −1, D = 4
The complementary function of (1) is,
c1
yc = c1e − z + c2e 4 z = c1 x −1 + c2 x 4 = + c2 x 4
x
The particular integral of (1) is,
yp =
1
D − 3D − 4
2 ( e4 z ) =
z
2D − 3
( e4 z ) =
z
2.4 − 3
( e 4 z ) = ( e 4 z ) = x 4 ln x
z
5
1
5
Therefore the general solution is,
c1 1
y = yc + y p = + c2 x 4 + x 4 ln x where c1 , c2 are arbitrary constants.
x 5

d2y dy
Problem-04: Solve x 2
2
− 3x + 4 y = 2 x 2
dx dx
2
d y dy
Solution: Given that, x 2 2 − 3x + 4 y = 2x2 (1)
dx dx
d
Putting x = e z and D = in equation (1) we get,
dz
D ( D − 1) y − 3Dy + 4 y = 2e2 z
 D2 y − 4Dy + 4 y = 2e2 z
( D 2
− 4 D + 4 ) y = 2e 2 z ( 2)
Then the auxiliary equation of (2) is,
D2 − 4D + 4 = 0
 ( D − 2) = 0
2

 D = 2, D = 2
The complementary function of (1) is,
yc = ( c1 + c2 z ) e2 z
= x2 ( c1 + c2 ln x )
The particular integral of (1) is,
z2
yp =
1
D − 4D + 4
2 ( 2 e 2z
) =
z
2D − 4
( 2 e 2z
) =
2
( )
2e2 z = z 2 e 2 z = ( ln x ) x 2
2

Therefore the general solution is,


y = yc + y p = x 2 ( c1 + c2 ln x ) + ( ln x ) x 2 where c1 , c2 are arbitrary constants.
2

d2y dy
Problem-05: Solve x 2 2
− x − 3 y = x 2 ln x
dx dx
2
d y dy
Solution: Given that, x 2 2 − x − 3 y = x 2 ln x (1)
dx dx
d
Putting x = e z and D = in equation (1) we get,
dz

Mohammad Abdul Halim, Assistant Professor in Mathematics


Page 155 of 249

D ( D −1) y − Dy − 3 y = ze2 z
 D2 y − 2Dy − 3 y = ze2 z
 ( D 2 − 2 D − 3) y = ze 2 z ( 2)
Then the auxiliary equation of (2) is,
D2 − 2D − 3 = 0
 D 2 − 3D + D − 3 = 0
 D ( D − 3) + ( D − 3) = 0
 ( D + 1)( D − 3) = 0
 D = −1, D = 3
The complementary function of (1) is,
yc = c1e − z + c2e3 z = c x −1 + c x 3 = c1 + c x 3
1 2 2
x

The particular integral of (1) is,


yp =
1
( ze 2 z ) = e 2 z
1
(z)
D − 2D − 3 ( D + 2) − 2 ( D + 2) − 3
2 2

1 1
= e2 z ( z ) = e2 z 2 ( z)
D + 4D + 4 − 2D − 4 − 3
2
D + 2D − 3
−1
e2 z 1 e2 z   D2 2 
=− ( )
z = − 1 −  + D  ( z)
3   D2 2  3   3 3 
1 −  + D 
  3 3 
e2 z   D 2 2   D 2 2  
2

=− 1 +  + D + + D +  ( z)
3   3 3   3 3  

e2 z   D 2 2   D 2 2  
2

=− z +  + D z + + D z + 
3   3 3   3 3  

e  
2z
2  e 
2z
2 x 
2
2
=−  z +  0 +  + 0 = −  z +  = −  ln x + 
3   3  3  3 3 3
Therefore the general solution is,
c1 x2  2
y = yc + y p = + c2 x3 −  ln x +  where, c1 , c2 are arbitrary constants.
x 3 3
2
d y dy
Problem-06: Solve x 2 2 + 4 x + 2 y = x + sin x
dx dx
2
2 d y dy
Solution: Given that, x 2
+ 4 x + 2 y = x + sin x (1)
dx dx
d
Putting x = e z and D = in equation (1) we get,
dz
D ( D − 1) y + 4Dy + 2 y = e z + sin e z

Mohammad Abdul Halim, Assistant Professor in Mathematics


Page 156 of 249

 D 2 y + 3Dy + 2 y = e z + sin e z
 ( D 2 + 3D + 2 ) y = e z + sin e z ( 2)
Then the auxiliary equation of (2) is,
D 2 + 3D + 2 = 0
 D2 + 2D + D + 2 = 0
 D ( D + 2) + ( D + 2) = 0
 ( D + 1)( D + 2 ) = 0
 D = −1, D = −2
The complementary function of (1) is,
c1 c2
yc = c1e − z + c2 e −2 z = c1 x −1 + c2 x −2 = +
x x2
The particular integral of (1) is,
yp =
1
D + 3D + 2
2 ( e z + sin e z )

= 2
1
D + 3D + 2
( ez ) + 2
1
D + 3D + 2
( sin e z )

= 2
1
1 + 3.1 + 2
ez + ( ) 1
( D + 2 )( D + 1)
sin e z ( )
ez
= +
1
6 ( D + 2 )( D + 1)
( sin e z )

Now let,
1
( D + 1)
( sin e z ) = u

 ( D + 1) u = sin e z
du
 + u = sin e z which is linear equation
dz
Therefore, I .F = e  = e z
dz

du
 ez + e z u = e z sin e z
dt
or ,
d z
dt
( e u ) = e z sin e z
Integrating,
e z u =  e z sin e z dz
= − cos e z
 u = −e− z cos e z
Again,
1
( D + 2)( D + 1)
( sin e z ) =
1
u=
1
( D + 2) ( D + 2)
( −e− z cos e z ) = v ( say )


1
( D + 2)
( −e− z cos e z ) = v

Mohammad Abdul Halim, Assistant Professor in Mathematics


Page 157 of 249

or, ( D + 2) v = −e− z cos e z


dv
or , + 2v = −e − z cos e z which is also a linear equation
dz
Therefore, I .F = e = e2 z
2 dz

dv
 e2 z + 2ve 2 z = −e z cos e z
dz
or ,
d
dz
( ve2 z ) = −e z cos e z

Integrating,
ve 2 z = −  e z cos e z dz
= − sin e z
1
 v = − 2 z sin e z
e
1
= − 2 sin x
x
Therefore the general solution is,
y = yc + y p
c1 c2 1
= + − sin x
x x2 x2
where, c1 , c2 are arbitrary constants.
d2y dy
Problem-07: Solve x 2
2
+ 4x + 2 y = ex
dx dx
2
d y dy
Solution: Given that, x 2 2 + 4 x + 2 y = ex (1)
dx dx
d
Putting x = e z and D = in equation (1) we get,
dz
D ( D − 1) y + 4 Dy + 2 y = ee
z

 D 2 y + 3Dy + 2 y = ee ( 2)
z

Then the auxiliary equation of (2) is,


D 2 + 3D + 2 = 0
 D2 + 2D + D + 2 = 0
 D ( D + 2) + ( D + 2) = 0
 ( D + 1)( D + 2 ) = 0
 D = −1, D = −2
The complementary function of (1) is,
yc = c1e − z + c2 e −2 z
= c1 x −1 + c2 x −2

Mohammad Abdul Halim, Assistant Professor in Mathematics


Page 158 of 249

c1 c2
= +
x x2
The particular integral of (1) is,
yp =
1
D + 3D + 2
2
ee
z

( )
=
1
( D + 1)( D + 2 )
ee( )
z

=
 1

1  ez
 D + 1 D + 2 
e ( )
=
1
D +1
ee −
z

( )
1
D+2
ee
z

( )
Let ,
1
D +1
( )
ee = u
z

or , ( D + 1) u = ( e ) ez

du
+ u = e e which is linear equation
z
or ,
dz
Therefore, I .F = e  = e z
dz

du
 ez + e z u = e z .e e
z

dt
d z
( e u ) = e z .e e
z
or ,
dz
Integrating,
e z u =  e z .ee dz ; as e z = v
z

= ev
= ee
z

= ex
 u = e− z .e x
= x −1.e x
ex
=
x
Again,
1
( D + 2)
z

( )
ee = w ( say )

or , ( D + 2 ) w = ee
t

dw
+ 2 w = e e which is also a linear equation
z
or ,
dz
Therefore, I .F = e = e2 z
2 dz

dw
 e2 z + 2 we 2 z = e 2 z .e e
z

dz

Mohammad Abdul Halim, Assistant Professor in Mathematics


Page 159 of 249

d
( we 2 z ) = e 2 z .e e
z
or ,
dt
Integrating,
we2t =  e2 z .ee dz
z

=  e z .e z .ee dz
z

=  ze z dz ; as et = z
= ze z − e z
= xe x − e x
 v = e −2 z ( xe x − e x )
= x −2 ( xe x − e x )
ex ex
− =
x x2
ex  ex ex 
 P.I = −  − 2 
x  x x 
ex
= 2
x
Therefore the general solution is,
c1 c2 e x
y = yc + y p = + + where, c1 , c2 are arbitrary constants.
x x2 x2
Supplementary Problems
Solve the following ODEs:
d2y dy y = c1 x −1 + c2 x 2
1x 2
2
+ 2x − 2 y = 0
dx dx
2
d y dy y = c1 x 3 + c2 x −3
2. x 2 2
+ x −9y = 0
dx dx
2 1 4
3. x 2
d y dy
+ 5x + 4 y = x4 y = ( c1 + c2 ln x ) x −2 + x
dx 2
dx 36
2
d y dy y = ( c1 + c2 ln x ) x + 2ln x
4. x 2 2 − x + y = 2 ln x
dx dx
MCQs
d2y dy
1. What is the constant coefficient form of x 2 2
+ 2x + 2 y = 0 ?
dx dx
a. (𝐷 2 + 3𝐷 − 2)𝑦 = 0 b. (𝐷 2 + 3𝐷 − 1)𝑧 = 0
𝑐. (𝐷 2 + 𝐷 + 2)𝑦 = 0 d. (𝐷 2 + 𝐷 − 1)𝑧 = 0
d2y dy
2. What is the complementary function of x 2 2
+ 2x − 2 y = 0 ?
dx dx
1 1
√7 √7 √7 √7
a. 𝑦𝑐 = 𝑒 −2𝑧 (𝑐1 𝑐𝑜𝑠 2
𝑧 + 𝑐2 𝑠𝑖𝑛 2 𝑧) b. 𝑦𝑐 = 𝑒 −2𝑙𝑛𝑥 (𝑐1 𝑐𝑜𝑠 2
𝑙𝑛𝑥 + 𝑐2 𝑠𝑖𝑛 2
𝑙𝑛𝑥)
1
√7 √7
c. 𝑦𝑐 = 𝑒 −2𝑧 (𝑐1 𝑐𝑜𝑠 2 𝑧 − 𝑐2 𝑠𝑖𝑛 2 𝑧) d. a and b

Mohammad Abdul Halim, Assistant Professor in Mathematics


Page 160 of 249

3. How to solve the variable coefficients ODEs:


a. reducing it to constant coefficient form b. multiplying by integrating factor
c. Dividing by x d. dividing by y
d2y dy
4. What are the root of auxiliary equation of x 2 2
+ 2x + 2 y = 0 ?
dx dx
−1±𝑖√7 1±𝑖√7 −1±𝑖√5 1±𝑖√5
a. b. c. d.
2 2 2 2
5𝑥 𝑑2 𝑦 𝑑𝑦
5. If 𝑦 = 𝑒 is thesolution to 𝑑𝑥 2 − 7 𝑑𝑥 + 𝑚𝑦 = 0 then what is the value of ‘m’ ?
a. 2 b. 5 c. 10 d. 1

Mohammad Abdul Halim, Assistant Professor in Mathematics


Page 161 of 249

Mohammad Abdul Halim, Assistant Professor in Mathematics


Page 162 of 249

Introduction:
The Laplace Transform is a powerful Mathematical tool that is very useful in Electrical
Engineering. It is an operation that transforms a function of t (i.e., a function of time domain-
signal), defined on [0, ∞), to a function of s (i.e., of frequency domain-Laplace transform). The
Laplace transform provides a useful method of solving certain types of differential equations and
integral equations when certain initial and Boundary conditions are given, especially when the
initial values are zero. It is also very useful in the area of circuit analysis. It is often easier to
analyse the circuit in its Laplace form (By transfer function), than to form differential equations.
The techniques of Laplace transform are not only used in circuit analysis, but also in
• Proportional-Integral-Derivative (PID) controllers
• DC motor speed control systems
• DC motor position control systems
• Second order systems of differential equations (underdamped, over-damped and critically
damped)
• Laplace Transform is used to handle piecewise continuous or impulsive force.
For example:
For the design of a control system, it is important to know how the system of interest behaves and
how it responds to different controller designs. To do this, the dynamic equations of the system
are obtained and are solved to get the dynamic response. There are three different domains within
which the dynamic response of a system is studied for the purpose of control design. These are the
Laplace domain, the frequency domain and the state-space. This module provides an introduction
to the Laplace domain and covers the mathematics of the Laplace transform.
Definition: Let 𝑭(𝑡) be a function of 𝑡 specified for 𝑡 ≥ 0. Then the Laplace transform of𝑭(𝑡),
denoted byℒ{𝑭(𝑡)}, and is defined by

ℒ{𝑭(𝑡)} = 𝑓(𝑠) = ∫ 𝑒 −𝑠𝑡 𝑭(𝑡)𝑑𝑡


0

where we assume at present that the parameter 𝑠 is real. Later it will be found useful to consider
𝑠 complex.
The Laplace transform of 𝑭(𝑡) is said to exist if the integral (1) converges for some value of 𝑠;
otherwise it does not exist. For sufficient condition under which the Laplace Transform does
exist.

L F (t )

F(t) f(s)

L −1  f (s)

Mohammad Abdul Halim, Assistant Professor in Mathematics


Page 163 of 249

Notation: If a function of t is indicated in terms of a capital letter, such as 𝑭(𝑡), 𝑮(𝑡), 𝒀(𝑡), etc.,
the Laplace transform of the function is denoted by the corresponding lower case letter, i.e.
𝑓(𝑠), 𝑔(𝑠), 𝑦(𝑠), etc.
Laplace operator: The symbol L which transforms 𝑭(𝑡) into 𝑓(𝑠) is often called the Laplace
transformation operator and L is a linear operator. Laplace transform can only be used to transform
variables that cover a range from “zero (0)” to infinity, (∞), that is: 0 < t < ∞. Any variable that
does not vary within this range cannot be transformed using Laplace transform. Because time
variable t is most common variable that varies from (0 to ∞), functions with variable t are
commonly transformed by Laplace transform.
Section-ally continuous function:
A function f ( x ) is called section-ally continuous or Piecewise continuous if any interval  a, b if
it is continuous and has finite left and right hand limits in every subinterval  ai , bi  , i = 1, 2,3,....., n
as shown in the following figure.

Function of exponential order:


A function f ( x ) is said to be of exponential order ‘a’ as x →  if lim e− a x f ( x) = finite quantity
x→

.Sometimes we write it as f ( x) = O ( e ax
) ,x →.
Application of Laplace Transformation:
1. Laplace transformation turns differential equation into an algebraic equation.
2. It has also an application in solving the electronic circuit equations.
Beside of these the Laplace transformation technique is applicable in many fields of science
and technology such as:
❖ Control Engineering
❖ Communication
❖ Signal Analysis and Design
❖ System Analysis
❖ Solving Differential Equations
Existence of Laplace transform:
If F (t) is section-ally continuous in every finite interval 0  t  N and of exponential order  for
t  N , then its Laplace transformation f (s) exists for all s   .It must be emphasized that the
stated conditions are sufficient to guarantee the existence of the Laplace transform. If the condition

Mohammad Abdul Halim, Assistant Professor in Mathematics


Page 164 of 249

are not satisfied however the Laplace transform may or may not exist. Thus the conditions are not
necessary for existence of the Laplace transform.
LAPLACE TRANSFORMS OF SOME ELEMENTARY FUNCTIONS
𝑭(𝑡) ℒ{𝑭(𝑡)} = 𝑓(𝑠)
01 1 1
𝑠>0
𝑠
02 𝑡 1
𝑠>0
𝑠2
03 𝑡 𝑛 ; 𝑛 = 0,1,2,3, … 𝑛!
𝑛+1
𝑠>0
𝑠
04 𝑒 𝑎𝑡 1
𝑠>𝑎
𝑠−𝑎
05 sin 𝑎𝑡 𝑎
𝑠>0
𝑠 + 𝑎2
2

06 cos 𝑎𝑡 𝑠
𝑠>0
𝑠 + 𝑎2
2

07 sinh 𝑎𝑡 𝑎
𝑠 > |𝑎|
𝑠 2 − 𝑎2
08 cosh 𝑎𝑡 𝑠
𝑠 > |𝑎|
𝑠 − 𝑎2
2

SOME IMPORTANT PROPERTIES OF LAPLACE TRANSFORMS


Property Example
1. Linearity Property. ℒ{4𝑡 2 − 3 sin 4𝑡 + 4𝑒 −2𝑡 }
If 𝑐1 and 𝑐2 are any constants while 𝐹1 (𝑡) and 𝐹2 (𝑡) are = 4ℒ{𝑡 2 } − 3ℒ{sin 4𝑡} + 4ℒ{𝑒 −2𝑡 }
functions with Laplace transforms 𝑓1 (𝑠) and 2! 4
= 4 ( 3) − 3 ( 2 )
𝑓2 (𝑠)respectively, then 𝑠 𝑠 + 16
ℒ{𝑐1 𝐹1 (𝑡) + 𝑐2 𝐹2 (𝑡)} = 𝑐1 ℒ{𝐹1 (𝑡)} + 𝑐2 ℒ{𝐹2 (𝑡)} 1
+ 4( )
𝑐1 𝑓1 (𝑠) + 𝑐2 𝑓2 (𝑠) 𝑠+2
This result is easily extended to more than two functions.

Mohammad Abdul Halim, Assistant Professor in Mathematics


Page 165 of 249

8 12 4
= 3
− 2 +
𝑠 𝑠 + 16 𝑠 + 2

2. First Translation Or Shifting Property. 4


Since ℒ{sin 4𝑡} = 𝑠2 +16 = 𝑓(𝑠)
If ℒ{𝐹(𝑡)} = 𝑓(𝑠) then
∴ ℒ{𝑒 −2𝑡 sin 4𝑡} = 𝑓(𝑠 + 2)
ℒ{𝑒 𝑎𝑡 𝐹(𝑡)} = 𝑓(𝑠 − 𝑎) 4
=
(𝑠 + 2)2 + 16
4
= 2
𝑠 + 4𝑠 + 20
3. Second Translation Or Shifting Property. 3} 3! 6
Since ℒ{𝑡 = 𝑠4 = 𝑠4 , the Laplace
If ℒ{𝐹(𝑡)} = 𝑓(𝑠) and
transform of the function
𝐹(𝑡 − 𝑎) 𝑡 > 𝑎 3
𝐺(𝑡) = { (𝑡
0 𝑡<𝑎 𝐺(𝑡) = { − 2) 𝑡>2
ℒ{𝐺(𝑡)} = 𝑒 −𝑎𝑠 𝑓(𝑠) 0 𝑡<2
6𝑒 2𝑠
is 𝑠4 .
4. Change Of Scale Property. 1
Since ℒ{sin 𝑡} = 𝑠2 +1, we have
If ℒ{𝐹(𝑡)} = 𝑓(𝑠) then 1 1
1 𝑠 ℒ{sin 3𝑡} = 2
ℒ{𝐹(𝑎𝑡)} = 𝑓( ) 3 𝑠
𝑎 𝑎 (3) + 1
3
= 2
𝑠 +9
5. Laplace Transform Of Derivative.
If ℒ{𝐹(𝑡)} = 𝑓(𝑠) then
ℒ{𝐹′(𝑡)} = 𝑠𝑓(𝑠) − 𝐹(0)
If 𝐹(𝑡) is continuous for 0 ≤ 𝑡 ≤ 𝑁 and of exponential
order for 𝑡 > 𝑁 while 𝐹′(𝑡) is sectionally continuous for
0 ≤ 𝑡 ≤ 𝑁.
➢ If ℒ{𝐹(𝑡)} = 𝑓(𝑠) then
ℒ{𝐹′′(𝑡)} = 𝑠 2 𝑓(𝑠) − 𝑠𝐹(0) − 𝐹′(0)
➢ If ℒ{𝐹(𝑡)} = 𝑓(𝑠) then
ℒ{𝐹 (𝑛) (𝑡)} = 𝑠 𝑛 𝑓(𝑠) − 𝑠 𝑛−1 𝐹(0)
− 𝑠 𝑛−2 𝐹 ′ (0)−. . . −𝑠𝐹 (𝑛−2) (0)
− 𝐹 (𝑛−1) (0)
6. Laplace Transforms Of Integrals. 2
Since ℒ{sin 2𝑡} = 𝑠2 +4, we have
𝑡 𝑓(𝑠)
If ℒ{𝐹(𝑡)} = 𝑓(𝑠) then ℒ {∫0 𝐹(𝑢) 𝑑𝑢} = 𝑠 𝑡
2
ℒ {∫ sin 2𝑢 𝑑𝑢} =
𝑠(𝑠 2+ 4)
0
7. Multiplication by 𝒕𝒏 . 1
Since ℒ{𝑒 2𝑡 } = 𝑠−2, we have
If ℒ{𝐹(𝑡)} = 𝑓(𝑠) then

Mohammad Abdul Halim, Assistant Professor in Mathematics


Page 166 of 249

𝑑𝑛 𝑑 1 1
ℒ{𝑡 𝑛 𝐹(𝑡)} = (−1)𝑛 𝑓(𝑠) = (−1)𝑛 𝑓 (𝑛) (𝑠) ℒ{𝑡𝑒 2𝑡 } = − ( )=
𝑑𝑠 𝑛 𝑑𝑠 𝑠 − 2 (𝑠 − 2)2
2
𝑑 1
ℒ{𝑡 2 𝑒 2𝑡 } = 2 ( )
𝑑𝑠 𝑠 − 2
2
=
(𝑠 − 2)3
8. Division by t. 2
Since ℒ{sin 2𝑡} = 𝑠2 +4, and
If ℒ{𝐹(𝑡)} = 𝑓(𝑠) then sin 2𝑡
∞ lim = 2, we have
𝐹(𝑡) 𝑡→0 𝑡

ℒ{ } = ∫ 𝑓(𝑢) 𝑑𝑢 sin 2𝑡 2
𝑡
𝑠 ℒ{ }=∫ 2 𝑑𝑢
𝐹(𝑡) 𝑡 (𝑢 + 4)
provided lim exists. 𝑠
𝑡→0 𝑡 𝜋 𝑠
= − tan−1
2 2
9. Periodic Functions.
Let 𝐹(𝑡) have period 𝑇 > 0 so that 𝐹(𝑡 + 𝑇) = 𝐹(𝑡), then
𝑇
∫0 𝑒 −𝑠𝑡 𝐹(𝑡) 𝑑𝑡
ℒ{𝐹(𝑡)} =
1 − 𝑒 −𝑠𝑇
10. Behavior of 𝒇(𝒔) as 𝒔 → ∞.
If ℒ{𝐹(𝑡)} = 𝑓(𝑠) then lim 𝑓(𝑠) = 0.
𝑠→∞
Proof of Some Elementary Functions Laplace Transformations:
𝑎 𝑠
1. Prove that (a) ℒ{sin 𝑎𝑡} = 𝑠2 +𝑎2 (b) ℒ{cos 𝑎𝑡} = 𝑠2 +𝑎2 if 𝑠 > 0.
𝑎 𝑎
(𝑐) ℒ{sinh 𝑎𝑡} = 2 2 (d) ℒ{cosh 𝑎𝑡} = 𝑠2 −𝑎2
𝑠 −𝑎


Solutions: (a) L sin at =  e− st sin at dt
0
𝑒 𝑎𝑥 (𝑎 sin 𝑏𝑥−𝑏 cos 𝑏𝑥)
 ** ∫ 𝑒 𝑎𝑡 sin 𝑏𝑥 𝑑𝑥 =
e
=
− st
( − s sin at − a cos at )  𝑎2 +𝑏 2

 s2 + a2 0 and ∫ 𝑒 𝑎𝑥 cos 𝑏𝑥 𝑑𝑥 =
𝑒 𝑎𝑥 (𝑎 cos 𝑏𝑥+𝑏 sin 𝑏𝑥)
𝑎2 +𝑏 2

 a 
= 0 + 2
 s + a 2 

a
= if s  0
s + a2
2


(b) L cos at =  e− st cos at dt
0


 e − st ( − s cos at + a sin at ) 
= 
 s2 + a2 0

Mohammad Abdul Halim, Assistant Professor in Mathematics


Page 167 of 249

 s 
= 0 + 2
 s + a 2 

s
= if s  0
s + a2
2

𝑒 𝑎𝑡 −𝑒 −𝑎𝑡
(c) ℒ{sinh 𝑎𝑡} = ℒ { }
2
1 1
= 2 ℒ{𝑒 𝑎𝑡 } − 2 ℒ{𝑒 −𝑎𝑡 }

1 1 1
= { − }
2 𝑠−𝑎 𝑠+𝑎
𝑎
= 𝑠2 −𝑎2 for 𝑠 > |𝑎|

𝑒 𝑎𝑡 +𝑒 −𝑎𝑡
(d) ℒ{cosh 𝑎𝑡} = ℒ { }
2

1 1
= ℒ{𝑒 𝑎𝑡 } + ℒ{𝑒 −𝑎𝑡 }
2 2
1 1 1
= { + }
2 𝑠−𝑎 𝑠+𝑎
𝑠
= 𝑠2 −𝑎2 for 𝑠 > |𝑎|.

H.W:
Prove that the followings:
1
(a) L 1 = , s0
s
1
(b) L t = 2 , s  0
s

 
(c) L e at =
1
s−a
, sa

  n!
(d) L t n = n +1
s

2. Evaluates L 4e5t + 6t 3 − 3sin 4t + 2 cos 2t . 
Solution:
We have,

L 4e5t + 6t 3 − 3sin 4t + 2 cos 2t 
By using linearity property, we get
   
L 4e5t + L 6t 3 − L 3sin 4t + L 2 cos 2t

Mohammad Abdul Halim, Assistant Professor in Mathematics


Page 168 of 249

  
=4L e5t + 6L t 3 − 3L sin 4t + 2L cos 2t
1 3! 4 s
=4. + 6. 4 − 3. 2 + 2. 2
s −5 s s +4 2
s + 22
4 36 12 2s
= + 4 − 2 + 2 (As desired)
s −5 s s + 16 s + 4
H.W:
Evaluate the followings:


(a) L 3t 4 − 2t 3 + 4e − 3t − 2sin 5t + 3cos 2t 
(b) L e 4t
+ 4t 3 − 2sin 3t + 3cos 5t 
(c) ℒ{4𝑒 5𝑡 + 6𝑡 3 − 3 sin 4𝑡 + 2 cos 2𝑡}
3. Find the Laplace transform of t e or L t e .
3 5t 3 5t
 
Solution:
Let F (t ) = t 3 and L F (t ) = f (s) .
We have,
 
L t3 =
3!
s4
= f (s)
We know that the shifting property is
ℒ{𝑒 𝑎𝑡 𝐹(𝑡)} = 𝑓(𝑠 − 𝑎)
By using first shifting property, we get
 
L t 3e5t = f ( s − 5 )
3!
=
( s − 5)
4

6
= (As desired)
( s − 5)
4

−2 t
4. Find the Laplace transform of e −2t sin 4t or L e sin 4t .  
Solution:
Let F (t ) = sin 4t and L F (t ) = f (s) .
We have,
4 4
L  F (t ) = L sin 4t = = 2 = f ( s)
s +4
2 2
s + 16
We know that the shifting property is
ℒ{𝑒 𝑎𝑡 𝐹(𝑡)} = 𝑓(𝑠 − 𝑎)
By using first shifting property, we get
 
L e −2t sin 4t = f ( s + 2 )
4
= (As desired)
( s + 2)
2
+ 16

Mohammad Abdul Halim, Assistant Professor in Mathematics


Page 169 of 249

 
5. Find the Laplace transform of e−2t ( 3cos 6t − 5sin 6t ) or L e ( 3cos 6t − 5sin 6t ) .
−2 t

Solution:
Let F (t ) = 3cos 6t − 5sin 6t and L F (t ) = f (s) .
We have,
L F (t ) = L 3cos 6t − 5sin 6t
= 3L cos6t − 5L sin 6t
s 6
= 3. 2 − 5. 2
s +6 2
s + 62
3s 30
= 2 − 2
s + 36 s + 36
3s − 30
= 2 = f (s)
s + 36

We know that the shifting property is


ℒ{𝑒 𝑎𝑡 𝐹(𝑡)} = 𝑓(𝑠 − 𝑎)
By using first shifting property, we get

L  F (t ) = L e −2t ( 3cos 6t − 5sin 6t ) 
= f ( s + 2)
3 ( s + 2 ) − 30
=
( s + 2)
2
+ 36
3s + 6 − 30
=
( s + 2)
2
+ 36
3s − 24
= (As desired)
( s + 2)
2
+ 36

H.W:
(a) Find ℒ{𝑒 −2𝑡 (3 cos 6𝑡 − 5 sin 6𝑡)}
(b) Find the followings:
i.  
L t 3e −3t

ii. L e cos 2t


−t

iii. L 2e sin 4t 3t

iv. L e ( 3sin 4t − 4 cos 4t )


2t

v. ℒ{𝑒 −𝑡 (3 sinh 2𝑡 − 5 cosh 2𝑡)}


vi. ℒ{𝑒 −2 𝑡 (5 sinh 3𝑡 − 7 cosh 3𝑡)}
vii. ℒ{𝑒 −𝑡 sin2 𝑡}
6. Find the Laplace transformation of t sin at or L t sin at

Mohammad Abdul Halim, Assistant Professor in Mathematics


Page 170 of 249

Solution:
Let F (t ) = sin at and L F (t ) = f (s) .
From the Laplace transformation, we have
a
L  F (t ) = L sin at = = f ( s)
s + a2
2

The Multiplication Property, by t n ,we have


𝑑𝑛
ℒ{𝑡 𝑛 𝐹(𝑡)} = (−1)𝑛 𝑑𝑠𝑛 𝑓(𝑠)

From above Property, we get,


d
L t sin at = ( −1)
1
f (s)
ds
d  a 
=−  2 
ds  s + a 2 

( )
d 2 −1
= −a s + a2
ds
= a ( s 2 + a 2 )  ( 2s )
−2

2as
= (As desired)
(s )
2
2
+ a2
7. Find the Laplace transformation of t 2 cos at or L t 2 cos at .  
Solution:
Let F (t ) = cos at and L F (t ) = f (s) .
From the Laplace transformation, we have
s
L  F (t ) = L cos at  = = f (s)
s + a2
2

The Multiplication Property, by t n ,we have


𝑑𝑛
ℒ{𝑡 𝑛 𝐹(𝑡)} = (−1)𝑛 𝑑𝑠𝑛 𝑓(𝑠)

From above Property, we get,


d2
 
L t 2 cos at = ( −1)
ds 2
2
f (s)

d2  s 
= 2 2 2
ds  s + a 

= 
 2 (2 
d  s + a .1 − s ( 2s + 0 )  )

( )
2
ds 
 s 2
+ a 2

 
d  s 2 + a 2 − 2s 2 
=  
(
ds  s 2 + a 2 2 
  )

Mohammad Abdul Halim, Assistant Professor in Mathematics


Page 171 of 249

 
d  a2 − s2 
=  2
(
ds  s 2 + a 2
 ) 

(s ) ( 0 − 2s ) − ( a − s ) .2 ( s + a ) .2s
2
2
+ a2 2 2 2 2

=
(s + a )
4
2 2

=
( s + 2a s + s ) ( −2s ) − 4s ( a − s )( s + a )
4 2 2 4 2 2 2 2

(s + a )
4
2 2

=
( s + 2a s + s ) ( −2s ) − 4s ( a − s )
4 2 2 4 4 4

(s + a )
4
2 2

=
( s + 2a s + s ) ( −2s ) − ( 4sa − 4s )
4 2 2 4 4 5

(s + a )
4
2 2

=
( −2s − 4a s − 2s ) − ( 4sa − 4s )
5 2 3 5 4 5

(s + a )
4
2 2

−2s 5 − 4a 2 s 3 − 2s 5 − 4sa 4 + 4s 5
=
(s )
4
2
+ a2
−4a 2 s 3 − 4sa 4
=
(s )
4
2
+ a2
− 4a 2 s
= (As desired)
(s )
3
2
+ a2
8. 𝓛{𝒆−𝒕 𝐬𝐢𝐧𝟐 𝒕} =?
1 1 1 1 𝑠
Solution: ℒ{sin2 𝑡} = 2 ℒ{2 sin2 𝑡} = 2 ℒ{1 − cos 2𝑡} = 2 [𝑠 − 𝑠2 +4] = 𝑓(𝑠)

1 1 𝑠+1 𝑠 2 + 2𝑠 + 5 − 𝑠 2 − 2𝑠 − 1
ℒ{𝑒 −𝑡 sin2 𝑡} = 𝑓(𝑠 + 1) = [ − ] =
2 𝑠 + 1 (𝑠 + 1)2 + 4 2(𝑠 + 1)(𝑠 2 + 2𝑠 + 5)
2
= 2
(𝑠 + 1)(𝑠 + 2𝑠 + 5)

9. 𝓛{𝒕𝟐 𝐬𝐢𝐧 𝒕} =?
1
Solution: ℒ{sin 𝑡} = [𝑠2 +1] = 𝑓(𝑠)

2
𝑑2 𝑑2 1
ℒ{𝑡 sin 𝑡} = (−1)2 𝑓(𝑠) = ( )
𝑑𝑠 2 𝑑𝑠 2 𝑠 2 + 1

Mohammad Abdul Halim, Assistant Professor in Mathematics


Page 172 of 249

2 3 2
𝑑 1 1 1 4𝑠 2 − 2(𝑠 2 + 1) 6𝑠 2 − 2
= [− ( 2 ) . 2𝑠] = 2 ( 2 ) . 2𝑠 − ( 2 ) .2 = =
𝑑𝑠 𝑠 +1 𝑠 +1 𝑠 +1 (𝑠 2 + 1)3 (𝑠 2 + 1)3

H.W:

(a) Show that L t sin 2t = ?
2

2s3 − 6s

(b) Show that L t cos t =
2

(s )
3
2
+1

𝟐𝝅 𝟐𝝅
𝐜𝐨𝐬 (𝒕 − ) 𝒕>
10. 𝓛{𝑭(𝒕)} =? , 𝐢𝐟 𝑭(𝒕) = { 𝟑 𝟑
𝟐𝝅
𝟎 𝒕< 𝟑

𝑠 2𝜋
Solution: Since ℒ{cos 𝑡} = 𝑠2 +1, using the second translation property with 𝛼 = we get,
3
2𝜋𝑠

𝑠 𝑒 3
ℒ{𝐹(𝑡)} = 𝑠2 +1

11. 𝓛{𝐬𝐢𝐧 √𝒕} =?

3 5 7 3 5 7
(√𝑡) (√𝑡) (√𝑡) 1
𝑡2 𝑡2 𝑡2
Solution: We have, sin √𝑡 = √𝑡 − + − + ⋯ = 𝑡 − 3! + 5! − 7! + ⋯
2
3! 5! 7!

Then the Laplace transform is


3 5 7
1 𝑡2 𝑡2 𝑡2
ℒ{sin √𝑡} = ℒ {𝑡 2 } −ℒ{ }+ℒ{ }−ℒ{ }+⋯
3! 5! 7!

1 3 5 7
Γ (2 + 1) Γ (2 + 1) Γ (2 + 1) Γ (2 + 1)
= 1 − 3 + 5 − 7 +⋯
𝑠 2+1 3! 𝑠 2+1 5! 𝑠 2+1 7! 𝑠 2+1
3 5 7 9
Γ (2) Γ (2) Γ (2) Γ (2)
= 3 − 5 + 7 − 9 +⋯
𝑠2 3! 𝑠 2 5! 𝑠 2 7! 𝑠 2
1 1 3 1 1 5 3 1 1 7 5 3 1 1
Γ (2) 2 . 2 Γ (2) 2 . 2 . 2 Γ (2) 2 . 2 . 2 . 2 Γ (2)
= 2 − + − +⋯
3 5 7 9
𝑠2 3! 𝑠 2 5! 𝑠 2 7! 𝑠 2
√𝜋 3√𝜋 5.3 √𝜋 7.5.3 √𝜋
= 3− 5+ 7− 9 +⋯
2 𝑠2 4. 6 𝑠 2 8. 5.4.3.2 𝑠 2 16. 7.6.5.4.3.2 𝑠 2

Mohammad Abdul Halim, Assistant Professor in Mathematics


Page 173 of 249

2 3
1 1
√𝜋 1 2 ( ) 2 ( )
= 3 {1 −( 2 )+ 2 𝑠 − 2 𝑠 +⋯
2 𝑠 2! 3!
2 𝑠2
√𝜋 1
− 2
= 3𝑒
2 𝑠
2 𝑠2
Remember the Formulae:
𝑛! = Γ(𝑛 + 1) = 𝑛Γ(𝑛) = 𝑛(𝑛 − 1)Γ(𝑛 − 1) ;
𝑥2 𝑥3 1
𝑒 −𝑥 = 1 − 𝑥 + − + ⋯ ; Γ (2) = √𝜋
2! 3!

𝐜𝐨𝐬 √𝒕
12. 𝓛{ } =?
√𝒕

2 4 6
(√𝑡) (√𝑡) (√𝑡) 𝑡1 𝑡2 𝑡3
Solution: We have, cos √𝑡 = 1 − + − + ⋯ = 𝑡 0 − 2! + 4! − 6! + ⋯
2! 4! 6!
1 3 5
cos √𝑡 −
1 𝑡2 𝑡2 𝑡2
∴ =𝑡 2 − + − +⋯
√𝑡 2! 4! 6!
Then the Laplace transform is
1 3 5
cos √𝑡 1 𝑡2 𝑡2 𝑡2
= 𝑡 −2 − + − +⋯
√𝑡 2! 4! 6!
1 3 5
cos √𝑡 −
1 𝑡2 𝑡2 𝑡2
ℒ{ } = ℒ {𝑡 2} −ℒ{ }+ℒ{ }−ℒ{ }+⋯
√𝑡 2! 4! 6!

1 1 3 5
Γ (− 2 + 1) Γ (2 + 1) Γ (2 + 1) Γ (2 + 1)
= 1 − 1 + 3 − 5 +⋯
𝑠 −2+1 2! 𝑠 2+1 4! 𝑠 2+1 6! 𝑠 2+1
1 3 5 7
Γ (2) Γ (2) Γ (2) Γ (2)
= 1 − 3 + 5 − 7 +⋯
𝑠2 2! 𝑠 2 4! 𝑠 2 6! 𝑠 2
1 1 1 3 1 1 5 3 1 1
Γ (2) Γ (2) 2 . 2 Γ (2) 2 . 2 . 2 Γ (2)
= − 2 + − +⋯
1 3 5 7
𝑠2 2! 𝑠 2 4! 𝑠 2 6! 𝑠 2

Mohammad Abdul Halim, Assistant Professor in Mathematics


Page 174 of 249

√𝜋 √𝜋 3 √𝜋 5.3 √𝜋
= 1 − 3 + 5 − 7 +⋯
𝑠2 2.2 𝑠2 2.2.4.3.2 𝑠2 8.6.5.4.3.2 𝑠2
2
1 1 3
√𝜋 1 2 ( ) ( 2 )
= 1 {1 − ( 2 ) + 2 𝑠 − 2 𝑠 +⋯
2 𝑠 2! 3!
𝑠2
√𝜋 1
− 2
= 1 𝑒 2 𝑠
𝑠2
𝟎 𝟎<𝒕<2
13. If 𝑭(𝒕) = { then 𝓛{𝑭(𝒕)} =?
𝟒 𝒕>2
4
Solution: Since ℒ{4} = 𝑠 , using th 4e second translation property with 𝛼 = 2 we get,

4 −2𝑠
ℒ{𝐹(𝑡)} = 𝑒
𝑠
𝟐𝒕 𝟎≤𝒕≤𝟓
14. If 𝑭(𝒕) = { then 𝓛{𝑭(𝒕)} =?
𝟏 𝒕>5
∞ 5 ∞
Solution: ℒ{𝐹(𝑡)} = ∫0 𝑒 −𝑠𝑡 𝐹(𝑡)𝑑𝑡 = ∫0 𝑒 −𝑠𝑡 2𝑡 𝑑𝑡 + ∫5 𝑒 −𝑠𝑡 (1)𝑑𝑡
5 ∞
5
−𝑠𝑡
𝑑
= [2𝑡 ∫ 𝑒 𝑑𝑡] − 2 ∫ { (𝑡) ∫ 𝑒 −𝑠𝑡 𝑑𝑡} 𝑑𝑡 + ∫ 𝑒 −𝑠𝑡 𝑑𝑡
0 𝑑𝑡
0 5

5 5 ∞
𝑒 −𝑠𝑡 𝑒 −𝑠𝑡 𝑒 −𝑠𝑡
= [2𝑡 ] − 2∫ 𝑑𝑡 + [ ]
−𝑠 0 −𝑠 −𝑠 5
0

2 2 1
= − [5𝑒 −5𝑠 − 0] − 2 [𝑒 −𝑠𝑡 ]50 − [𝑒 −∞ − 𝑒 −5𝑠 ]
𝑠 𝑠 𝑠
2 2 1
= − [5𝑒 −5𝑠 ] − 2 [𝑒 −5𝑠 − 1] − [0 − 𝑒 −5𝑠 ]
𝑠 𝑠 𝑠
10 −5𝑠 2 −5𝑠 1
=− 𝑒 − 2 [𝑒 − 1] + 𝑒 −5𝑠
𝑠 𝑠 𝑠
2 9
= 2
[1 − 𝑒 −5𝑠 ] + 𝑒 −5𝑠
𝑠 𝑠

Mohammad Abdul Halim, Assistant Professor in Mathematics


Page 175 of 249

H.W:
Evaluate the followings:
2𝜋 2𝜋
cos (𝑡 − ) 𝑡>
3 3
(a) 𝐹(𝑡) = { 2𝜋
0 𝑡< 3
0 0<𝑡<2
(b) 𝐹(𝑡) = {
4 𝑡>2
2𝑡 0≤𝑡≤5
(c) 𝐹(𝑡) = {
1 𝑡>5
(𝑡 − 1)2 𝑡>1 .
(d) 𝐹(𝑡) = {
0 0<𝑡<1
(e) ℒ{(𝑡 + 2)2 𝑒 𝑡 }
(f) ℒ{sin √𝑡}
cos √𝑡
(g) ℒ { }
√𝑡
(h) ℒ{cosh2 4𝑡}
sin 2t
15. Find the Laplace Transform of .
t
Solution:
Let F (t ) = sin 2t and L F (t ) = f (s) .
From the Laplace transformation, we have
2 2
L  F (t ) = L sin 2t = = 2 = f ( s)
s +2
2 2
s +4
The Division Property, by t ,we have
𝐹(𝑡) ∞
ℒ{ } = ∫𝑠 𝑓(𝑢) 𝑑𝑢
𝑡
Using above property, we get,

 F (t )   sin 2t 
L =L  =  f (u ) du
 t   t  s

2
= du
s
u +4 2


1
= 2 du
s
u + 22
2


1 u
= 2  tan −1 
2 2 s

1 u
= 2.  tan −1 
2 2 s

Mohammad Abdul Halim, Assistant Professor in Mathematics


Page 176 of 249


 u
=  tan −1 
 2 s
s
= tan −1  − tan −1
2
 s
= − tan −1
2 2
s
= cot −1 (As desired)
2
e − at − e − bt  e− at − e−bt 
16. Find the Laplace transformation of or L  .
t  t 
Solution:
Let F (t ) = e− at − e−bt and L F (t ) = f (s) .
From the Laplace transformation, we have

     
L  F (t ) = L e − at − e − bt = L e − at − L e − bt =
1
+
1
s+a s+b
= f (s)
The Division Property, by t ,we have
𝐹(𝑡) ∞
ℒ{ } = ∫𝑠 𝑓(𝑢) 𝑑𝑢
𝑡
Using above property, we get,
 F (t )   e− at − e−bt    1 1 
L =L =  −  du
 t   t  s u + a u + b 

 1 1 
=  − du
s 
u + a u + b 

= ln ( u + a ) − ln(u + b)  s

 u +a
= ln
 u + b  s
s+b
= ln (As desired)
s+a
H.W:
Find the Laplace transform of the following function:
(a) L  cos at − cos bt 
 t 
 sinh t 
(b) L 
 t 

(c) L 1 − e 
t


 t 

Mohammad Abdul Halim, Assistant Professor in Mathematics


Page 177 of 249

1 − cos at 
(d) L 
 t 

(e) L  e sin t 
−t


 t 

(f) L  sin
t 2


 t 

Mohammad Abdul Halim, Assistant Professor in Mathematics


Page 178 of 249

THE INVERSE LAPLACE TRANSFORM

Definition: If the Laplace Transform of a function 𝐹(𝑡) is 𝑓(𝑠), i.e. if ℒ{𝐹(𝑡)} = 𝑓(𝑠), then
𝐹(𝑡) is called an inverse Laplace transform of 𝑓(𝑠) and we write symbolically 𝐹(𝑡) =
ℒ −1 {𝑓(𝑠)} where ℒ −1 is called the inverse Laplace transformation operator.
Some Inverse Laplace Transforms
𝑓(𝑠) 𝓛−1 {𝑓(𝑠)} = 𝐹(𝑡)
01 1 1
𝑠>0
𝑠
02 1 𝑡
𝑠>0
𝑠2
03 1 𝑡𝑛
𝑠>0 ; 𝑛 = 0,1,2,3, …
𝑛+1 𝑛!
𝑠
04 1 𝑒 𝑎𝑡
𝑠>𝑎
𝑠−𝑎
05 1 sin 𝑎𝑡
𝑠>0
𝑠 + 𝑎2
2 𝑎
06 𝑠 cos 𝑎𝑡
𝑠>0
𝑠 + 𝑎2
2

07 1 sinh 𝑎𝑡
𝑠 > |𝑎|
𝑠 2 − 𝑎2 𝑎
08 𝑠 cosh 𝑎𝑡
𝑠 > |𝑎|
𝑠 − 𝑎2
2

Some Important Properties of Inverse Laplace Transform


Property Example
1. Linearity Property. 4 3𝑠 5
ℒ −1 { − 2 + 2 }
If 𝑐1 and 𝑐2 are any constants while 𝑓1 (𝑠) and 𝑓2 (𝑠) are 𝑠 − 2 𝑠 + 16 𝑠 + 4
functions with Laplace transforms 𝐹1 (𝑡) and 𝐹2 (𝑡) 1 𝑠
= 4ℒ −1 { } − 3ℒ −1 { 2 }
respectively, then 𝑠−2 𝑠 + 16
ℒ −1 {𝑐1 𝑓1 (𝑠) + 𝑐2 𝑓2 (𝑠)} = 𝑐1 ℒ −1 {𝑓1 (𝑠)} + 𝑐2 ℒ −1 {𝑓2 (𝑠)} 5
+ 5ℒ −1 { 2 }
= 𝑐1 𝐹1 (𝑡) + 𝑐2 𝐹2 (𝑡) 𝑠 +4
This result is easily extended to more than two functions. 5
= 4𝑒 2𝑡 − 3 cos 4𝑡 + sin 2𝑡
2
2. First Translation Or Shifting Property. −1 4
Since ℒ {𝑠2 +16} = sin 4𝑡
If ℒ −1 {𝑓(𝑠)} = 𝐹(𝑡) then 4
ℒ −1 {𝑓(𝑠 − 𝑎)} = 𝑒 𝑎𝑡 𝐹(𝑡) ∴ ℒ −1 { } = 𝑒 −2𝑡 sin 4𝑡
(𝑠 + 2)2 + 16

3. Second Translation Or Shifting Property. 1


Since ℒ −1 {𝑠2 +1} = sin 𝑡, We have

Mohammad Abdul Halim, Assistant Professor in Mathematics


Page 179 of 249

If ℒ −1 {𝑓(𝑠)} = 𝐹(𝑡) then


6𝑒 −𝜋𝑠/3
𝐹(𝑡 − 𝑎) 𝑡>𝑎 ℒ −1 { }
ℒ −1 {𝑒 −𝑎𝑠 𝑓(𝑠)} = { 𝑠2 + 1
0 𝑡<𝑎 𝜋 𝜋
sin (𝑡 − ) , 𝑡 >
={ 3 3
𝜋
0 , 𝑡<
3
4. Change Of Scale Property. −1 1
Since ℒ {𝑠2 +1} = sin 𝑡, we have
If ℒ −1 {𝑓(𝑠)} = 𝐹(𝑡), then 3 1 3𝑡
1 𝑡 ℒ −1 { } = sin
ℒ −1 {𝑓(𝑘𝑠)} = 𝐹( ) 2
(2𝑠) + 9 2 2
𝑘 𝑘
5. Laplace Transform Of Derivative. 1
Since ℒ −1 {𝑠2 +1} = sin 𝑡
If ℒ −1 {𝑓(𝑠)} = 𝐹(𝑡), then 𝑑 1 2𝑠
𝑑𝑛 and 𝑑𝑠 (𝑠2 +1) = − (𝑠2+1)2, we have
ℒ −1 {𝑓 (𝑛) (𝑠)} = ℒ −1 { 𝑛 𝑓(𝑠)} = (−1)𝑛 𝑡 𝑛 𝐹(𝑡)
𝑑𝑠 2𝑠
ℒ −1 {− 2 } = −𝑡 sin 𝑡
(𝑠 + 1)2
6. Laplace Transforms Of Integrals. 1 1 1
Since ℒ −1 {𝑠(𝑠+1)} = ℒ −1 {𝑠 − (𝑠+1)}
If ℒ −1 {𝑓(𝑠)} = 𝐹(𝑡) then
∞ = 1 − 𝑒 −𝑡 , we have
𝐹(𝑡) ∞
ℒ −1 {∫ 𝑓(𝑢) 𝑑𝑢} = 1 1
𝑡 ℒ −1 {∫ ( − ) 𝑑𝑢}
𝑠 𝑢 𝑢+1
𝑠
1
= ℒ −1 {ln (1 + )}
𝑠
1 − 𝑒 −𝑡
=
𝑡
7. Multiplication by 𝑠 𝒏 . −1 1
Since ℒ {𝑠2 +1} = sin 𝑡 and sin 0 =
If ℒ −1 {𝑓(𝑠)} = 𝐹(𝑡) and 𝐹(0) = 0, then
0Then
ℒ −1 {𝑠𝑓(𝑠)} = 𝐹 ′ (𝑡) 𝑠 𝑑
If 𝐹(0) ≠ 0, then ℒ −1 { 2 } = (sin 𝑡) = cos 𝑡
➢ ℒ −1 {𝑠𝑓(𝑠) − 𝐹(0)} = 𝐹 ′ (𝑡) 𝑠 +1 𝑑𝑡
➢ ℒ −1 {𝑠𝑓(𝑠)} = 𝐹 ′ (𝑡) + 𝐹(0)𝛿(𝑡)
8. Division by s. 1 1
Since ℒ −1 {𝑠2 +4} = 2 sin 2𝑡
If ℒ −1 {𝑓(𝑠)} = 𝐹(𝑡) then
𝑡 We have,
𝑓(𝑠) 𝑡
ℒ −1 { } = ∫ 𝐹(𝑢) 𝑑𝑢 1 1
𝑠 ℒ −1 { 2 } = ∫ sin 2𝑢 𝑑𝑢
0 𝑠(𝑠 + 4) 2
0
1
= (1 − cos 2𝑡)
4
9. The Convolution Property. −1 1
Since ℒ {𝑠−1} = 𝑒 𝑡 and
If ℒ −1 {𝑓(𝑠)} = 𝐹(𝑡) and ℒ −1 {𝑔(𝑠)} = 𝐺(𝑡), then 1
𝑡 ℒ −1 {𝑠−2} = 𝑒 2𝑡 , we have
ℒ −1 {𝑓(𝑠)𝑔(𝑠)} = ∫ 𝐹(𝑢)𝐺(𝑡 − 𝑢)𝑑𝑢 = 𝐹 ∗ 𝐺 1
ℒ −1 { }
0 (𝑠 − 1)(𝑠 − 2)

Mohammad Abdul Halim, Assistant Professor in Mathematics


Page 180 of 249

𝑡
We call 𝐹 ∗ 𝐺 the convolution or Faltung of 𝐹and 𝐺, and the
theorem is called the convolution theorem or Property. = ∫ 𝑒 𝑢 𝑒 2(𝑡−𝑢) 𝑑𝑢 = 𝑒 2𝑡 − 𝑒 𝑡
0
Methods of Finding Inverse Laplace Transforms
1. Partial Fractions Method: Any Rational Function 𝑃(𝑠)/𝑄(𝑠) where 𝑃(𝑠) and 𝑄(𝑠) are
polynomials, with the degree of 𝑃(𝑠) less than that of 𝑄(𝑠), can be written as the sum of rational
functions [ called partial fractions] having the form
𝐴 𝐴𝑠 + 𝐵
,
(𝑎𝑠 + 𝑏)𝑟 (𝑎𝑠 2 + 𝑏𝑠 + 𝑐)𝑟
where𝑟 = 1,2,3, … By finding the inverse Laplace transform of each the partial fractions, we can
find
𝑃(𝑠)
ℒ −1 { }
𝑄(𝑠)

2. Series methods: If 𝑓(𝑠) has a series expansion in inverse powers of 𝑠 given by


𝑎0 𝑎1 𝑎2 𝑎3
𝑓(𝑠) = + + + +⋯
𝑠 𝑠2 𝑠3 𝑠4
then under suitable conditions we can invert term by term to obtain
𝑎2 𝑡 2 𝑎3 𝑡 3 𝑎4 𝑡 4
𝐹(𝑡) = 𝑎0 + 𝑎1 𝑡 + + + +⋯
2! 3! 4!
3. The Heaviside Expansion Formula:
Let 𝑃(𝑠) and 𝑄(𝑠) be polynomials where 𝑃(𝑠) has degree less than that of 𝑄(𝑠). Suppose that
𝑄(𝑠) has 𝑛 distinct zeros
𝑎𝑘 , 𝑘 = 1,2,3, … , 𝑛. Then
𝑛
𝑃(𝑠) 𝑃(𝑎𝑘 ) 𝑎 𝑡
ℒ −1 { }=∑ ′ 𝑒 𝑘
𝑄(𝑠) 𝑄 (𝑎𝑘 )
𝑘=1

This is often called 𝐻𝑒𝑎𝑣𝑖𝑠𝑖𝑑𝑒’𝑠 𝑒𝑥𝑝𝑎𝑛𝑠𝑖𝑜𝑛 𝑡ℎ𝑒𝑜𝑟𝑒𝑚 𝑜𝑟 𝑓𝑜𝑟𝑚𝑢𝑙𝑎

Worked Out Problems


2 s − 11  2s − 11 
1. Find the inverse Laplace transform of or L −1  2 .
s −s−6
2
s − s − 6
Solution:
 2 s − 11  −1  2 s − 11 
We have, L −1  2 =L  2 
s − s − 6  s − 3s + 2 s − 6 

Mohammad Abdul Halim, Assistant Professor in Mathematics


Page 181 of 249


 2s − 11 

= L −1  
 s ( s − 3) + 2 ( s − 3 ) 
 

 2s − 11  
= L −1  
 ( s − 3)( s + 2 ) 
 
 −1 3 
= L −1  + 
s −3 s + 2
 3 1 
= L −1  − 
 s + 2 s − 3
 3  −1  1 
= L −1  −L   [Using Linearity Property]
s + 2  s − 3
 1  −1  1 
= 3L −1  −L  
s + 2  s − 3
= 3e −2t − e3t (As desired)
6s − 4 −1  6s − 4 
2. Find the inverse Laplace transform of or L  2 .
s − 4s + 20
2
 s − 4 s + 20 
Solution:
We have,
 6s − 4   6s − 4 
L −1  2 =L
−1
 2 
 s − 4 s + 20   s − 4 s + 4 + 16 

−1
 6 ( s − 2 ) + 8 
=L  
 ( s − 2 ) + 16 
2

 ( s − 2 ) 
−1 −1
 4 
= 6L  + 2L  
 ( s − 2 ) + 16   ( s − 2 ) + 4 
2 2 2

= 6e 2t cos 4t + 2e 2t sin 4t (As desired)

2s − 5  2s − 5 
3. Find the inverse Laplace transform of or L −1
 2 .
9 s 2 − 25  9 s − 25 
Solution:
We have,
 2s − 5   2s 5 
L −1  2 =L
−1
 2 − 2 
 9 s − 25   9 s − 25 9 s − 25 
−1  2s  −1  5 
=L  2 −L  2 
 9 s − 25   9 s − 25 

Mohammad Abdul Halim, Assistant Professor in Mathematics


Page 182 of 249

   
 2s   5 
= L −1  −L
−1
 
 9  s 2 − 25    9  s 2 − 25  
  9     9  
   
2 −1  s  5 −1  1 
= L  − L  
9  s 2 − 25  9  s 2 − 25 
 9   9 
   5 
   
2  s  1 −1  3 
= L −1  2
− L  2
9 s −
2  5   3  s2 −  5  
     
 3    3 
2 5t 1 5t
= cosh − sinh (As desired)
9 3 3 3
s−2  s−2 
4. Find the inverse Laplace transform of or L −1
 2 .
6 s 2 + 20  6 s + 20 
Solution:
We have,
 
 s−2   s − 2 
−1 
L −1  2 =L  
 6s + 20   6  s 2 + 20  
  6  
 
1 −1  s−2 
= L  
6  s 2 + 10 
 3
 
1 −1  s 2 
= L  − 
6  s 2 + 10 s 2 + 10 
 3 3
    
1 −1
 s   2 
= L   − L −1  
6  s 2 + 10   s 2 + 10 
  3  3 
   
1 −1  s  1 −1  1 
= L  − L  
6  s 2 + 10  3  s 2 + 10 
 3  3

Mohammad Abdul Halim, Assistant Professor in Mathematics


Page 183 of 249

   
   10 
1 −1  s  1 3 −1  3 
= L  2
− . L  2
6  2  10   3 10  2  10  
s + 3   s + 3  
       
   
   10 
1 −1  s  1 1 −1  3 
= L  2
− . L  2
6  2  10   3 10  2  10  
 s +    s + 3  
  3      
1 10 1 10
= cos t− sin t (As desired)
6 3 30 3

 5s + 4 2s − 18 24 − 30 s 

5. Find the inverse Laplace transform L −1  3 − 2 + .

 s s + 9 s 4


Solution:
We have,
 5s + 4 2s − 18 24 − 30 s 
  
−1  5 4 2s 18 
24 30 s 
L −1  3 − 2 + =L  2 + 3 − 2 − 2 + 4 − 4 

 s s +9 s 4

 
s s s +9 s +9 s s  

  7 
 1 Γ 
2! s 3 3!
= L −1 5 2 + 2 3 − 2 2 2 − 6 2 2 + 4 4 −
30  2 

 s s s +3 s +3 s 7 7
Γ  s2 
 2 

 
−1 30 5

= L 5t + 2t − 2 cos 3t + 6sin 3t + 4t −
2 3
t2 
 5 3 1
. .  
 2 2 2 
 16 2 
5
= L −1 5t + 2t 2 − 2cos 3t + 6sin 3t + 4t 3 − t  (As desired)
  
  1 
6. Find the inverse Laplace transform L −1 ln 1 + 2   .
  s 
Solution:
Given that,
  1 
L −1 ln 1 + 2  
  s 

We have L F (t ) = f (s)  F (t ) = L −1
 f (s)

Mohammad Abdul Halim, Assistant Professor in Mathematics


Page 184 of 249

1 1
∴ ℒ −1 {𝑓(𝑠)} = ℒ −1 {ln (1 + 𝑠2 )} ⇒ 𝑓(𝑠) = ln (1 + 𝑠2 )
𝑑 1 2 1 𝑠
Now 𝑓 ′ (𝑠) = 𝑑𝑠 {ln (1 + 𝑠2 )} = − 𝑠(𝑠2 +1) = −2 {𝑠 − 𝑠2 +1}

1 𝑠
Then, ℒ −1 {𝑓 ′ (𝑠)} = −2ℒ −1 {𝑠 − 𝑠2 +1} = −2(1 − cos 𝑡)
𝑑
Since ℒ{𝑡 𝐹(𝑡)} = (−1)1 𝑑𝑠 𝑓(𝑠) = −𝑓 ′ (𝑠)

⇒ 𝑡 𝐹(𝑡) = −ℒ −1 {𝑓 ′ (𝑠)} = −{−2(1 − cos 𝑡)} = 2(1 − cos 𝑡)


2(1−cos 𝑡)
⇒ 𝐹(𝑡) = 𝑡
(As desired)
(𝑠+1)𝑒 −𝜋𝑠
7. Find ℒ −1 { } =?
𝑠2 +𝑠+1
Solution:

1 1 1
𝑠+1 𝑠+1 𝑠+ + 𝑠+
Since ℒ −1 {𝑠2 +𝑠+1} = ℒ −1 { 1 2
} = ℒ −1 { 2 2
1 2 3
} = ℒ −1 { 2
2 }+
(𝑠+ ) +
3
(𝑠+ ) + 1 2 √3
2 4 2 4 (𝑠+ ) +( )
2 2

√3 1
1 1 − 𝑡
1 −1 2 − 𝑡 √3 1 − 𝑡 √3 𝑒 2 √3 √3
ℒ { 2 }=𝑒 2 cos ( 2 𝑡) + 𝑒 2 sin ( 2 𝑡) = {√3 cos ( 𝑡) + sin ( 𝑡)}
√3 1 2 √3 √3 √3 2 2
(𝑠+ ) +( )
2 2

1
(𝑡−𝜋)
(𝑠 + 1)𝑒 −𝜋𝑠 𝑒 −2 √3 √3
−1
∴ℒ { 2 }={ (√3 cos (𝑡 − 𝜋) + sin (𝑡 − 𝜋)) 𝑡>𝜋
𝑠 +𝑠+1 √3 2 2
0 𝑡<𝜋
(As desired)
𝑒 −1/𝑠 cos 2√𝑡 𝑒 −𝑎/𝑠
8. If ℒ −1 { 𝑠1/2 } = , find ℒ −1 { 𝑠1/2 } where 𝑎 > 0.
√𝜋𝑡

Solution:
1 𝑡
By the change of scale property, we have ℒ −1 {𝑓(𝑘𝑠)} = 𝑘 𝐹 (𝑘).

𝑒 −1/𝑠 cos 2√𝑡


Given, ℒ −1 { }= ; Replacing 𝑠 by 𝑘𝑠,
𝑠1/2 √𝜋𝑡

𝑡 𝑡 𝑡
−1 𝑒 −1/𝑘𝑠 1 cos 2√𝑘 1 cos 2√𝑘 1 𝑒 −1/𝑘𝑠 1 cos 2√𝑘
we have ℒ { }= = ⇒ ℒ −1 { 𝑠1/2 } =
(𝑘𝑠)1/2 𝑘 𝜋𝑡 √𝑘 √𝜋𝑡 √𝑘 √𝑘 √𝜋𝑡

𝑘

Mohammad Abdul Halim, Assistant Professor in Mathematics


Page 185 of 249

𝑡
𝑒 −1/𝑘𝑠 cos 2√ 1
𝑘
⇒ ℒ −1 { 𝑠1/2 } = ; letting 𝑘 = 𝑎
√𝜋𝑡

𝑒 −𝑎/𝑠 cos 2√𝑎𝑡


⇒ ℒ −1 { 𝑠1/2 } =
√𝜋𝑡

(As desired)
𝑠
9. Find ℒ −1 {(𝑠2+𝑎2)2 } =?

Solution:
We have
𝑑 1 2𝑠
{ 2 2 } = − (𝑠2 2 )2,
𝑑𝑠 𝑠 +𝑎 +𝑎
𝑠 1 𝑑 1
Thus (𝑠2 +𝑎2)2 = − 2 𝑑𝑠 {𝑠2 +𝑎2};
1 sin 𝑎𝑡 𝑠 1 𝑑 1
Then since ℒ −1 {𝑠2 +𝑎2 } = ∴ ℒ −1 {(𝑠2 +𝑎2)2} = − 2 ℒ −1 {𝑑𝑠 (𝑠2 +𝑎2)} =
𝑎
1 sin 𝑎𝑡 𝑡 sin 𝑎𝑡
𝑡 ( )=
2 𝑎 2𝑎
(As desired)
H.W:
Find the following inverse Laplace transformation:
 4 s + 12 
(a) L −1  2 
 s + 8s + 16 
 3s + 7 
(b) L −1  2 
 s − 2s − 3 
 3s − 8 4 s − 24 
(c) L −1  2 − 2 
 s + 4 s − 16 
 6 3 + 4s 8 − 6s 
(d) L −1  − 2 + 
 2 s − 3 9 s − 16 16 s + 9 
2

 2s − 5 4 s − 18 
(e) L −1  2 + 2 
 4 s + 25 9 − s 
 5s − 10 
(f) L −1  2 
 9 s − 16 
 s +1 
(g) L −1  2 
 s + s + 1
𝑠 1 1
(h) If ℒ −1 {(𝑠2 +1)2} = 2 𝑡 sin 𝑡, find ℒ −1 {(𝑠2 +1)2}

Partial Fractions:
3𝑠+1
10. Find ℒ −1 {(𝑠−1)(𝑠2+1)}

Mohammad Abdul Halim, Assistant Professor in Mathematics


Page 186 of 249

Solution:
Given that,
3𝑠+1
ℒ −1 {(𝑠−1)(𝑠2 +1)}

3𝑠+1 𝐴 𝐵𝑠+𝐶
= 𝑠−1 + 𝑠2 +1
(𝑠−1)(𝑠2 +1)

⇒ 3𝑠 + 1 = 𝐴(𝑠 2 + 1) + (𝐵𝑠 + 𝐶)(𝑠 − 1) = (𝐴 + 𝐵)𝑠 2 + (−𝐵 + 𝐶)𝑠 + (𝐴 − 𝐶)


Equating similar terms of both sides
𝐴 + 𝐵 = 0, −𝐵 + 𝐶 = 3, 𝐴−𝐶 = 1
Solving 3 equations above, we get 𝐴 = 2, 𝐵 = −2, 𝐶 = 1.
3𝑠 + 1 2 −2𝑠 + 1 2 2𝑠 1
∴ = + = − +
(𝑠 − 1)(𝑠 2 + 1) 𝑠 − 1 𝑠2 + 1 𝑠 − 1 𝑠2 + 1 𝑠2 + 1
3𝑠+1 2 2𝑠 1
⇒ ℒ −1 {(𝑠−1)(𝑠2+1)} = ℒ −1 {𝑠−1} − ℒ −1 {𝑠2 +1} + ℒ −1 {𝑠2 +1} = 2𝑒 𝑡 − 2 cos 𝑡 + sin 𝑡 (As
desired)
𝑠2 +2𝑠+3
11. Find ℒ −1 {(𝑠2+2𝑠+2)(𝑠2 +2𝑠+5)}

Solution:
Here,
𝑠2 +2𝑠+3 𝐴𝑠+𝐵 𝐶𝑠+𝐷
(𝑠2 +2𝑠+2)(𝑠2 +2𝑠+5)
= (𝑠2 +2𝑠+2) + (𝑠2 +2𝑠+5)

⇒ 𝑠 2 + 2𝑠 + 3 = (𝐴𝑠 + 𝐵)(𝑠 2 + 2𝑠 + 5) + (𝐶𝑠 + 𝐷)(𝑠 2 + 2𝑠 + 2)


⇒ 𝑠 2 + 2𝑠 + 3 = (𝐴 + 𝐶)3 + (2𝐴 + 𝐵 + 2𝐶 + 𝐷)𝑠 2 + (5𝐴 + 2𝐵 + 2𝐶 + 2𝐷)𝑠 + (5𝐵 + 2𝐷)
Then equating similar terms of both sides, we get
𝐴 + 𝐶 = 0, 2𝐴 + 𝐵 + 2𝐶 + 𝐷 = 1, 5𝐴 + 2𝐵 + 2𝐶 + 2𝐷 = 2, 5𝐵 + 2𝐷 = 3
Solving, 𝐴 = 0, 𝐵 = 1/3, 𝐶 = 0, 𝐷 = 2/3
1 2
−1 𝑠2 +2𝑠+3 −1 3 −1 3
Thus, ℒ {(𝑠2 }=ℒ { (𝑠2 +2𝑠+2) } + ℒ { (𝑠2 +2𝑠+5) }
+2𝑠+2)(𝑠2 +2𝑠+5)

1 1 2 −1 1
= ℒ −1 { } + ℒ { }
3 (𝑠 + 1)2 + 1 3 (𝑠 + 1)2 + 4
1 2 1 1
= 3 𝑒 −𝑡 sin 𝑡 + 3 . 2 𝑒 −𝑡 sin 2𝑡 = 3 𝑒 −𝑡 {sin 𝑡 + sin 2𝑡}

(As desired)

Mohammad Abdul Halim, Assistant Professor in Mathematics


Page 187 of 249

H.W:
Find the following inverse Laplace transformation:
3𝑠+7
(a) ℒ −1 { 2 }
𝑠 −2𝑠−3
2𝑠2 −4
(b) ℒ −1 {(𝑠+1)(𝑠−2)(𝑠−3)}
−1 1 
(c) L  2 
 s − 5s + 6 
 s −1 
(d) L −1  2 
 s − 6 s + 25 
−1  s+2 
(e) L  2 
 s − 4 s + 13 
−1  11s 2 − 2s + 5 
(f) L  3 
 2s − 3s − 3s + 2 
2

−1

 2s 2 − 6s + 5 

(g) L  
 ( s − 1)( s − 2 )( s − 3) 
 

 3s + 1 

−1
(h) L  
 (
 ( s − 1) s + 1
2
) 


 1 

−1
(i) L  
 (
 ( s + 1) s + 2s + 2
2
) 


 s+4 

−1
(j) L  

2
(
 s( s − 1) s + 4) ) 


 s2 

−1
(k) L  2 2 2 2 
 (s + a ) s + b )
 ( ) 

The Convolution Theorem: If L −1


 f (s) = F (t ) and  L −1
g(s) = G(t ) then
t
L −1
 f (s).g(s) =  F (u).G(t − u)du = F * G .
0

Prove that 𝑭 ∗ 𝑮 = 𝑮 ∗ 𝑭.
𝑡
Proof. We have 𝐹 ∗ 𝐺 = ∫0 𝐹(𝑢)𝐺(𝑡 − 𝑢)𝑑𝑢; Letting 𝑡 − 𝑢 = 𝑣 or 𝑢 = 𝑡 − 𝑣, we have

Mohammad Abdul Halim, Assistant Professor in Mathematics


Page 188 of 249

𝑡 𝑡

= ∫ 𝐹(𝑡 − 𝑣)𝐺(𝑣)𝑑𝑣 = ∫ 𝐺(𝑣)𝐹(𝑡 − 𝑣)𝑑𝑣 = 𝐺 ∗ 𝐹


0 0
This shows that the convolution of 𝐹 and 𝐺 obeys the commutative law of algebra.
 1 
12. Using the convolution theorem obtain L −1  2 2 
.
 s( s + a ) 
Solution:
1 1
Let f ( s ) = and g ( s ) = 2
s s + a2
1 
 L −1  f ( s ) = L −1   = 1 = F (t ) and
s
 1  sin at
L −1  g ( s ) = L −1  2 2 
= = G (t )
s + a  a
Hence by the convolution theorem

t
L −1
 f (s).g(s) =  F (u).G(t − u)du
0

 1  t sin ( at − au )
 2 2  =  1.
−1
L du
 s( s + a )  0 a
t
1
sin ( at − au ) du
a 0
=

1  − cos ( at − au ) 
t

=  
a −a 0
1
 cos ( at − au )  0
t
= 2 
a
1
= 2 ( cos 0 − cos at )
a
1 − cos at
= (As desired)
a2

−1  1 
13. Show that L  2  = ?.
 s( s + 9) 

1 1
Solution:Let f ( s ) = and g ( s ) = 2
s s +9

Mohammad Abdul Halim, Assistant Professor in Mathematics


Page 189 of 249

 f ( s) = L −1 
−1 1
L  = 1 = F (t ) and
s

 g ( s) = L −1 
−1 1  −1  1  sin 3t
L =L  2 2= = G (t )
s +9 s +3 
2
3
Hence by the convolution theorem

t
L −1
 f (s).g(s) =  F (u).G(t − u)du
0

 1  t sin ( 3t − 3u )
 =  1.
−1
L  2 du
 s( s + 9)  0 3
t
1
sin ( 3t − 3u ) du
3 0
=

1  − cos ( 3t − 3u ) 
t

=  
3 −3 0
1
 cos ( 3t − 3u )  0
t
=
9
1
= ( cos 0 − cos 3t )
9
1 − cos 3t
= (As desired)
9
𝑠
14. Find ℒ −1 {(𝑠2+𝑎2)2 }.
Solution:
𝑠 𝑠 1 𝑠
We can write (𝑠2 +𝑎2 )2
= (𝑠2 +𝑎2) ∙ . Then since ℒ −1 {𝑠2 +𝑎2} = cos 𝑎𝑡
𝑠2 +𝑎2
1 sin 𝑎𝑡
and ℒ −1 {𝑠2 +𝑎2 } = .
𝑎
𝑠 𝑡 sin 𝑎(𝑡−𝑢)
By the convolution Theorem, ℒ −1 {(𝑠2+𝑎2 )2} = ∫0 cos 𝑎𝑢 ∙ 𝑑𝑢
𝑎
𝑡
1
= ∫ cos 𝑎𝑢 (sin 𝑎𝑡 cos 𝑎𝑢 − cos 𝑎𝑡 sin 𝑎𝑢) 𝑑𝑢
𝑎
0
𝑡 𝑡
1 1
= sin 𝑎𝑡 ∫ cos 2 𝑎𝑢 𝑑𝑢 − cos 𝑎𝑡 ∫ sin 𝑎𝑢 cos 𝑎𝑢 𝑑𝑢
𝑎 𝑎
0 0
𝑡 𝑡
1 1
= sin 𝑎𝑡 ∫(1 + cos 2𝑎𝑢)𝑑𝑢 − cos 𝑎𝑡 ∫ sin 2𝑎𝑢 𝑑𝑢
2𝑎 2𝑎
0 0
1 sin 2𝑎𝑢 𝑡 1 cos 2𝑎𝑢 𝑡
= sin 𝑎𝑡 [𝑢 + ] − cos 𝑎𝑡 [− ]
2𝑎 2𝑎 0 2𝑎 2𝑎 0

Mohammad Abdul Halim, Assistant Professor in Mathematics


Page 190 of 249

1 sin 2𝑎𝑡 sin 𝑎𝑡 1 cos 2𝑎𝑡 1


= 2𝑎 [𝑡 sin 𝑎𝑡 + ] − 2𝑎 cos 𝑎𝑡 [− + 2𝑎]
2𝑎 2𝑎
1
= 2𝑎 [𝑡 sin 𝑎𝑡 − cos 𝑎𝑡 + cos 2𝑎𝑡 cos 𝑎𝑡 + sin 2𝑎𝑡 sin 𝑎𝑡]
1
= 2𝑎 [𝑡 sin 𝑎𝑡 − cos 𝑎𝑡 + cos (2𝑎𝑡 − 𝑎𝑡)
1
= 2𝑎 [𝑡 sin 𝑎𝑡 − cos 𝑎𝑡 + cos 𝑎𝑡]
1
= 2𝑎 𝑡 sin 𝑎𝑡 (As desired)
1
15. Find ℒ −1 {𝑠2 (𝑠+1)2}.
Solution:
1 1
We have ℒ −1 {𝑠2 } = 𝑡, ℒ −1 {(𝑠+1)2} = 𝑡𝑒 −𝑡 . then by the convolution theorem,
𝑡
1
ℒ −1 { 2 } = ∫(𝑢 𝑒 −𝑢 )(𝑡 − 𝑢)𝑑𝑢
𝑠 (𝑠 + 1)2
𝑜
𝑡

= ∫(𝑢𝑡 − 𝑢2 )𝑒 −𝑢 𝑑𝑢
𝑜
= [(𝑢𝑡 − 𝑢2 )(−𝑒 −𝑢 ) − (𝑡 − 2𝑢)(𝑒 −𝑢 ) + (−2)(−𝑒 −𝑢 )]𝑡0
= 𝑡𝑒 −𝑡 + 2𝑒 −𝑡 + 𝑡 − 2
(As desired)
H.W:
Using the convolution theorem calculate the followings:
𝑠
(a) ℒ −1 {(𝑠+2)2(𝑠−2)}
1
(b) ℒ −1 {(𝑠+1)(𝑠2 +1)}
𝑠2
(c) ℒ −1 {(𝑠2 +4)2}

−1  1 
(d) L  2 3
 ( s + 1) 
−1  1 
(e) L  2 2 2 
 s (s − a ) 
−1

 1 

(f) L  
 ( s+ 1) ( s + 1) 
2
 

Laplace Transforms Of Integrals.


∞ 𝐹(𝑡)
If ℒ −1 {𝑓(𝑠)} = 𝐹(𝑡) then ℒ −1 {∫𝑠 𝑓(𝑢) 𝑑𝑢} = 𝑡

Mohammad Abdul Halim, Assistant Professor in Mathematics


Page 191 of 249




1 


−1
16. Find L du 
 s u(u + 1) 
 
Solution:
Given that,



1 


−1
L du 
 s u(u + 1) 
 
1 1 −1 1 1
Let f (u ) = = + = −
u(u + 1) u u + 1 u u + 1
Replacing u by s we get,
1 1
f (s) == −
s s +1
Taking Inverse Laplace transformation on both-sides, we get
1   1 
L −1  f (s) = L −1   − L −1  
s  s + 1
F (t ) = 1 − e−t
From Inverse Laplace transformation of Integral we have,
∞ 𝐹(𝑡)
If ℒ −1 {𝑓(𝑠)} = 𝐹(𝑡) then ℒ −1 {∫𝑠 𝑓(𝑢) 𝑑𝑢} = 𝑡
Using this property, we get



1 
 1 − e−t
L −1  du  = (As desired)
 s u(u + 1) 
  t
  1
 1 2 u   
  2
−1
17. Obtain L + − + 2  du 
s  u − 4 u u − 2 u + 4  
 
Solution:
Given that,
 
  1 1 2 u   
  2
−1
L + − + 2  du 
s  u − 4 u u − 2 u + 4  
 
1 1 2 u
Let f (u ) = 2 + − + 2
u −4 u u−2 u +4
Replacing u by s we get,
1 1 2 s
f (s) = 2 + − + 2
s −4 s s−2 s +4
Taking Inverse Laplace transformation on both-sides, we get
 1  −1  1  −1  2  −1  s 
L −1  f (s) = L −1  2 + L   − L  +L  2 2
s − 4 s s − 2 s + 2 
sinh 2t
F (t ) = + 1 − 2e 2t + cos 2t
2

Mohammad Abdul Halim, Assistant Professor in Mathematics


Page 192 of 249

From Inverse Laplace transformation of Integral we have,


∞ 𝐹(𝑡)
If ℒ −1 {𝑓(𝑠)} = 𝐹(𝑡) then ℒ −1 {∫𝑠 𝑓(𝑢) 𝑑𝑢} = 𝑡
Using this property, we get
sinh 2t
 + 1 − 2e2t + cos 2t
u  

  1 1 2 
L −1    2 + − + 2  du  =
2
s  u − 4 u u − 2 u + 4  
  t
  1
 1 2 u    sinh 2t + 2 − 4e2t + 2cos 2t
  2
−1
L + − + 2  =
du (As
s  u − 4 u u − 2 u + 4  
  2t
desired)
H.W:
Evaluate the followings:
  2
 1 3 u   
(a) L −1   2 + 3+ 2 + 2  du 
s  u + 4 u u − 2 u + 4  
 

 5 


−1
(b) L du 
 s (u + 1)(u + 5u + 6) 
2
 

 1 


−1
(c) L du 
 s (u − 3)(u − 5u + 6) 
2
 

 u −1 

 2
−1
(d) L du 
 s u − 6u + 25 
 



2u 2 − 6u + 5 


−1
(e) L du 
 s ( u − 1)( u − 2 )( u − 3) 
 

Applications to Differential Equations


One of the applications of Laplace transformation to convert the initial value problems and boundary
problems into an algebraic equation that means solve the differential equations.
Let us consider ℒ{𝑌(𝑡)} = 𝑦(𝑠) then we have the Laplace transformation of n-th derivative of 𝑌(𝑡) is as
follows:

L Y ( n ) (t ) = s n y ( s) − s n −1Y (0) − s n −2Y (0) − − sY n − 2 (0) − Y n −1 (0)

Method of Solving Differential Equation:


It is performed in four steps
1. First Take Laplace transformation on both-sides of the Differential
equation.
2. Put the Initial conditions into the resulting equations.
3. Express y in terms of S (i.e. Solve for the output variable.)

Mohammad Abdul Halim, Assistant Professor in Mathematics


Page 193 of 249

4. Finally taking inverse Laplace transform operator if needed on both-


sides and simplify it to get solution.

Note: Ordinary Differential equations with variable coefficients


A particular differential equation where the method proves useful is one in which the terms have the
form
𝑑𝑚
𝑡 𝑚 𝑦 (𝑛) (𝑡) the Laplace Transform of which is (−1)𝑚 𝑑𝑠𝑚 ℒ{𝑦 (𝑛) (𝑡)}.

Worked Out Problems on ODE


Problem 01: Solve Y  − 2Y = e3t , Y (0) = −5 using Laplace transformation.
Solution:
Given IVP is
Y  − 2Y = e3t , Y (0) = −5
Taking the Laplace Transforms of both sides, we get

L Y  − 2L Y  = L e3t 

1
( sy ( s) − Y (0) ) − 2 y ( s) =
s −3
1
sy ( s ) + 5 − 2 y ( s ) = [Using initial value]
s −3
1
sy ( s ) − 2 y ( s ) = −5
s −3
1 − 5s + 15
( s − 2 ) y(s) =
s −3
16 − 5s
( s − 2 ) y(s) =
s −3
16 − 5s
y ( s) =
( s − 2 )( s − 3)
−6 1
y(s) = +
s −2 s −3
1 6
y(s) = −
s −3 s −2

 y ( s) = L −1 
−1 1  −1  6 
Taking Inverse we obtain the desired results, L −L  
 s − 3 s − 2

Mohammad Abdul Halim, Assistant Professor in Mathematics


Page 194 of 249

Y (t ) = e3t − 6e2t (As desired)

Problem 02: 𝒀′′ + 𝒀 = 𝒕, 𝒀(𝟎) = 𝟏, 𝒀′ (𝟎) = −𝟐


Solution: Taking the Laplace Transforms of both sides of the differential equations and using the
given conditions, we have
ℒ{𝑌 ′′ } + ℒ{𝑌} = ℒ{𝑡}
1
⇒ 𝑠 2 𝑦 − 𝑠𝑌(0) − 𝑌 ′′ (0) + 𝑦 =
𝑠2
1
⇒ 𝑠2𝑦 − 𝑠 + 2 + 𝑦 =
𝑠2
1
⇒ (𝑠 2 + 1)𝑦 = +𝑠−2
𝑠2
1 𝑠 2
⇒ 𝑦 = ℒ{𝑌} = + − 2
𝑠 2 (𝑠 2 + 1) (𝑠 2 + 1) (𝑠 + 1)
1 𝑠 3
= ℒ −1 { 2 } + ℒ −1 { 2 } − ℒ −1 { 2 }
𝑠 𝑠 +1 𝑠 +1
= 𝑡 + cos 𝑡 − 3 sin 𝑡 (As desired)

Problem 03 : 𝒀′′′ − 𝟑𝒀′′ + 𝟑𝒀′ − 𝒀 = 𝒕𝟐 𝒆𝒕 , 𝒀(𝟎) = 𝟏, 𝒀′ (𝟎) = 𝟎, 𝒀′′ (𝟎) = −𝟐.


Solution: ℒ{𝑌 ′′′ } − 3ℒ{𝑌 ′′ } + 3ℒ{𝑌 ′ } − ℒ{𝑌} = ℒ{𝑡 2 𝑒 𝑡 }
⇒ {𝑠 3 𝑦 − 𝑠 2 𝑌(0) − 𝑠𝑌 ′ (0) − 𝑌 ′′ (0)} − 3{𝑠 2 𝑦 − 𝑠𝑌(0) − 𝑌 ′ (0)} + 3{𝑠𝑦 − 𝑌(0)} − 𝑦
2
=
(𝑠 − 1)3
2
Thus (𝑠 3 − 3𝑠 2 + 3𝑠 − 1)𝑦 − 𝑠 2 + 3𝑠 − 1 = (𝑠−1)3

2
(𝑠 − 1)3 𝑦 − 𝑠 2 + 3𝑠 − 1 =
(𝑠 − 1)3
𝑠 2 − 3𝑠 + 1 2 (𝑠 − 1)2 − 𝑠 2
⇒𝑦= 3
+ 6
= 3
+
(𝑠 − 1) (𝑠 − 1) (𝑠 − 1) (𝑠 − 1)6
(𝑠 − 1)2 𝑠 − 1 + 1 2 1 𝑠−1 1 2
⇒𝑦= − + = − − +
(𝑠 − 1)3 (𝑠 − 1)3 (𝑠 − 1)6 (𝑠 − 1) (𝑠 − 1)3 (𝑠 − 1)3 (𝑠 − 1)6
1 1 1 2
⇒𝑦= − 2
− 3
+
(𝑠 − 1) (𝑠 − 1) (𝑠 − 1) (𝑠 − 1)6
1 1 1 2
⇒ 𝑌 = ℒ −1 { } − ℒ −1 { 2
} − ℒ −1 { 3
} + ℒ −1 { }
(𝑠 − 1) (𝑠 − 1) (𝑠 − 1) (𝑠 − 1)6

Mohammad Abdul Halim, Assistant Professor in Mathematics


Page 195 of 249

1 1
= 𝑒 𝑡 − 𝑡𝑒 𝑡 − 2 𝑡 2 𝑒 𝑡 + 60 𝑡 5 𝑒 𝑡 (As desired)
𝝅
Problem 04: 𝒀′′ + 𝟗𝒀 = 𝐜𝐨𝐬 𝟐𝒕 if 𝒀(𝟎) = 𝟏, 𝒀 ( 𝟐 ) = −𝟏.

Solution: Since 𝑌 ′ (0) is not known, Let 𝑌 ′ (0) = 𝑐. Then ℒ{𝑌 ′′ } + 9ℒ{𝑌} = ℒ{cos 2𝑡}
𝑠
⇒ 𝑠 2 𝑦 − 𝑠𝑌(0) − 𝑌 ′ (0) + 9𝑦 =
𝑠2 + 4
𝑠
⇒ (𝑠 2 + 9)𝑦 − 𝑠 − 𝑐 =
𝑠2 +4
𝑠+𝑐 𝑠 𝑠 𝑐 𝑠 𝑠
⇒𝑦= 2
+ 2 2
= 2 + 2 + 2
− 2
(𝑠 + 9) (𝑠 + 4)(𝑠 + 9) (𝑠 + 9) (𝑠 + 9) 5(𝑠 + 4) 5(𝑠 + 9)
4 𝑠 𝑐 𝑠
⇒𝑦= ( 2 )+ 2 +
5 𝑠 +9 (𝑠 + 9) 5(𝑠 2 + 4)
4 𝑠 𝑐 𝑠 4 𝑐 1
Thus 𝑌 = ℒ −1 {5 (𝑠2 +9) + (𝑠2 +9) + 5(𝑠2 +4)} = 5 cos 3𝑡 + 3 sin 3𝑡 + 5 cos 2𝑡

𝜋 𝑐 1 12
To determine 𝑐, we have 𝑌 ( 2 ) = −1 ⇒ 0 − 3 − 5 = −1 ⇒ 𝑐 = .
5
4 4 1
Then, 𝑌 = 5 cos 3𝑡 + 5 sin 3𝑡 + 5 cos 2𝑡 (As desired)

H.W:

(a) 𝑌"(𝑡) + 9𝑌 = 40𝑒 𝑡 , 𝑌(0) = 5, 𝑌′(0) = −2


(b) 𝑌 ′′ + 𝑌 = 𝑡, 𝑌(0) = 1, 𝑌 ′ (0) = −2 ; Solution: 𝑌 = 𝑡 + cos 𝑡 − 3 sin 𝑡
(c) 𝑌 ′′ − 3𝑌 ′ + 2𝑌 = 4𝑒 2𝑡 , 𝑌(0) = −3, 𝑌 ′ (0) = 5; Solution: 𝑌 = −7𝑒 𝑡 + 4𝑒 2𝑡 + 4𝑡𝑒 2𝑡
1
(d) 𝑌 ′′ + 2𝑌 ′ + 5𝑌 = 𝑒 −𝑡 sin 𝑡 , 𝑌(0) = 0, 𝑌 ′ (0) = 1; Solution: 𝑌 = 3 𝑒 −𝑡 (sin 𝑡 +
sin 2𝑡)
(e) 𝑌 ′′ − 3𝑌 ′ + 2𝑌 = 4𝑡 + 12𝑒 −𝑡 , 𝑌(0) = 6, 𝑌 ′ (0) = −1; Solution: 𝑌 = 3𝑒 𝑡 − 2𝑒 2𝑡 +
2𝑡 + 3 + 2𝑒 −𝑡
(f) 𝑌 ′′ + 𝑌 = 8 cos 𝑡 , 𝑌(0) = 1, 𝑌 ′ (0) = −1; Solution: 𝑌 = cos 𝑡 − sin 𝑡 + 4𝑡 sin 𝑡

Partial Differential Equations


• Given the Function 𝑈(𝑥, 𝑡) defined for 𝑎 ≤ 𝑥 ≤ 𝑏, 𝑡 > 0. Then
𝜕𝑈 𝜕𝑈 𝑑𝑢
(a) ℒ { 𝜕𝑡 } = 𝑠(𝑥, 𝑠) − 𝑈(𝑥, 0) (b) ℒ { 𝜕𝑥 } = 𝜕𝑥 ; assuming suitable restrictions on 𝑈 =
𝑈(𝑥, 𝑡).
𝜕2𝑈 𝜕2𝑈 𝑑2 𝑢
(c) ℒ { 𝜕𝑡 2 } = 𝑠 2 𝑢(𝑥, 𝑠) − 𝑠𝑈(𝑥, 0) − 𝑈𝑡 (𝑥, 0) (d) ℒ { 𝜕𝑥 2 } = 𝑑𝑥 2
𝜕𝑈
where𝑈𝑡 (𝑥, 0) = | and 𝑢 = 𝑢(𝑥, 𝑠) = ℒ{𝑈(𝑥, 𝑡)}.
𝜕𝑡 𝑡=0

Mohammad Abdul Halim, Assistant Professor in Mathematics


Page 196 of 249

𝝏𝑼 𝝏𝑼
01. Find the solution of 𝝏𝒙 = 𝟐 𝝏𝒕 + 𝑼, 𝑼(𝒙, 𝟎) = 𝟔 𝒆−𝟑𝒙 which is bounded for
𝒙 > 0, 𝑡 > 0, using Laplace Transform.
Solution: Taking the Laplace transform of the given partial differential equation with respect
to 𝑡, we get
𝑑𝑢 𝑑𝑢
= 2{𝑠𝑢 − 𝑈(𝑥, 0)} + 𝑢 or 𝑑𝑥 − (2𝑠 + 1)𝑢 = −12 𝑒 −3𝑥 …………(1)
𝑑𝑥
which is an ordinary differential equation.
To solve (1) multiply both sides by the integrating factor 𝑒 ∫ −(2𝑠+1)𝑑𝑥 = 𝑒 −(2𝑠+1)𝑥 .
Then (1) can be written as
𝑑
{𝑢𝑒 −(2𝑠+1)𝑥 } = −12 𝑒 −(2𝑠+4)𝑥
𝑑𝑥
−12 6
⇒ 𝑢𝑒 −(2𝑠+1)𝑥 = −12 ∫ 𝑒 −(2𝑠+4)𝑥 𝑑𝑥 = 𝑒 −(2𝑠+4)𝑥 + 𝑐 = 𝑒 −(2𝑠+4)𝑥 + 𝑐
−2(𝑠 + 2) (𝑠 + 2)
6
⇒𝑢= 𝑒 −3𝑥 + 𝑐𝑒 (2𝑠+1)𝑥
(𝑠 + 2)
Now since 𝑈(𝑥, 𝑡) must be bounded as 𝑥 → ∞, we must have 𝑢(𝑥, 𝑠) also bounded as 𝑥 → ∞
and it follows that we must choose 𝑐 = 0. Then
6
𝑢 = ℒ{𝑈(𝑥, 𝑡)} = 𝑒 −3𝑥
(𝑠 + 2)
6
⇒ 𝑈(𝑥, 𝑡) = ℒ −1 { 𝑒 −3𝑥 } = 6𝑒 −2𝑡 𝑒 −3𝑥 = 6𝑒 −2𝑡−3𝑥
(𝑠 + 2)
which is the required solution.

Mohammad Abdul Halim, Assistant Professor in Mathematics


Page 197 of 249

Mohammad Abdul Halim, Assistant Professor in Mathematics


Page 198 of 249

FOURIER TRANSFORMATION
The Fourier Transform is a tool that breaks a waveform (a function or signal) into an alternate
representation, characterized by sine and cosines. The Fourier Transform shows that any waveform can be
re-written as the sum of sinusoidal functions.

Also it is an important image processing tool which is used to decompose an image into its sine
and cosine components. The output of the transformation represents the image in the Fourier or
frequency domain, while the input image is the spatial domain equivalent. In the Fourier domain
image, each point represents a particular frequency contained in the spatial domain image.

The Fourier Transform is used in a wide range of applications, such as image analysis, image
filtering, image reconstruction and image compression.

Definition (Complex Integral):


Fourier’s complex integral is defined as of the following form:
 
1
f ( x) =
2 
−  −
f (u )ei (x −u) dud

And Fourier sine and cosine integral are as of the following form:
 
2
f ( x) =
  sin uxdu  f (t )sin utdt
0 0
And
 
2
f ( x) =
  cos uxdu  f (t ) cos utdt
0 0
❖ Fourier Transform (Complex Fourier Transform):
If a function F ( x ) is defined, continuous, integrable and piecewise smooth on ( −,  ) then
Fourier transform of F ( x ) is denoted by f ( ) and is defined as


F  F ( x) = e
− inx
F ( x)dx = f (n)
−

Inverse Fourier Transform:


The inverse transform of f ( ) , F ( x ) is defined by the following formula

1
F −1  f (n) = e
i x
f (n)dn = F ( x)
2 −
1
Remember!!!!! The constants 1 and in the above definitions of Fourier Transform and
2
1
Inverse Fourier Transform could be replaced by any two constants whose product is .
2

Mohammad Abdul Halim, Assistant Professor in Mathematics


Page 199 of 249

So, some Mathematician write then transforms are as of the following forms:
 
1 1
e e
− inx − inx
1. f (n) = F ( x)dx or f (n) = F ( x)dx
2 − 2 −
 
1
F ( x) = e F ( x) = e
inx inx
2. f (n)dn or f (n)dn
− 2 −

Note: Some Authors also define the Fourier transform in the form
 
1
F  F ( x) = e
inx
F ( x)dx = f (n) and then F −1  f (n) = e
− i x
f (n)dn = F ( x)
−
2 −

❖ Fourier Sine Transformation (Infinite):


If F( x) is an odd function, then its Fourier sine transformation is defined as follows:


Fs  F ( x) =  F ( x)sinn x dx = f s ( n )
0

And Inverse Fourier sine transformation is



2
F ( x) =
  f ( n ) sinn x dn
0
s

Remember!!!!!

2
The constants 1 and in the above definitions of Fourier sine Transform and Inverse Fourier

2
sine Transform could be replaced by any two constants whose product is .

So some Mathematician write then Fourier sine transforms are as of the following forms:

 
2 2
f s (n) =
  F ( x)sinn x dx
0
or F ( x) =
  f ( n ) sinn x dn
0
s

❖ Fourier Cosine Transformation (Infinite):


If F( x) is an even function, then its Fourier cosine transformation is defined as follows:


Fc  F ( x) =  F ( x) cosn x dx = f c ( n )
0

And inverse Fourier sine transformation is



2
F ( x) =
  f ( n ) cosn x d
0
c

Mohammad Abdul Halim, Assistant Professor in Mathematics


Page 200 of 249

Remember!!!!!

2
The constants 1 and in the above definitions of Fourier cosine Transform and Inverse Fourier

2
sine Transform could be replaced by any two constants whose product is .

So some Mathematician write then Fourier cosine transforms are as of the following forms:

 
2 2
f c (n) =
  F ( x) cosn x dx
0
or F ( x) =
  f ( n ) cosn x dn
0
c

❖ Finite Fourier Transform(Complex Form):


If a function F ( x ) is defined, continuous, integrable and piecewise smooth on ( −l , l ) then

Finite Fourier transform of F ( x ) is denoted by f (n) and is defined as


l i n x

f ( n) =  F ( x) e
−l
l
dx

And the Inverse Finite Fourier transform is defined as


i n x
1 
F ( x) = 
2l n = − 
f ( n ) e l

❖ Finite Fourier Sine Transform:


Let F ( x ) is a function which is section-ally continuous over the interval ( o, l ) ,then the Finite
Fourier Sine transform of F ( x ) on this interval is defined as
n x
l
f s (n) =  F ( x)sin dx , where n is an integer.
0
l
And Inverse Finite Fourier Sine transform is given by
2  n x
F ( x) = 
l n =1
f s (n) sin
l
2
Remark: The factor may be associated with either the transformation or the inverse of the
l
2
transformation or the factor may be associated with both the transform and the inverse.
l
❖ Finite Fourier Cosine Transform:
Let F ( x ) is a function which is section-ally continuous over the interval ( o, l ) ,then the Finite
Fourier cosine transform of F ( x ) on this interval is defined as

Mohammad Abdul Halim, Assistant Professor in Mathematics


Page 201 of 249

n x
l
fc (n) =  F ( x) cos dx , where n is an integer.
0
l
And Inverse Finite Fourier cosine transform is given by
f c (0) 2  n x
F ( x) = +  f c (n) cos
l l n =1 l
2
Remark: The factor may be associated with either the transformation or the inverse of the
l
2
transformation or the factor may be associated with both the transform and the inverse.
l
❖ Properties of Fourier Transform:
• Linearity Property:
ℱ [ aF(x)+ bG(x) ] = a ℱ [ F(x) ] + b ℱ [ G(x) ]
• Change of scale Property:
1 n
If F  F ( x) = f (n) then F  F (a x)  = f 
a a
• Time Shifting Property:
If F  F ( x) = f (n) then F  F ( x − a) = ei a f ( n ) = ei na F  F ( x)
• Frequency Shifting Property:
If F  F ( x) = f (n) then F einx F ( x)  = f ( n + a )
• Derivative property:
F  F ( x) = f (n) F  F n ( x ) = ( −in ) f (n) where n  1
n
If
• Frequency differentiation property:
dn
F  F ( x) = f (n) F  x F ( x )  = ( −i )
n
If
n
f (n) where n  1
d n
• Modulation Property:

F  F ( x) = f (n) then F  F ( x) sin ax  =


1
If  f ( n + a ) − f (n − a ) 
2
and
1
F  F ( x) cos ax  =  f ( n + a ) + f (n − a ) 
2
• Symmetry Property:
If F  F ( x) = f (n) then F  f (n) = 2 F ( − x )
• Fourier Transform of Integral:
x  f (n)
If F  F ( x) = f (n) then F   F ( x) dx  = .
a  ( −in )
• Convolution Property:
If two function F ( x ) and G ( x ) are defined on (-∞, ∞), then the convolution F ( x ) and G ( x ) is

denoted by F ( x) *G(x) and is defined by F ( x) *G(x) =  F ( x) G(x − u) du .


−

Mohammad Abdul Halim, Assistant Professor in Mathematics


Page 202 of 249

Mathematical Problems
Fourier Transformation:
x2

Problem 01: Find the Fourier Transform/Complex Transform of F ( x) = e 2
.
Solution:
Given Function is,
x2

F ( x) = e 2

We have

F F ( x) = e
− inx
F ( x)dx
−
 x2

e
− inx
= .e 2
dx
−
 x2
− inx −
= e
−
2
dx
 x2 
 −  + inx 
 2
= e
−
 
dx


1 2
(
x + 2 inx )
= e
−
2
dx


1 2
(
x + 2 inx + ( in ) −( in )
2 2
)
= e
−
2
dx


1

( x +in )2 + n2 
=
−
e 2
dx

 1 n2
− ( x +in )2 −
=
−
e 2 2
dx

 1 n2
− ( x +in )2 −
=
−
e 2
.e 2
dx

n2  1
− − ( x +in )2
=e 2
e
−
2
dx

Say
1
2
( )
x +in = y such that
1
2
( )
1+ 0 dx = dy  dx = 2 dy

Limit Change:
When x = −  then y = − 
When x =  then y = 
n2  1
− − ( x +in )2
Therefore F F ( x) = e 2
e 2
dx
−

Mohammad Abdul Halim, Assistant Professor in Mathematics


Page 203 of 249

2
n2   1 
− −  ( x +in )
=e 2

−
e  2 
dx

n2 

e ( )

− y2
=e 2
2 dy
−
n2 

e ( )

− y2
=e 2
2 dy
−
n2 

e
− y2
= 2e 2
dy
−

  a 
2 f ( x) dx , f ( x) even 
n2 

a

= 2e  2 e − y dy  f ( x) dx =  0
2
2
 
0  −a  , f ( x) odd 
  0

n2
  

= 2e  2  e dx =
−x 2

 
2
2  0
2 
n2

= 2 e 2

n2

= 2 e 2

n2

Finally, the Fourier Transform or complex transform of F ( x ) is = 2 e 2
.
(As desired)
Problem 02: Find the Complex Transform of F ( x) = e − a x where a  0 and −  x   .
Solution:
Given Function is,
−a x
F ( x) = e
We have

F F ( x) = e
− inx
F ( x)dx
−

e
− inx −a x
= .e dx
−
0 

 dx +  e − inx . e
−a x −a x
= e − inx . e dx
− 0
0
− a (− x)

  x , x  0
=  e − inx
.e dx +  e − inx . e − a ( x ) dx  x = 
− 0  − x , x  0
0 

e . e ax dx +  e − inx . e − ax dx
− inx
=
− 0

Mohammad Abdul Halim, Assistant Professor in Mathematics


Page 204 of 249

0 

e dx +  e − ax − inx dx
ax − inx
=
− 0
0 
( a − in ) x
= e dx +  e −( a + in ) x dx
− 0

 e ( a − in ) x   e −( a + in ) x 
0

=  +  
 a − in  −  − ( a + in )  0
1  ( a − in ) x  0 1  −( a + in ) x  
= e −
 −  a + in  e
a − in  0

=
1
a − in
( e 0 − e − ) −
1
a + in
( e− − e 0 )
=
1
a − in
( e 0 − e − ) −
1
a + in
( e − − e 0 )

1 1
= (1 − 0 ) − ( 0 − 1)
a − in a + in
1 1
= +
a − in a + in
a + in + a − in
=
( a − in )( a + in )
2a
=
( a − in )( a + in )
2a
=
( a − in )( a + in )
2a
= 2
a + n2
2a
Finally, the Fourier Transform or complex transform of F ( x ) is = .
a + n2
2

(As desired)
H.W:
Find the Fourier Transform or Complex Transform of the followings:
 x 2 , x  a
1. Find the Fourier Transform of F ( x) =  .
0 , x  a
1 , x  a
2. Find the Fourier Transform of F ( x) = 
0 , x  a
1 − x 2 , x  1
3. Find the Fourier Transform of F ( x) = 
0 , x 1

Mohammad Abdul Halim, Assistant Professor in Mathematics


Page 205 of 249

0 , x  a
 1  eibn − eian 
4. Show that the Fourier transform of F ( x) = 1 , a  x  b is  .
0 , x  b 2  i 

 2
 , x a sinna
5. Show that the Fourier transform of F ( x) =  2a is .
 0 na
 , x a
1
6. Find the Fourier transform of F ( x) = .
x
Fourier Sine/Cosine Transformation:

Problem 03: Find the Fourier Cosine and sine transform of F ( x) = e− ax , 0  x   .


Solution:
Given Function is,
F ( x) = e− ax , 0  x  
We have,

Fc  F ( x) =  F ( x) cos  x dx
0

=  e− ax cosn x dx
0

 e− ax ( −a cosn x + n sinn x ) 
= 
( −a ) + n 2
2
  0
1 
= e − ax ( −a cosn x + n sinnx ) 
2 
a +n 2 0

= 2
1
a +n 2 (
0 − e0 ( − a + 0 ) )
a
= 2
a + n2
a
Therefore, the Fourier cosine Transform of F ( x ) is = 2 .
a + n2
And
Again, we know that,

Fs  F ( x) =  F ( x)sin nx dx
0

=  e− ax sin nx dx
0

Mohammad Abdul Halim, Assistant Professor in Mathematics


Page 206 of 249


 e− ax ( −a sin nx −  cos nx ) 
= 
( −a ) + n 2
2
  0
1 
= e − ax ( −a sin nx − n cos nx ) 
( −a ) + n 2
2 0

1 
= e − ax ( −a sin nx − n cos nx ) 
2 
a +n 2 0

= 2
1
a +n 2
0 − e0 ( 0 − n ) ( )
n
= 2
a + n2
n
Therefore, the Fourier cosine Transform of F ( x ) is = 2 .
a + n2
(As desired)
− x2
Problem 04: Find the Fourier Cosine and sine transform of F ( x) = e ,0  x   .
Solution:
Given function is,
F ( x) = e− x ,0  x  
2

We have,

Fc  F ( x) =  F ( x) cos nx dx
0

=  e− x cos nx dx
2

0

I =  e− x cos nx dx
2
Say (i)
0
Considering differentiation with respect to n under integration we get,

dI d
=  e− x cos nx dx
2

dn dn 0

=
0
 − x2
n
e cos nx dx( )

=  e− x ( − x sin nx ) dx
2

0

=
1
20 (
−2 xe− x sin nx dx
2

)

1
2
(
d
)  
= sin nx  −2 xe− x dx −   ( sin nx )  −2 xe− x dx  dx 
2

 dx
2

 0
( )

Mohammad Abdul Halim, Assistant Professor in Mathematics


Page 207 of 249

1 
= sin nxe − x −  n cos nx e − x dx 
2 2

2 0
 
1 n
= sin nxe− x  −  cos nx e − x dx
2 2

2 0 2
0

1 n 
= sin nxe − x  − I
2

2   0 2
1 n
= 0− I
2 2
dI n
=− I
d 2
dI n
= − dn
I 2
Integrating both-sides, we get,
dI 1
 I
= −  ndn
2
1 n2
ln I = −  + ln C
2 2
n2
ln I = − + ln C
4
n2

ln I = ln e 4
+ ln C
n2

ln I = lnC e 4

n2

I = Ce 4
(ii)
Putting n = 0 in (i) and (ii) we get,
 

I =  e− x cos 0 dx =  e− x dx =
2 2

0 0
2
And

02

I = Ce 4
=C =
2
 −
n2
Now, I= e 4
2

 −
n2

e cos  x dx =
− x2
Therefore e 4

0
2
(As desired)

Mohammad Abdul Halim, Assistant Professor in Mathematics


Page 208 of 249

H.W:
Find the Fourier sine /cosine Transform of the followings:
1
1. Find the Fourier sine Transform of F ( x ) = .
x
1
2. Find the Fourier sine Transform of . F ( x) =
x
−2 x
3. Find the Fourier cosine Transform of F ( x) = e + 4e −3 x .
e− ax
4. Find the Fourier sine Transform of F ( x) = .
x
1 ,0  x  a
5. Find the Fourier sine and cosine Transform of F ( x) =  .
0 ,x  a
e− a
2 2
x
6. Find the Fourier Cosine transform of and hence evaluate Fourier sine transform of
xe − a
2 2
x
.
x
7. Find Fourier sine transform of .
1 + x2
1
8. Find Fourier cosine transform of .
1 + x2
− n
9. Find the inverse sine transform of e .
−mn
e
10. Find the inverse sine transform of .
n

Finite Fourier Transformation:

Problem 05: Find the Finite Fourier transform of F ( x) = x3 , −  x   .


Solution:
Given function is,
F ( x) = x3 , −  x  
We have,
l i n x

f ( n) =  F ( x) e
−l
l
dx

Now,
 i n x

f ( n) = 

x3 e 
dx

=  x e− inx dx
3



 3 e− inx e− inx e− inx 
= x  − 2x  + 2 
−in ( −in ) ( −in )  −
2 3


Mohammad Abdul Halim, Assistant Professor in Mathematics


Page 209 of 249


 x3e− inx 2 xe− inx 2e− inx 
= − + + 
 in n2 in3  −

 2 xe− inx 2e− inx x3e− inx 
= 2
+ 3
− 
 n in in  −
 2 e− in 2e− in  3e− in   2 ein 2ein  3e in 
= 2
+ − −− + 3 − 
 n in3 in   n2 in in 
2 e− in 2e− in  3e− in 2 ein 2ein  3e in
= + − + − 3 +
n2 in3 in n2 in in
− in in − in in 3 − in 3 in
2 e 2 e 2e 2e  e  e
= 2
+ 2
+ 3
− 3 − +
n n in in in in
2  3
= 2 ( e− in + ein ) + 3 ( e− in − ein ) + ( e in − e −in )
2
n in in
2 3
= 2 ( e− in + ein ) − 3 ( e in − e −in ) + ( e in − e −in )
2
n in in
2 3
= 2 ( e− in + ein ) − 3 ( e in − e −in ) + ( e in − e −in )
2
n in in
2 2 3
= 2 cosh n − 3 sinh n + sinh n (As desired)
n in in
H.W:
Find the Finite Fourier Transform of the followings:

1. Find the Finite Fourier transform of F ( x) = e


ax
, −  x   .
2. Find the Finite Fourier transform of F ( x) = x 2 , −  x   .

Finite Fourier sine/cosine Transformation:

Problem 06: Find the Finite Fourier sine and cosine transform of F ( x) = e
ax
,0  x   .
Solution:
Given function is,
F ( x) = eax , 0  x  
We have,
n x
l
f s (n) =  F ( x)sin dx
0
l
Now,

n x
f s (n) =  F ( x)sin dx
0


=  ea x sin nx dx
0

Mohammad Abdul Halim, Assistant Professor in Mathematics


Page 210 of 249


 e a x ( a sin nx − n cos nx ) 
= 
 a 2 + n2 0
1 
= 2 e a x ( a sin nx − n cos nx ) 
2 
a +n 0

2 
e a  ( a sin n − n cos n ) − e0 ( a sin 0 − n cos 0 )
1
= 2
a +n

2 
e a  ( 0 − n ) − ( 0 − n )
1
= 2
a +n
= 2
1
a + n2
−nea + n
(
n 1 − ea
= 2
)
a + n2
Again:
We have
n x
l
fc (n) =  F ( x) cos dx
0
l
Now,

n x
f c (n) =  ea x cos dx
0


=  ea x cos nx dx
0

 e a x ( a cos nx + n sin nx ) 
= 
 a 2 + n2 0
1 
= 2 e a x ( a cos nx + n sin nx ) 
2 
a +n 0

2 
e a  ( a cos n + n sin n ) − e0 ( a cos 0 + n sin 0 )
1
= 2
a +n

2 
e a  ( a cos n + 0 ) − ( a )
1
= 2
a +n
= 2
1
a + n2
( aea cos n − a )
= 2
a
a + n2
( ea cos n − 1)
a ( e a  cos n − 1)
=
a 2 + n2

=

a ea ( −1) − 1
n

a +n
2 2

(As desired)

Mohammad Abdul Halim, Assistant Professor in Mathematics


Page 211 of 249

H.W:
Find the Finite Fourier Transform of the followings:

1. Find the Finite Fourier sine and cosine transform of F ( x ) = 2 x, 0 


x4.
2. Find the Finite Fourier sine and cosine transform of F ( x) = sin mx, 0  x   .
3. Find the Finite Fourier sine and cosine transform of F ( x) = cos mx , 0  x   .
x
4. Find the Finite Fourier sine and cosine transform of F ( x ) = 1 − ,0  x   .

2
 x
5. Find the Finite Fourier cosine transform of  1 −  , 0  x   .
 
 
 1 , 0  x  2
6. Find the Finite Fourier sine and cosine transform of F ( x ) =  .
 −1 ,   x  
 2
 
 x , 0  x  2
7. Find the Finite Fourier sine and cosine transform of F ( x ) =  .
 − x ,   x  
 2
8. Find the Finite Fourier sine and cosine transform of F ( x) = x in (0, l ) .
3

Application of Fourier Transform


Finite Fourier Sine and Cosine Transform:
n x
l
1. Finite Fourier Sine Transform f s (n) =  F ( x)sin dx and Inverse Finite
0
l
2 
n x
Fourier Sine Transform is F ( x ) = 
l n =1
f s (n) sin
l
.

n x
l
2. Finite Fourier Cosine Transform f c (n) = F ( x) cos  0
l
dx and Inverse Finite

f c (0) 2 
n x
Fourier Cosine Transform is F ( x) = +  f c (n) cos .
l l n =1 l
Mathematical Problems
Problem 01: Use Finite Fourier Transform to solve the boundary value
U  2U
problem = 2 ;U (0, t ) = U (4, t ) = 0, U ( x, 0) = 2 x where 0  x  4, t  0 .
t x
Solution:
Given Partial Differential Equation is:

Mohammad Abdul Halim, Assistant Professor in Mathematics


Page 212 of 249

U  2U
= 2 ;U (0, t ) = U (4, t ) = 0, U ( x, 0) = 2 x where 0  x  4, t  0 .
t x
Taking Finite Fourier Sine Transform with l = 4 on both-sides of above
equation, we get

U n x  2U n x
4 4

0 t sin
4
dx = 
0
x 2
sin
4
dx (i)

n x
4


Say u (n, t ) = U ( x, t ) sin
0
4
dx (ii )

Differentiating with respect to ‘t‘we get


n x
4
du d
=  U sin dx
dt dt 0 4
U n x
4
= sin dx
0
t 4
 2U n x
4
=  2 sin dx using equation (i)
0
x 4
 n x U  n n x U
4 4
= sin .  − cos dx
 4 x  0 0 4 4 x
 U  n n x U
4
=  sin n .
 x
− 0 −
 4 0
 cos
4 x
dx

n n x U
4
= ( 0 − 0) −  cos dx
4 0 4 x
n n x U
4
=− 
4 0
cos
4 x
dx

n 
 n x 
4 4
−n n x 

=−  cos .U ( x, t )  −  sin U ( x, t ) dx 
4 
 4 0 0 4 4 

n  n x  n n x 
4
 
4
=−  cos
4   4
.U ( x, t )  +
0 4 0
 sin
4
U ( x, t ) dx 


n  n n x 
4
= −  ( cos n .U (4, t ) − U (0, t ) ) +
4 0
sin U ( x, t ) dx  [Using given conditions]
4  4 
n  n n x 
4
= −  0 +
4  4 0 sin
4
U ( x, t ) dx 

n
2 2 4
n x
=−
16 0 sin
4
U ( x, t ) dx

n 2 2
=− u
16

Mohammad Abdul Halim, Assistant Professor in Mathematics


Page 213 of 249

du n 2 2
 =− dt
u 16
This is a variable separable form.
Integrating on both-sides, we get
n 2 2
ln u = − t + ln C
16
n2 2t

ln u = ln e 16
+ ln C
n2 2t

ln u = ln Ce 16

n2 2t

u ( n, t ) = Ce 16
(iii )
Putting t = 0 in (iii) and (ii) we get
u ( n,0) = C
And
n x
4
u ( n, 0 ) =  sin U ( x, 0) dx
0
4
n x
4
 C =  2 x sin dx
0
4
n x
4
= 2  x sin dx
0
4
4
  −4 n x   −4 4 n x  
= 2  ( x )  cos  − (1)  . sin 
  n 4   n n 4   0
n x 4 x n x 
4
 16
= 2   2 2 sin − cos
n  4 n 4  0
 16 ( −1)n  
= 2   0 −  − ( 0 − 0 )
 n 
 
16 ( −1)
n

= −2 
n
32 ( −1)
n

=−
n
Now from equation (iii) we get
32 ( −1) n2 2t
n

u ( n, t ) = − ( −1)
n 16
e
n
32 ( −1)
n +1 n 2 2t

u ( n, t ) = ( −1)
n 16
e
n
Applying Inverse Fourier Sine Transform we get

Mohammad Abdul Halim, Assistant Professor in Mathematics


Page 214 of 249

2  n x
U ( x, t ) =  u (n, t ) sin
l n =1 l
2  32 ( −1)
n +1 n2 2t
− n x
U ( x, t ) =  ( −1)
n
16
e sin
4 n=1 n 4
16 ( −1)
n +1 n2 2t
 − n x
U ( x, t ) =  e 16
sin
n =1 n 4
Which is the required Solution. (As
desired)

V  2V
Problem 02: Solve = 2 subject to conditions V(0, t ) = 1, V( , t ) = 3 and
t x
V( x, 0) = 1 for
0  x   ,t>0.
Solution:
Given Partial Differential Equation is:
V  2V
= 2 , V(0, t ) = 1, V( , t ) = 3 and V( x, 0) = 1 for 0  x   ,t>0.
t x
Taking Finite Fourier Sine Transform with l =  on both-sides of above
equation, we get

 
V n x  2V n x
0 t sin

dx = 
0
x 2
sin

dx
 
V  2V
0 t sin nx dx = 0 x 2 sin nx dx (i )


Say v( n, t ) = V( x, t ) sin nx dx
0
(ii )

Differentiating with respect to ‘t‘we get



du d
dt dt 0
= V sin nx dx

V
= sin nx dx
0
t

 2V
= sin nx dx using equation (i)
0
x 2
 
 V  V
= sin nx.  −  n cos nx dx
 x  0 0 x

V
= ( 0 − 0 ) − n  cos nx dx
0
x

Mohammad Abdul Halim, Assistant Professor in Mathematics


Page 215 of 249


V
= − n  cos nx dx
0
x

 
= − n  cos nxV
. ( x, t ) 0 −  − n sin nxV dx 

 0 

 
= − n  cos nxV
. ( x, t )0 + n  sin nxV ( x, t ) dx 

 0 

 
= − n ( cos n .V ( , t ) − V (0, t ) ) + n  sin nxV ( x, t ) dx 
 0 

 
= − n ( 3cos n − 1) + n  sin nxV ( x, t ) dx 
 0 
= − n ( 3cos n −1) − n v 2

dv
+ n 2 v = − n ( 3cos n − 1)
dt
This is a linear differential equation
Integrating factor:
I .F = e 
pdx

I .F = e  = en t
n2 dt 2

Multiplying above equation by I.F we get,


dv n2t
e + n 2 ve n t = − n ( 3cos n − 1) e n t
2 2

dt
d
dt
( )
v.e n t = − n ( 3cos n − 1) e n t
2 2

v.en t =  − n ( 3cos n − 1) en t dt + C
2 2

v.en t = n (1 − 3cos n )  en t dt + C
2 2

2
en t
v.e = n (1 − 3cos n )  2 + C
n 2t

v.en t =
2 (1 − 3cos n )  en2t + C
n
v(n, t ) =
(1 − 3cos n ) + C  e− n2t (iii )
n
Putting t = 0 we get

v(n, 0) =
(1 − 3cosn  ) + C 1
n
v(n, 0) =
(1 − 3cosn  ) + C
n

From (ii)

Mohammad Abdul Halim, Assistant Professor in Mathematics


Page 216 of 249


v( n, 0) =  sin nx dx
0

 − cos nx  1 1 1 − cos n
= −  cos nx 0 = −  ( cos n − 1) =

v(n, 0) =  
 n 0 n n n
(1 − 3cosn  ) + C = 1 − cos n
n n
1 − cos n 1 − 3cosn  1 − cos n − 1 + 3cosn  2 cos n
C= − = =
n n n n
From (iii),we get

v(n, t ) =
(1 − 3cos n ) + 2 cos n  e− n t 2

n n

Therefore the Inverse Fourier Sine Transform we get


2  n x
V ( x, t ) = 
l n =1
v(n, t ) sin
l
2 
n x
=  v(n, t ) sin
 
n =1

2  (1 − 3cos n ) 2cos n − n t 

=  +  e  sin nx
2

 n=1  n n 
Which is the required Solution. (As
desired)
U  2U
Problem-03: Use Finite Fourier Transform to solve =2 2 ;
t x
U (0, t ) = 0, U (4, t ) = 0 ,
U ( x, 0) = 3sin  x − 2sin 5 x where 0 x4 , t 0.
U  2U
Solution: Given that, = 2 2 ; U (0, t ) = 0, U (4, t ) = 0 ,
t x
U ( x, 0) = 3sin  x − 2sin 5 x
where 0  x  4 , t  0 .
Taking Finite Fourier Sine Transform with l = 4 on both-sides of above
equation, we get

U n x  2U n x
4 4

0 t sin
4
dx = 2 
0
x 2
sin
4
dx

d n x  U n x
4 4 2
or ,  U sin dx  = 2  sin dx (i )
dt  0 4  0
x 2
4
n x
4

Putting u (n, t ) =  U ( x, t ) sin dx (ii )


0
4

Mohammad Abdul Halim, Assistant Professor in Mathematics


Page 217 of 249

Then Eq.(i) becomes,


 2U n x
4
du
= 2  2 sin dx
dt 0
x 4
4
 n x  2U d  n x   2U  
4  x 2   dx  4   x2 dx  dx 
= 2 sin dx −  sin
 0

 U n x n n x U 
4

= 2 sin − cos . dx
 x 4 4 4 x  0
 U n x  n U n x
4 4
= 2
 x
sin −
4  0 4  x cos
0
4
dx
4
n  n x U d  n x  U  
= 2 ( 0 − 0) − 2  cos  dx −    cos  dx  dx 
4  4 x  dx  4  x  0
4
n  n x  n n x  
=− U ( x, t ) cos 4 −   − 4 sin 4  .U ( x, t )dx 
2    0
n n x n n x 
4

=−
2 U ( x, t ) cos 4 + 4  U ( x, t )sin 4 dx 
0

n n x  n  n x
4

2 2 4
=−
2 U ( x, t ) cos 4  − 8
0
 U ( x, t )sin
0
4
dx

n n 2 2
=− U (4, t ) cos n − U (0, t ) cos 0 − u
2 8
n n2 2
= −  0  cos n − 0 1 − u
2 8
n 2 2
= 0− u
8
n 2 2
=− u
8
du n 2 2
 =− dt
u 8
This is a variable separable form.
Integrating on both-sides, we get
n 2 2
ln u = − t + ln C
8
n 2 2t

or , ln u = ln e 8
+ ln C
n t
2 2

or , ln u = ln Ce 8

n2 2t

 u ( n, t ) = Ce 8
(iii )

Mohammad Abdul Halim, Assistant Professor in Mathematics


Page 218 of 249

Applying Inverse Fourier Sine Transform we get


2  n x
U ( x, t ) = 
l n =1
u (n, t ) sin
l
n 2 2 t
2  − n x
=  Ce 8
sin
4 n =1 4
n 2 2t
 − n x
 U ( x, t ) =  Be 8
sin Taking B = C 2 (iv)
n =1 4
To satisfy the boundary condition U ( x,0) = 3sin  x − 2sin 5 x we have to consider
only first two terms i.e. n = 1, 2
So Eq.(iv) becomes,
n 2 2t
2 − n x
U ( x, t ) =  Be 8
sin
n =1 4
n12 2t n22 2t
− nx − n2 x
or , U ( x, t ) = B1e 8
sin 1 + B2e 8
sin (v )
4 4
Putting t = 0 in (iv) we get,
n x 2
U ( x, 0) =  B sin
n =1 4
nx n x
or , 3sin  x − 2sin 5 x = B1 sin 1 + B2 sin 2
4 4
Comparing both sides we have,
B1 = 3 , B2 = −2 , n1 = 4 , n2 = 20
Hence Eq. (v) becomes,
2 2t 50 2t
U ( x, t ) = 3e− sin  x − 2e− sin 5 x
Which is the required Solution. (As desired)

Try Yourself:
1. Prove that the solution of the boundary value problem
U  2U
= 3 2 ;U (0, t ) = U (2, t ) = 0, U ( x, 0) = x where 0  x  2, t  0 is
t x
4 ( −1)
n +1 3n2 2t
 − n x
U ( x, t ) =  e 4
sin .
n =1 n 2
2. Use finite Fourier Transforms to solve
U  2U
= 2 ;U (0, t ) = U ( , t ) = 0, U ( x, 0) = 2 x
t x
4 ( −1)
n +1

where 0  x   , t  0 is U ( x, t ) =  e− n t sin nx .
2

n =1 n
𝜕𝑈 𝜕2 𝑈
3. Use finite Fourier Transforms to solve =3 ; 𝑈(0, 𝑡) = 0, 𝑈(𝜋, 𝑡) = 0;
𝜕𝑡 𝜕𝑥 2

𝑈(𝑥, 0) = 𝑥 where 0 < 𝑥 < 𝜋, 𝑡 > 0 .

Mohammad Abdul Halim, Assistant Professor in Mathematics


Page 219 of 249

𝜕𝑈 𝜕2 𝑈
4. Use finite Fourier Transforms to solve = ; 𝑈𝑥 (0, 𝑡) = 0, 𝑈𝑥 (6, 𝑡) = 0;
𝜕𝑡 𝜕𝑥 2

𝑈(𝑥, 0) = 2𝑥 where 0 < 𝑥 < 6, 𝑡 > 0 .

U  2U
5. Solve = 2 , 0  x  6, t  0 given that U (0, t ) = U (6, t ) = 0 and
t x
1 , 0  x  3
U ( x, 0) =  .
0 ,3  x  6

Mohammad Abdul Halim, Assistant Professor in Mathematics


Page 220 of 249

Mohammad Abdul Halim, Assistant Professor in Mathematics


Page 221 of 249

Glossary
Periodic function: Periodic function: A function 𝑦 = 𝑓(𝑥) is called a periodic
function of period T if it satisfies the condition 𝑓(𝑥 + 𝑻) = 𝑓(𝑥) where T is least
positive real number. It means that the function 𝑓(𝑥) possess same values after
an interval T.
Example: 𝑆𝑖𝑛𝑥 is a periodic function with period 2π

Note:
1.The fundamental period of sinx, cosx, secx, cosecx is 2π and also tanx, cotx is
π.
2.The period of a function of the form 𝑦 = 𝐴 𝒕𝒓𝒊𝒈𝒐𝒏𝒐𝒎𝒆𝒕𝒓𝒊𝒄 𝒓𝒂𝒕𝒊𝒐𝒔𝑛 (𝐵𝑥 + 𝐶) is
obtained by dividing the of fundamental period of that function by the product
of coefficient of x and the power n.
𝑓𝑢𝑛𝑑𝑎𝑚𝑒𝑛𝑡𝑎𝑙 𝑝𝑒𝑟𝑖𝑜𝑑 𝑜𝑓 𝑡ℎ𝑒 𝑓𝑢𝑛𝑐𝑡𝑖𝑜𝑛
That is, 𝑃𝑒𝑟𝑖𝑜𝑑 = .
𝑛×𝐵

3. The period of the sum or subtraction of two functions is the LCM (Least
Common Multiple) of the individual periods.
Problem: Find the period of the function 𝑓(𝑥) = sin4 𝑥 + 𝑐𝑜𝑠 4 𝑥 .
Solution:
Given function is,
𝑓(𝑥) = sin4 𝑥 + 𝑐𝑜𝑠 4 𝑥
𝑝𝑒𝑟𝑖𝑜𝑑 𝑜𝑓 𝑐𝑜𝑠𝑥 2𝜋 𝜋
Now the period of the function sin4 𝑥 is = 𝑛× 𝑐𝑜𝑒𝑓𝑓𝑖𝑐𝑖𝑒𝑛𝑡 𝑜𝑓 𝑥 = 4×1 = 2 .
Again,
𝑝𝑒𝑟𝑖𝑜𝑑 𝑜𝑓 𝑐𝑜𝑠𝑥 2𝜋 𝜋
Now the period of the function 𝑐𝑜𝑠 4 𝑥 is = = 4×1 = .
𝑛× 𝑐𝑜𝑒𝑓𝑓𝑖𝑐𝑖𝑒𝑛𝑡 𝑜𝑓 𝑥 2
𝜋 𝜋 𝜋
Now LCM of and is =
2 2 2
𝜋
Therefore 𝑓(𝑥) is a periodic function with period 2 .
(As desired)
Note:
LCM of Numerators of the fractions
➢ LCM of Fractions =
HCF of Deno min ators of the fractions
HCF of Numerators of the fractions
➢ HCF of Fractions =
LCM of Deno min ators of the fractions
Even Function: A function 𝑓(𝑥) is said to be even function if 𝑓(− 𝑥) = 𝑓(𝑥).
For example: 𝑓(𝑥) = 𝑠𝑖𝑛4 𝑥 is an even function.
Odd Function: A function 𝑓(𝑥) is said to be odd function if 𝑓(− 𝑥) = − 𝑓(𝑥).
For example: 𝑓(𝑥) = 𝑠𝑖𝑛3 𝑥 is an odd function.
Drichlet’s Condition:
Let 𝑓(𝑥) satisfy the following conditions:

Mohammad Abdul Halim, Assistant Professor in Mathematics


Page 222 of 249

1. 𝑓(𝑥) is defined in the interval − 𝑙 < 𝑥 < 𝑙


2. 𝑓(𝑥) and 𝑓′(𝑥) are sectionally continuous in − 𝑙 < 𝑥 < 𝑙
3. 𝑓(𝑥 + 2𝑙) = 𝑓(𝑥), i.e. 𝑓(𝑥) is periodic with period 2𝑙.
Then at every point of continuity, we have
𝑎0 𝑛𝜋𝑥 𝑛𝜋𝑥
𝑓(𝑥) = + ∑∞
𝑛=1 (𝑎𝑛 cos + 𝑏𝑛 sin ) (1)
2 𝑙 𝑙
1 𝑙 𝑛𝜋𝑥
Where 𝑎𝑛 = 𝑙 ∫−𝑙 𝑓(𝑥) cos 𝑑𝑥 (2)
𝑙
1 𝑙 𝑛𝜋𝑥
and 𝑏𝑛 = ∫ 𝑓(𝑥) sin 𝑙 𝑑𝑥 (3)
𝑙 −𝑙
The series (1) with coefficients (2) and (3) is called the Fourier Series of 𝑓(𝑥).
The above conditions are often called Dirichlet conditions and are sufficient
(but not necessary) conditions for convergence of Fourier Series.

Fourier Series
Fourier series of a function is to express the given function by a combination of
cosine and sine functions. The Fourier series is named in honor of Jean-Baptiste
Joseph Fourier (1768–1830), who made important contributions to the study of
trigonometric series, after preliminary investigations by Leonhard Euler, Jean le
Rond d'Alembert, and Daniel Bernoulli.
Fourier introduced the series for the purpose of solving the heat equation in a
metal plate, publishing his initial results in his 1807 (Treatise on the propagation
of heat in solid bodies), and publishing his (Analytical theory of heat) in 1822.

Application:
1. Fourier Series is used in sound processing.
2. Fourier Series is used in signal processing.
3. Fourier Series is used in image processing.
4. Fourier series is used to decompose a waveform into component waves of
different frequencies and Amplitude, allowing identification of different
sources from background or random noise in a signal.
5. Fourier Series is used to solve BVP such as Heat Flow, Laplace equation
and Vibrating System

Sound Processing

Mohammad Abdul Halim, Assistant Professor in Mathematics


Page 223 of 249

Definition of Fourier Series:


Let f (x ) is defined in the interval (− l , l ) and is determined outside of this interval
by f ( x + 2l ) = f ( x) that means f (x ) is a periodic function with period 2l .
Then the trigonometric series
𝑎0 𝑛𝜋𝑥 𝑛𝜋𝑥
𝑓(𝑥) = + ∑∞
𝑛=1 (𝑎𝑛 cos + 𝑏𝑛 sin )
2 𝑙 𝑙
is called Fourier series of f (x ) with Fourier coefficients
1 𝑙 𝑛𝜋𝑥
𝑎𝑛 = 𝑙 ∫−𝑙 𝑓(𝑥) cos 𝑑𝑥
𝑙
1 𝑙 𝑛𝜋𝑥
and 𝑏𝑛 = ∫ 𝑓(𝑥) sin 𝑙 𝑑𝑥 with n=0,1, 2,……..,n.
𝑙 −𝑙

 Fourier series of Even and Odd Function:

󠇗 If 𝑓(𝑥) is even function of period 2𝑙, then



𝑎0 𝑛𝜋𝑥
𝑓(𝑥) = + ∑ 𝑎𝑛 cos (𝐴)
2 𝑙
𝑛=1
2 𝑙
where (i) 𝑎0 = 𝑙 ∫0 𝑓(𝑥)𝑑𝑥
2 𝑙 𝑛𝜋𝑥
(ii) 𝑎𝑛 = 𝑙 ∫0 𝑓(𝑥) cos 𝑙 𝑑𝑥 ; 𝑛 = 1,2,3, …
(iii) 𝑏𝑛 = 0.
This series(A) is called half-range Fourier Cosine series.
󠇗 If 𝑓(𝑥) is odd function of period 2𝑙, then

𝑛𝜋𝑥
𝑓(𝑥) = ∑ 𝑏𝑛 sin (𝐵)
𝑙
𝑛=1
where (i) 𝑎0 = 0;
(ii) 𝑎𝑛 = 0 ;
2 𝑙 𝑛𝜋𝑥
(iii) 𝑏𝑛 = 𝑙 ∫0 𝑓(𝑥) sin 𝑑𝑥 ; 𝑛 = 1,2,3, …
𝑙
This series(B) is called half-range Fourier Sine series.
 Fourier series on the range (−  ,  ).

𝑎0
Fourier series of 𝑓(𝑥) is 𝑓(𝑥) = + ∑∞
𝑛=1(𝑎𝑛 cos 𝑛𝑥 + 𝑏𝑛 sin 𝑛𝑥)
2
Where Fourier coefficients are as follows:
1 𝜋
𝑎𝑛 = 𝜋 ∫−𝜋 𝑓(𝑥) cos 𝑛𝑥 𝑑𝑥
1 𝜋
and 𝑏𝑛 = 𝜋 ∫−𝜋 𝑓(𝑥) sin 𝑛𝑥 𝑑𝑥 with n=0,1, 2,…….., n.
Note:

Mohammad Abdul Halim, Assistant Professor in Mathematics


Page 224 of 249

If 𝑓(𝑥) is defined on (0 ,  ) ,we extend f (x ) over (−  ,0) either as an even function


or as an odd function and also known as half range Fourier series. Or we find

the value of l by letting 2l =  − 0 then l = .
2
 Parseval’s Identity For Fourier Series:
Parseval’s identity states that
1 𝑙 𝑎02
∫ {𝑓(𝑥)}2 𝑑𝑥 = + ∑∞ 2 2
𝑛=1(𝑎𝑛 + 𝑏𝑛 )
𝑙 −𝑙 2
1 2𝑙 𝑛𝜋𝑥 1 2𝑙 𝑛𝜋𝑥
where 𝑎𝑛 = 𝑙 ∫0 𝑓(𝑥) cos 𝑑𝑥 and 𝑏𝑛 = 𝑙 ∫0 𝑓(𝑥) sin 𝑑𝑥
𝑙 𝑙

 Complex form of Fourier series:


In complex notation, the Fourier series and Fourier coefficients can be written
as

𝑖𝑛𝜋𝑥
𝑓(𝑥) = ∑ 𝐶𝑛 𝑒 𝑙
𝑛=−∞
𝑙 𝑖𝑛𝜋𝑥
1 −
where, taking 𝑐 = −𝑙, 𝐶𝑛 = ∫ 𝑓(𝑥)𝑒 𝑙 𝑑𝑥
2𝑙 −𝑙

Some Well Used Formulas:


1. cos n = (− 1)
n

2. sin n = 0
3. cos 2n = 1
4. sin 2n = 0
5. e i = cos  + i sin 
6. cos(n  x) = (−1)n cos x
7. sin(n + x) = (−1)n sin x
8. sin(n − x) = (−1)n+1 sin x
 a
9.  f ( x) dx =  0
a
2 f ( x) dx , f ( x) even
−a 
 0 , f ( x) odd
10. 2sin A sin B = cos(A − B) − cos(A + B)
11. 2sin A cos B = sin(A + B) + sin(A − B)

Special formula to evaluate the integral of the form  x n f ( x) dx :

 uv dx = u  v − u  v + u  v −
1 2 Continue upto ‘’ u ‘’ will be constant.

Mohammad Abdul Halim, Assistant Professor in Mathematics


Page 225 of 249

Worked-Out Mathematical Problems


Problem 01: Find the Fourier series of f ( x) = x 2 , −   x   .
Solution:
Given that,
f ( x) = x 2 , −   x  
Calculation for Fourier Coefficients:
Taking n = 0 we get,

1
a0 =
 − f ( x) dx

1
= x
2
dx
 −

2
 0
= x 2
dx [Since f(x) is even function]


2  x3 
= 
  3  0
2 
=   x 3 
3 0

 ( 3 − 0 )
2
=
3
2 2
=
3
Again,

1
an =
 
−
f ( x)cos nx dx

1
 −
= x 2
cos nx dx

2
= 0 x
2
cos nx dx [Integrand is an even function]


2
 0
= x 2
cos nx dx

2  2  sin nx   − cos nx   − sin nx  
=  x  − ( 2x)   + ( 2) 
   n   n
2
  n
3 
0

2  x2 sin nx 2 x cos nx 2sin nx 
=  + −
  n n2 n3 
0

Mohammad Abdul Halim, Assistant Professor in Mathematics


Page 226 of 249

2  2 ( −1)  
n

=   0 + − 0  − ( 0 + 0 − 0 ) 
  n2 
 
4 ( −1)
n

=
n2

1
And bn =
 − f ( x)sin nx dx

1
 −
= x 2
sin nx dx

1
= 0 [ Integrand is an odd function]

=0
Therefore, the Fourier series of f(x) is,
a0 
f ( x) = +  ( an cos nx + bn sin nx )
2 n =1
 
4 ( −1) 
n
2
= +  cos nx + 0 
3  n 2

n =1
 
( −1)
n
2 
= + 4 cos nx (As desired)
3 n =1 n2
 − x , −   x  0
Problem 02: Find the Fourier series of f ( x) =  .
 + x , 0  x  
Solution:
Given that,
 − x , −   x  0
f ( x) = 
 + x , 0  x  
Replacing x by − x in the above equation, we get
 − (− x ) , −   − x  0
f (− x) = 
 + (− x ) , 0  − x  
 + x ,   x  0
f (− x) = 
 − x , 0  x  −
 − x , −   x  0
f (− x) = 
 + x , 0  x  
f (− x) = f ( x)
Therefore f (x ) is an even function.
Calculation for Fourier Coefficients:
Taking n = 0 we get,

1
a0 =
 − f ( x) dx

Mohammad Abdul Halim, Assistant Professor in Mathematics


Page 227 of 249


2
=
 0 f ( x) dx [ Since f(x) is even function]


2
=
 0 ( + x ) dx

2  x2 
=   x + 
  2 0
2  2 
=   2 + − 0 
  2 
2 3 2
= 
 2
= 3
Again,

1
an =
 −
 f ( x)cos nxdx

2
=
 0 f ( x)cos nxdx [Integrand is an even function]


2
=
 0 ( + x ) cos nxdx

2 
2
=
  cos nxdx +
0
 0 x cos nxdx
 
2
= 2 cos nxdx +
0
 0 x cos nxdx
 
 sin nx  2   sin nx   − cos nx  
= 2  + ( x )   − (1)   
 n 0    n   n 0
2

 
 sin nx  2  x sin nx cos nx 
= 2  +  +
 n 0   n n2  0
2  ( −1)   1  
n

= 2 ( 0 − 0 ) +  0 + 2  −  0 + 2  
  n   n 
 
2  ( −1) 1 
n

=  2 − 2
  n n 
2
= 2 ( −1) − 1
n
n
( )

Mohammad Abdul Halim, Assistant Professor in Mathematics


Page 228 of 249

And

1
bn =
 
−
f ( x)sin nx dx

1
= 0 [Integrand is an odd function]

=0
Therefore, the Fourier series of f(x) is,
a0 
f ( x) = +  ( an cos nx + bn sin nx )
2 n =1

=
3   2
2
+   2 ( −1) − 1 cos nx + 0 
n =1  n 
n 

 
3 2
 

1
= +  2 ( −1) − 1 cos nx
n
(As desired)
2  n =1 n
Problem 03: Find the Fourier series of 𝒇(𝒙) = 𝒙, −𝝅 ≤ 𝒙 ≤ 𝝅.
Solution: Given 𝑓(𝑥) = 𝑥; Here, 𝑓(−𝑥) = −𝑥 = −𝑓(𝑥)
Therefore 𝑓(𝑥) is an odd function. We have, the Fourier series of odd function
𝑛𝜋𝑥
𝑓(𝑥) = ∑∞ 𝑛=1 𝑏𝑛 sin ……….(i)
𝑙
2 𝜋 𝑛𝜋𝑥 2 𝜋
Here, 𝑏𝑛 = ∫0 𝑓(𝑥) sin 𝑑𝑥 = ∫0 𝑥 sin 𝑛𝑥 𝑑𝑥
𝜋 𝜋 𝜋
2 𝑥 cos 𝑛𝑥 sin 𝑛𝑥 𝜋 2 𝜋 cos 𝑛𝜋
= [− + ] = [− ]
𝜋 𝑛 𝑛2 0 𝜋 𝑛
2 2
= − (−1)𝑛 = (−1)𝑛+1 [∵ sin 𝑛𝜋 = 0 and cos 𝑛𝜋 = (−1)𝑛 ]
𝑛 𝑛
2 𝑛𝜋𝑥
Now from(i) we get, 𝑓(𝑥) = ∑∞
𝑛=1 (−1)
𝑛+1
sin
𝑛 𝜋
(−1)𝑛+1
= 2 ∑∞
𝑛=1 sin 𝑛𝑥
𝑛
sin 𝑥 sin 2𝑥 sin 3𝑥
= 2[ − + − ⋯] .
1 2 3
Which is the required series.
(As desired)
Problem 04: Find the Fourier series of 𝑓(𝑥) = 𝑒 𝑥 , −𝜋 ≤𝑥 ≤𝜋
Solution:
Given 𝐹(𝑥) = 𝑒 𝑥
We have, the Fourier series
𝑎 𝑛𝜋𝑥 𝑛𝜋𝑥
𝑓(𝑥) = 0 + ∑∞ 𝑛=1 (𝑎𝑛 cos + 𝑏𝑛 sin )
2 𝜋 𝜋
𝑎0
= + ∑∞
𝑛=1(𝑎𝑛 cos 𝑛𝑥 + 𝑏𝑛 sin 𝑛𝑥) ……….(i)
2
1 𝜋 1 𝜋 𝑒 𝜋 −𝑒 −𝜋
Here, 𝑎0 = ∫−𝜋 𝑓(𝑥) 𝑑𝑥 = ∫−𝜋 𝑒 𝑥 𝑑𝑥 = …..(ii)
𝜋 𝜋 𝜋
1 𝜋 𝑛𝜋𝑥 1 𝜋 𝑥
𝑎𝑛 = ∫−𝜋 𝑓(𝑥) cos 𝑑𝑥 = ∫−𝜋 𝑒 cos 𝑛𝑥 𝑑𝑥
𝜋 𝜋 𝜋
1 𝑒 (cos 𝑛𝑥+𝑛 sin 𝑛𝑥) 𝜋
𝑥
= [ ]
𝜋 12 +𝑛2 −𝜋
1
= [𝑒 𝜋 (cos 𝑛𝜋 + 𝑛 sin 𝑛𝜋) − 𝑒 −𝜋 (cos(−𝑛𝜋) + 𝑛 sin(−𝑛𝜋))]
𝜋(1+𝑛2 )
1
= [𝑒 𝜋 (−1)𝑛 − 𝑒 −𝜋 (−1)𝑛 ]
𝜋(1+𝑛2 )
(−1)𝑛
= (𝑒 𝜋 − 𝑒 −𝜋 ) ……(iii) [∵ sin 𝑛𝜋 = 0 and cos 𝑛𝜋 = (−1)𝑛 ]
𝜋(1+𝑛2 )
𝜋 𝜋
1 𝑛𝜋𝑥 1
𝑏𝑛 = ∫ 𝑓(𝑥) sin 𝑑𝑥 = ∫ 𝑒 𝑥 sin 𝑛𝑥 𝑑𝑥
𝜋 𝜋 𝜋
−𝜋 −𝜋

Mohammad Abdul Halim, Assistant Professor in Mathematics


Page 229 of 249

𝜋
1 𝑒 𝑥 (sin 𝑛𝑥−𝑛 cos 𝑛𝑥)
= [ ]
𝜋 12 +𝑛2 −𝜋
1
= [𝑒 𝜋 (sin 𝑛𝜋 − 𝑛 cos 𝑛𝜋) − 𝑒 −𝜋 (sin(−𝑛𝜋) − 𝑛 cos(−𝑛𝜋))]
𝜋(1+𝑛2 )
1
= [𝑛𝑒 𝜋 (−1)𝑛+1 − 𝑛𝑒 −𝜋 (−1)𝑛+1 ]
𝜋(1+𝑛2 )
(−1)𝑛+1
= 𝑛(𝑒 𝜋 − 𝑒 −𝜋 ) ……(iii) [∵ sin 𝑛𝜋 = 0 and cos 𝑛𝜋 = (−1)𝑛 ]
𝜋(1+𝑛2 )
Now from(ii) using (iii) and (iv) we get,
𝑒 𝜋 −𝑒 −𝜋 (−1)𝑛 (−1)𝑛+1
𝑓(𝑥) = + ∑∞
𝑛=1 (𝑒 𝜋 − 𝑒 −𝜋 ) cos 𝑛𝑥 + 𝑛(𝑒 𝜋 − 𝑒 −𝜋 ) sin 𝑛𝑥
2𝜋 𝜋(1+𝑛2 ) 𝜋(1+𝑛2 )

𝑒 𝜋 −𝑒 −𝜋 (𝑒 𝜋 −𝑒 −𝜋 ) (−1)𝑛
= + ∑∞
𝑛=1 (cos 𝑛𝑥 − 𝑛 sin 𝑛𝑥)
2𝜋 𝜋 (1+𝑛2 )

𝑒 𝜋 −𝑒 −𝜋 1 (−1)𝑛
= [ + ∑∞
𝑛=1 (cos 𝑛𝑥 − 𝑛 sin 𝑛𝑥)] (As desired)
𝜋 𝜋 (1+𝑛2 )

x ,0  x  
Problem 05: Find the Fourier series of f ( x) =  .
0 ,   x  2
Solution:
Given that,
x ,0  x  
f ( x) = 
0 ,   x  2

Calculation for Fourier Coefficients:


2
1
a0 =
 0 f ( x) dx
 2
1  
=    f ( x) dx +  f ( x) dx 
  0 


 2
1  
=    x dx +  0 dx 
 0  

1  
=    x dx + 0 
 0 

1
=   x dx
 0

1  x2 
= 
  2  0
1 
=   x 2 
2 0

 ( 2 − 0 )
1
=
2

=
2
Again,

Mohammad Abdul Halim, Assistant Professor in Mathematics


Page 230 of 249

2
1
an =
 0 f ( x)cos nx dx
 2
1 
=   f ( x)cos nx dx +  f ( x)cos nx dx 
 0  
 2
1 
=   x.cos nx dx +  0.cos nx dx 
 0  

1
=   x cos nx dx
 0

1   sin nx   − cos nx  
=  ( x )   − (1)  
   n   n
2
0

1  x sin nx cos nx 
=  +
  n n 2  0
1  ( −1)   1  
n

=   0 + 2  −  0 + 2  
  n   n 

1  ( −1) 1 
n

=  2 − 2 
  n n 

=
1
n
2
 ( −1) − 1
n
 
And ,
2
1
bn =
 0 f ( x)sin nx dx
 2
1 
=   f ( x)sin nx dx +  f ( x)sin nx dx 
 0  
 2
1 
=   x sin nx dx +  0.sin nx dx 
 0  

1 
=   x sin nx dx + 0 
 0 

1
=   x sin nx dx
 0

1   − cos nx   − sin nx  
=  ( x )   − (1)  
   n   n
2
0

1  sin nx x cos nx 
=  −
  n2 n 
0

Mohammad Abdul Halim, Assistant Professor in Mathematics


Page 231 of 249

1   ( −1)  
n

=   0 −  − ( 0 − 0 )
  n 
 

1  ( −1)
n

=− 
 n
( −1)
n

=−
n
Therefore, the Fourier series of the function f ( x ) is,
a0 
f ( x) = +  ( an cos nx + bn sin nx )
2 n =1
   1 ( −1) 
 
n

= +    2 ( −1) − 1  cos nx − sin nx 
n
(As desired)
4 n =1    n  n 

Problem 06: Find the Fourier series of f ( x) = x ; −   x   .
2

Solution: Given that,f ( x) = x 2 ; −   x  


Here, Period, 2l =  − ( − ) = 2  l =  .
The Fourier coefficients are,
l
1
a0 =  f ( x) dx
l −l

1
 −
= x 2
dx

2
 0
= x 2
dx [Since integrand is an even function]


2  x3 
=
  3  0
2 3 
= x 
3   0
=
2 3
3
( − 0 )
2 2
=
3
l
1 n x
Again, an =  f ( x) cos dx
l −l l

1
 −
= x 2
cos nx dx

Mohammad Abdul Halim, Assistant Professor in Mathematics


Page 232 of 249


2
= 0 x
2
cos nx dx [Since integrand is an even function]


2  2  sin nx   − cos nx   − sin nx  
=  x  − ( 2x)   + ( 2) 
   n   n
2
  n
3 
0


2  x2 sin nx 2 x cos nx 2sin nx 
= + −
  n n2 n3 
0
2  2 (−1)n 
= − 0
  n 2

2 2 (−1) n
= 
 n2
4( −1) n
=
n2
l
1 n x
l −l
And bn = f ( x )sin dx
l

1
= x
2
sin nx dx
 −
1
= 0 [Since integrand is an odd function]

=0
Therefore, the Fourier series of f ( x ) is,
a0   n x n x 
f ( x) = +   an cos + bn sin 
2 n =1  l l 
2  4 ( −1)n
 
= +  cos nx + 0 
3  n2 
n =1
 
( −1)
n
2 
= + 4 cos nx (As desired).
3 n =1 n2
Problem 07: Find the Fourier series of f ( x) = x + x 2 , −   x   .
Solution: Given that,f ( x) = x + x 2 , −   x  
Here, Period, 2l =  − ( − ) = 2  l =  .
The Fourier coefficients are,
l
1
a0 =  f ( x) dx
l −l

1
= − (x + x ) dx
2

Mohammad Abdul Halim, Assistant Professor in Mathematics


Page 233 of 249

 
1  

=   x dx +  x2 dx 
 − − 


1  

= 0 + 2 x dx 
2
  0 


2
 0
= x dx 2


2  x3 
=
  3  0
2 3 
= x 
3   0
=
3
( − 0 )
2 3

2 2
=
3
l
1 n x
Again, an =  f ( x) cos dx
l −l l

1
 −
= ( x + x ) cos nx dx 2

 
1  
=   x cos nx dx +  x cos nx dx 
2

 − − 

1  
= 0 + 2 x cos nx dx 
2

  0 

2
 0
= x 2
cos nx dx

2  2  sin nx   − cos nx   − sin nx  
=  x  − ( 2x)   + ( 2) 
   n   n
2
  n
3 
0

2  x2 sin nx 2 x cos nx 2sin nx 
= + −
  n n2 n3 
0
2  2 (−1)n 
= − 0
  n 2

2 2 (−1) n
= 
 n2
4( −1) n
=
n2

Mohammad Abdul Halim, Assistant Professor in Mathematics


Page 234 of 249

l
1 n x
And bn = 
l −l
f ( x)sin
l
dx

1
 −
= ( x + x ) sin nx dx
2

 
1  
=   x sin nx dx +  x sin nx dx 
2
 − − 

1  
= 2 x sin nx dx + 0
  0 

2
=
 0 x sin nx dx

2  − cos nx   − sin nx  
= ( x )   − (1)  
  n   n
2
0

2  − x cos nx sin nx 
= +
  n n 2  0
2  − (−1)n 
=
  n 

−2(−1) n
=
n
Therefore, the Fourier series of f ( x ) is,
a0   n x n x 
f ( x) = +   an cos + bn sin 
2 n =1  l l 
2  4 ( −1)n

2(−1) n 
= +  cos nx − sin nx  (As desired).
3  n2 n 
n =1
 
Problem 08: Find the Fourier series of f ( x) = sin ax , −   x   .
Solution: Given that, f ( x) = sin ax , −   x  
Here, Period, 2l =  − ( − ) = 2  l =  .
The Fourier coefficients are,
l
1
a0 =  f ( x) dx
l −l

1
=
  sin ax dx
−
1
= 0 [Since integrand is an odd function]

=0

Mohammad Abdul Halim, Assistant Professor in Mathematics


Page 235 of 249

l
1 n x
Again, an = 
l −l
f ( x) cos
l
dx

1
=
 −
 sin ax cos nx dx
1
= 0 [Since integrand is an odd function]

=0
l
1 n x
And bn =  f ( x)sin dx
l −l l

1
=
 − sin ax sin nx dx

2
=
 0 sin nx sin ax dx [Since integrand is an even function]


1
=
 0 cos(nx − ax) − cos(nx + ax)dx

1
=
 0 cos(n − a) x − cos(n + a) xdx

1  sin(n − a) x sin(n + a) x 
= −
 
n−a n + a  0
1  sin( n − a) sin( n + a) 
=  −
  n−a n + a 
1  sin(n − a ) sin(n + a ) 
=  − 
  n−a n+a
1  (−1)n+1 sin a (−1)n sin a 
=  −
 n−a n + a 
1  (−1)n+1 sin a (−1)n+1 sin a 
= +
  n−a n+a 

1 1 1 
=  +  (−1) n +1 sin a
 n − a n + a
2n
= (−1) n +1 sin a
 (n − a 2 )
2

Therefore, the Fourier series of f ( x ) is,


a 
 n x n x 
f ( x) = 0 +   an cos + bn sin 
2 n =1  l l 

Mohammad Abdul Halim, Assistant Professor in Mathematics


Page 236 of 249


 2n 
= 0 +  0 + (−1) n +1 sin a sin nx 
n =1   (n − a )
2 2

2sin a 
n sin nx
=


n =1
(−1) n +1 2
n − a2
(As desired).

−2 , −2  x  0
Problem 09: Find the Fourier series of f ( x) =  .
 0 , 0 x2
Solution:
Given that,
−2 , −2  x  0
f ( x) = 
 0 , 0 x2
Calculation for Fourier Coefficients:
Here 2l = 2 − ( −2) = 4  l = 2
Now,
l
1
a0 =  f ( x) dx
l −l
2
1
=  f ( x) dx
2 −2
1 
0 2
=   f ( x) dx +  f ( x) dx 
2  −2 0 
1 
0 2
=   −2 dx +  0. dx 
2  −2 0 
1 
0
=   −2 dx + 0 
2  −2 
0
1
=  −2  dx
2 −2

= −  x −2
0

= − ( 0 − 2)
=2
Again,

n x
l
1
l −l
an = f ( x )cos dx
l
n x
2
1
=  f ( x)cos dx
2 −2 2
1 n x n x 
0 2
=   f ( x)cos dx +  f ( x)cos dx 
2  −2 2 0
2 

Mohammad Abdul Halim, Assistant Professor in Mathematics


Page 237 of 249

1 n x n x 
0 2
=   −2cos dx +  0.cos dx 
2  −2 2 0
2 
1 n x 
0
=   −2cos dx + 0 
2  −2 2 
n x
0
1
=  −2  cos dx
2 −2
2
n x
0
= −  cos dx
−2
2
 n x 
0

 sin 2 
= −
n 
 
 2  −2
 n x 
0
2
=− sin 2 
n −2

2
=− (0 − 0) = 0
n
And

n x
l
1
bn =  f ( x)sin dx
l −l l
n x
2
1
= 
2 −2
f ( x)sin
2
dx

1 n x n x 
0 2
=   f ( x)sin dx +  f ( x)sin dx 
2  −2 2 0
2 
1 n x n x 
0 2
=   f ( x)sin dx +  f ( x)sin dx 
2  −2 2 0
2 
1 n x n x 
0 2
=   −2.sin dx +  0.sin dx 
2  −2 2 0
2 
1 n x 
0
=   −2.sin dx + 0 
2  −2 2 
n x
0
1
=  −2  sin dx
2 −2
2
n x
0
= −  sin dx
−2
2

Mohammad Abdul Halim, Assistant Professor in Mathematics


Page 238 of 249

n x 
0

 − cos 2 
= − 
n
 
 2  −2
n x 
0
2 
=  cos
n  2  −2
2
= (1 − cos n )
n
=
2
n
(
1 − ( −1)
n
)
Therefore, the Fourier series of the function f(x) is,
2  
f ( x) = +   0.cos nx +
2 n =1 
2
n
 
1 − ( −1) sin nx 
n

 

 2 
f ( x) = 1 +   1 − ( −1) sin nx 
n
(As desired)
n =1  n 
H.W:
Find the Fourier series of the following functions:
i) 𝑓(𝑥) = 𝑒 −𝑥 , −𝜋 ≤𝑥 ≤𝜋 sin x , 0  x  
xi) f ( x) = 
0 ,   x  2
ii) 𝑓(𝑥) = sin 𝑝𝑥 , −𝜋 ≤𝑥 ≤𝜋 xii) f ( x) = e− x , 0  x  2
iii) 𝑓(𝑥) = cos 𝑝𝑥 , −𝜋 ≤𝑥 ≤𝜋  − x 
2

xiii) f ( x) =   , 0  x  2
 2 
iv) 𝑓(𝑥) = |𝑥|, −𝜋 <𝑥 <𝜋 −2 , −2  x  0
xiv) f ( x) = 
2 , 0  x  2
0 , −  x  0 −1 , −1  x  0
 xv) f ( x) = 
v) f ( x) =   x x , 0  x  1
 4 , 0  x  
 2x xvi) f ( x) = x 2 , −1  x  0
1 +  , −  x  0
vi) f ( x) = 
1 − 2 x , 0  x  
 
− , −  x  0 xvii) f ( x) = x, 0  x  2
vii) f ( x) = 
 x ,0  x  
0 , −  x  0 1 1
viii) f ( x) =   4 − x ,0  x 
x , 0  x   xviii) f ( x) =  2
x − 3 1
,  x 1
 4 2

Mohammad Abdul Halim, Assistant Professor in Mathematics


Page 239 of 249

0 , −  x  0 x , 0  x  4
ix) f ( x) =  xix) f ( x) = 
1 , 0  x   8 − x , 4  x  8
x) f ( x) = x sin x , −  x  

Half Range Fourier Series

Problem-10: Find the Half-Range Fourier sine series of f ( x) = eax ; 0  x  


Solution: Given that f ( x) = eax ;0  x  
To obtain the half range Fourier sine series consider f ( x ) as an odd function and extending the interval
as −  x  
where, Period, 2l =  − (− ) = 2  l =  .
For half range Fourier sine series the coefficients are,

1
a0 =
 −
 f ( x) dx = 0 [Since f ( x ) is odd function]

Again,

1
an =
 
−
f ( x) cosnx dx = 0 [Since f ( x ) is odd function]


1 n x
 −
and bn = f ( x)sin dx


1
=
 
−
f ( x)sin nx dx

1
=  2  f ( x)sin nx dx [Since integrand is an even function]
 0

2
=
 0 f ( x)sin nx dx

2
 0
= e ax
sin nx dx

2  eax (a sin nx − n cos nx) 
=
  a 2 + n2 
0
2 
=  e ax
( a sin nx − n cos nx ) 
 (a 2 + n 2 )  0
2
= 2 
e a (a sin n − n cos n ) − (0 − n) 
 (a + n )
2

Mohammad Abdul Halim, Assistant Professor in Mathematics


Page 240 of 249

2
= {e a {0 − n (−1) n } + n}
 (a + n )
22

2
= {n − n (−1) n e a }
 (a + n 2 )
2

Therefore, the half range Fourier sine series of f ( x ) is,



 n x 
f ( x) = 0 +   0 + bn sin
n =1   

2
= 2 
 n − n (−1)n ea  sin nx
n =1  (a + n )
2

2  {n − n (−1)n ea }
=  sin nx (As desired).
 n=1 a 2 + n2
Problem-11: Find the Half-Range Fourier sine and cosine series of f ( x) = x ; 0  x  2 .
Solution: Given that, f ( x) = x ; 0  x  2
To obtain the half range Fourier sine series consider f ( x ) as an odd function and extending the interval
as −2  x  2
where, Period, 2l = 2 − (−2) = 4  l = 2.
For half range Fourier sine series the coefficients are,
2
1
2 −2
a0 = f ( x) dx = 0 [Since f ( x ) is odd function]

Again,
n x
2
1
an = 
2 −2
f ( x) cos
2
dx = 0 [Since f ( x ) is odd function]

2
1 n x
and bn =  f ( x)sin dx
2 −2 2
2
1 n x
=  f ( x)sin dx
2 −2 2
2
1 n x
=  2  f ( x)sin dx [Since integrand is an even function]
2 0 2
n x
2
2
=  x sin dx
20 2
n x
2
=  x sin dx
0
2
2
  −2 n x   −2 2 n x 
= ( x )  cos  − (1)  . .sin 
  n 2   n n 2 0

Mohammad Abdul Halim, Assistant Professor in Mathematics


Page 241 of 249

 −2 x n x n x 
2
4
= cos + 2 2 sin
 n 2 n 2  0
 −4 4  
=  cos n + 2 2 sin n  − 0 
 n n  
−4(−1) n
= +0
n
−4(−1) n
=
n

Therefore, the half range Fourier sine series of f ( x ) is,


a0   n x n x 
f ( x) = +   an cos + bn sin 
2 n =1  l l 

 n x 
= 0 +   0 + bn sin 
n =1  2 

n x
=  bn sin
n =1 2
−4(−1)n

n x
= sin
n =1 n 2

−4(−1) n
n x
= sin (As desired).
n =1 n 2
Again, for half range Fourier cosine series consider f ( x ) as an even function and extending the interval
as −2  x  2
where, Period, 2l = 2 − (−2) = 4  l = 2.
2
1
a0 =  f ( x) dx
2 −2
2
2
=  f ( x) dx [Since f ( x ) is odd function]
20
2
= 1. x dx
0
2
=  x dx
0
2
 x2 
= 
 2 0
 22 
= − 0
 2 
=2

Mohammad Abdul Halim, Assistant Professor in Mathematics


Page 242 of 249

2
1 n x
Again an = 
2 −2
f ( x) cos
2
dx
2
2 n x
=  f ( x) cos dx [Since f ( x ) is even function]
20 2
2
n x
= 1. f ( x) cos dx
0
l
n x
2
=  x cos dx
0
2
n x
2
=  x cos dx
0
2
2
  2 n x   2 −2 n x 
= ( x )  sin  − (1)  . .cos 
  n 2   n n 2 0
n x n x 
2
 2x 4
= sin + 2 2 cos
 n 2 n 2  0
 4 4  4
=  sin n + 2 2 cos n  − 2 2
 n n  n
 4(−1)n  4
= 0 + 2 2  − 2 2
 n  n
4(−1) n 4
= − 2 2
n 2 2
n
n x
2
1 1
and bn =  f ( x)sin dx =  0 = 0 [Since integrand is an odd function]
2 −2 2 2
Therefore, the half range Fourier cosine series of f ( x ) is,
a0   n x n x 
f ( x) = +   an cos + bn sin 
2 n =1  l l 
2   n x 
= +   an cos + 0
2 n =1  l 

 4(−1)n 4  n x
= 1 +   2 2 − 2 2  cos
n =1  n  n  2
4   (−1)n 1  n x
= 1 + 2   2 − 2  cos (As desired).
 n=1  n n  2
Problem-12: Find the Fourier sine series of f ( x) = cos x ; 0  x  
Solution: Given that f ( x) = cos x ; 0  x  
To obtain the half range Fourier sine series consider f ( x ) as an odd function and extending the interval
as −  x  

Mohammad Abdul Halim, Assistant Professor in Mathematics


Page 243 of 249

where, Period, 2l =  − (− ) = 2 l =  .

For the Fourier sine series the coefficients are,



1
a0 =
 
−
f ( x) dx = 0 [Since f ( x ) is odd function]

Again,

1
an =
 
−
f ( x) cosnx dx = 0 [Since f ( x ) is odd function]


1 n x
 −
and bn = f ( x)sin dx


1
=
 
−
f ( x)sin nx dx

1
=  2  f ( x)sin nx dx [Since integrand is an even function]
 0

2
=
 0 f ( x)sin nx dx

2
=
 0 cos x sin nxdx

1
=
 0 {sin(nx + x) + sin(nx − x)}dx

1
=
 0 {sin(n +1) x + sin(n −1) x}dx

1  − cos(n + 1) x − cos(n − 1) x 
= + ;n 1
  n +1 n −1 
0

1   cos(n + 1)
cos(n − 1)   1 1 
=  − −  − − − 
  n +1 n − 1   n + 1 n − 1
1   cos( + n ) cos( − n )  1 1 
=  − − + + 
  n +1 n −1  n + 1 n − 1
1   − cos n
− cos n  1 1 
=  − − + +
  n + 1 n − 1  n + 1 n − 1 
1  (−1)n (−1)n 1 1 
=  + + +
  n + 1 n − 1 n + 1 n − 1
1  n(−1)n − (−1)n + n(−1)n + (−1)n + n − 1 + n + 1
=
  n2 − 1 

Mohammad Abdul Halim, Assistant Professor in Mathematics


Page 244 of 249

2n(−1)n + 2n
=
 (n2 − 1)
2n{1 + (−1) n }
=
 (n2 − 1)
Therefore, the Fourier sine series of f ( x ) is,
a0   n x n x 
f ( x) = +   an sin + bn cos 
2 n=2  l l 
a 
 n x n x 
= 0 +   an sin + bn cos 
2 n=2  l l 
2n{1 + (−1)n }

= sin nx
n=2  (n2 − 1)
2  {1 + (−1)n }
=  sin nx ; n  1 (As desired).
 n=2 n2 − 1
 
1 , 0  x  2
Problem 13: Find the half range Fourier series of f ( x) =  .
0 ,   x  
 2
Solution:
Given that,
 
1 , 0  x  2
f ( x) = 
0 ,   x  
 2
Here f ( x ) is defined on ( 0,  ) .we extent f ( x ) over ( − , 0 ) as an even function.
Therefore bn = 0 .
Here,

2
a0 =
 0 f ( x) dx
 2  
2 
=   f ( x) dx +  f ( x) dx 
 0  
 2 

  
22 
=  1dx +  0 dx 
 0  
 2 

2 
=   x 02 + 0 
 

Mohammad Abdul Halim, Assistant Professor in Mathematics


Page 245 of 249

2 
=  =1
 2

2
And an =  f ( x)cos nx dx
 0

 2  
2 
=   f ( x)cos nx dx +  f ( x)cos nx dx 
 0  
 2 

  
22 
=  1.cos nx dx +  0.cos nx dx 
 0  
 2 

2
2
=
 0 cos nx dx

2  sin nx  2
= 
  n  0

2
=  sin nx 02
n
2  n 
=   sin − 0
n  2 
2 n
= sin
n 2
Therefore, the Fourier series of even function f ( x) is,
a0 
f ( x) = +  ( an cos nx + bn sin nx )
2 n =1
1   2 n 
= +  sin cos nx + 0.sin nx 
2 n =1  n 2 
1 
 2 n 
= +  sin cos nx 
2 n =1  n 2 

Again, We extent f ( x ) over ( − , 0 ) as an odd function.


Therefore, a0 = 0 and an = 0
And

2
bn =
  f ( x) sin nx dx
0

Mohammad Abdul Halim, Assistant Professor in Mathematics


Page 246 of 249

 

22 
=   f ( x)sin nx dx +  f ( x)sin nx dx 
 0  
 2 

 

22 
=   1.sin nx dx +  0.sin nx dx 
 0  
 2 

 
22 
=   sin nx dx + 0 
 0 
 

2
2
=
  sin nx dx
0

2  − cos nx  2
= 
  n  0

2
=−   cos nx 02
n
2  n 
=−   cos − 1
n  2 
2  n 
=   1 − cos 
n  2 
Therefore, the Fourier series of Odd function f ( x ) is,
a0 
f ( x) = +  ( an cos nx + bn sin nx )
2 n =1
0   2  n  
= +   0.cos nx + 1 − cos  sin nx 
2 n =1  n  2  
 2 2 n  


=   2sin  sin nx 
n =1  n  4  

 4  2 n  
=   sin  sin nx  (As desired)
n =1  n  4  
H.W:
Find the Fourier series of the following functions

 
 x , 0  x  2
(a) f ( x) = 
 − x ,   x  
 2
(b) f ( x) = e , 0  x  
x

Mohammad Abdul Halim, Assistant Professor in Mathematics


Page 247 of 249

(c) f ( x) =  − x, 0  x  
(d) f ( x) = x 4 , 0  x  

Complex Fourier Series

0 , −  x  0
Problem 14: Find the complex Fourier series of f ( x) =  .
1 , 0  x  
Solution: Given that,
0 , −  x  0
f ( x) = 
1 , 0  x  

Here, Period, 2l =  − ( − ) = 2  l = 
The Fourier coefficients for complex form are,

1
 f ( x)e
− inx
Cn = dx
2 −

1  
0
=    f ( x)e dx +  f ( x)e−inx dx 
− inx

2  − 0 

1  
0
=    0.e−inx dx +  1.e−inx dx 
2  − 0 

1  
=  0 +  e−inx dx 
2  0 

1
2 0
=  e − inx dx


1  e−inx 
= 
2  −in  0
1 
=−   e − inx 
2ni 0

 ( e − n i − 1)
1
=−
2ni
1
=−  ( cos n − i sin n − 1)
2ni
1
= {1 − ( −1) n }
2ni
Therefore, the complex Fourier series is
 in x
f ( x) =  Cne
n =−
l


1 1
=
2i
 n {1 − (−1) }e
n =1
n inx
(As desired)

Mohammad Abdul Halim, Assistant Professor in Mathematics


Page 248 of 249

H.W:
Find the complex Fourier series of the followings:
(a) f ( x) = e5 x , −  x  
(b) f ( x) = cos x, −  x  

Mohammad Abdul Halim, Assistant Professor in Mathematics

You might also like